You are on page 1of 170

ĐẠI HỌC QUỐC GIA TP.

HCM
TRƯỜNG PHỔ THÔNG NĂNG KHIẾU

Chuyên đề
Toán học
Số 11

Tháng 12/2018
MỤC LỤC
Ban biên tập
Lời nói đầu . . . . . . . . . . . . . . . . . . . . . . . . . . . . . . . . . . . . . . . . . . . . . . . 7

Nguyễn Tăng Vũ
Phép vị tự quay cùng tâm . . . . . . . . . . . . . . . . . . . . . . . . . . . . . . . . . . . . . . . . 9

Phạm Tuấn Huy


Suy luận phản chứng trong hình học . . . . . . . . . . . . . . . . . . . . . . . . . . . . . . . . . . 21

Nguyễn Nguyễn
Các bài toán về hệ số nhị thức . . . . . . . . . . . . . . . . . . . . . . . . . . . . . . . . . . . . . 49

Nguyễn Tiến Hoàng


Một số định hướng tiếp cận lời giải đối với các bài toán Olympic . . . . . . . . . . . . . . . . . . . 67

Phan Quốc Vượng


Về bài toán tổ hợp trong đề thi chọn đội tuyển 2017 . . . . . . . . . . . . . . . . . . . . . . . . . . 85

Nguyễn Mạc Nam Trung


Về bài toán xác định hàm số trong giải phương trình hàm . . . . . . . . . . . . . . . . . . . . . . . 91

Võ Kế Huy - Bùi Duy Khang


Về phương pháp quy nạp trong giải toán hình học tổ hợp . . . . . . . . . . . . . . . . . . . . . . . 101

Trần Nguyễn Nam Hưng


Bài toán chia kẹo Euler . . . . . . . . . . . . . . . . . . . . . . . . . . . . . . . . . . . . . . . . . 113

Ban biên tập


Lời giải đề chọn đội dự tuyển 2017 . . . . . . . . . . . . . . . . . . . . . . . . . . . . . . . . . . . 125

Nguyễn Tăng Vũ - Nguyễn Ngọc Duy - Vương Trung Dũng - Lê Phúc Lữ


Lời giải để chọn đội tuyển các năm 2016 - 2017 - 2018 . . . . . . . . . . . . . . . . . . . . . . . . 131

Ban biên tập


Giới thiệu một số bài toán hay . . . . . . . . . . . . . . . . . . . . . . . . . . . . . . . . . . . . . 169

5
LỜI NÓI ĐẦU

Các bạn đang cầm trên tay Chuyên đề Toán học số 11 của trường Phổ thông Năng khiếu.
Đây là một dự án được bắt đầu từ cuối năm 2016 sau thành công của Chuyên đề Toán học số
10. Quyển số 10 đã đánh dấu kỷ niệm 20 năm thành lập trường Phổ thông Năng khiếu. Nhưng
quan trọng hơn, đó còn là đánh dấu của một sự chuyển giao giữa các thế hệ với những gương
mặt mới xuất hiện trong đội ngũ tác giả và biên tập viên. Và quyển số 11 này không nằm ngoài
dòng chảy đó. Ngoài bài viết của anh Phạm Tuấn Huy từ năm 2013, các bạn sẽ thấy rất nhiều bài
viết mà tác giả vẫn còn là học sinh của trường. Nội dung các bài viết vẫn được trải dài ở cả 4 phân
môn Hình học - Đại số - Số học - Tổ hợp, trong đó có nhiều vấn đề thú vị được tiếp cận bởi các góc
nhìn khác nhau nhưng vẫn với lối suy nghĩ tự nhiên khi giải toán của học sinh THPT. Tin rằng
bạn đọc sẽ tìm thấy nhiều điều thú vị.
Dự án này không chỉ có những thành công. Bởi đội ngũ trẻ còn thiếu kinh nghiệm mà quá
trình ra mắt của quyển chuyên đề đã gặp không ít khó khăn. Mặc dù một seminar giới thiệu đã
được tổ chức vào cuối tháng 7/2017 khi mọi việc gần hoàn tất, mọi thứ đã phải dừng lại vào 1
tháng sau đó bởi nhiều lý do khách quan. Đến tận tháng 10/2018, những phần việc còn lại mới
được bắt đầu và ban biên tập đã nhận được nhiều sự giúp đỡ hơn.
Để hoàn thành được quyển chuyên đề, không thể phủ nhận được sự đóng góp của nhiều bên
với những vai trò khác nhau. Xin gửi lời cảm ơn đến thầy Nguyễn Tăng Vũ đã gợi ý và hỗ trợ rất
nhiều về mặt chuyên môn khi dự án chỉ mới là ý tưởng cho đến khi gần hoàn tất. Cảm ơn thầy
Trần Nam Dũng đã hỗ trợ nhiệt tình những công đoạn cuối cùng, đặc biệt là khâu in ấn, và lắng
nghe rất cầu thị để các tác giả và biên tập viên có thể yên tâm sáng tạo và hoàn thiện chuyên đề.
Xin gửi lời cảm ơn đến các anh Lâm Bảo Chánh, Nguyễn Văn Huyện, Lương Văn Khải, Lê Viết Ân
và Phạm Hoàng Minh đã tham gia đánh máy và hỗ trợ về kỹ thuật. Ban biên tập cũng gửi lời cảm
ơn đến bạn Nguyễn Lê Duy, học sinh khối chuyên Anh niên khoá 2016-2019 đã thiết kế một mẫu
bìa đẹp, rất đơn giản mà hiện đại cho chuyên đề số này.
Những số đếm cơ bản là từ 1 đến 10 nên số 11 thường gợi đến sự khởi đầu mới trên một nền
tảng lâu dài sẵn có. Ban biên tập cũng tin như vậy. Mong rằng đây là bước đầu một sự phát triển
mới của Chuyên đề Toán học với những hoạt động liên tục hơn nhằm duy trì sự say mê tìm tòi
của học sinh khối Toán. Đó chính là cơ sở để họ còn đi xa hơn nữa.
Hãy mở sách ra đi, rồi cùng nhau khám phá xem “thầy trò Năng Khiếu viết gì".

7
PHÉP VỊ TỰ QUAY CÙNG TÂM
Nguyễn Tăng Vũ
Giáo viên trường Phổ thông Năng khiếu

1. ĐỊNH NGHĨA VÀ TÍNH CHẤT


Định nghĩa 1. Cho điểm O và số thực dương k và góc lượng giác α. Phép biến hình biến
điểm M thành điểm M0 sao cho SM0 = k.SM, ∠(OM, OM0 ) = α được gọi là phép vị tự
quay cùng tâm (hay còn được gọi là Spiral similarity). Kí hiệu là S(O; k; α).

Định lý 1. Cho phép biến hình S(O; k; α). Khi đó:

1. Ảnh của đường thẳng d là đường thẳng d0 sao cho ∠(d, d0 ) = α.


2. Ảnh của tam giác ABC là tam giác A0 B0 C 0 đồng dạng cùng hướng với ABC với tỉ số
đồng dạng là k.

Định lý 2. Cho phép vị tự quay cùng tâm S(O; k; α) biến A thành A0 và B thành B0 . Khi đó

1. ∆OAB ∼ ∆OA0 B0 .
2. ∆OAA0 ∼ OBB0 .

Định nghĩa 2. Ta nói hai tam giác ABC và A0 B0 C 0 là đồng dạng cùng hướng nếu
AB AC
∠( AB; AC ) = ∠( A0 B0 ; A0 C 0 ) (mod 2π ) và 0 0 = 0 0 .
AB AC
Định lý 3. Hai tam giác OAB và OA0 B0 đồng dạng cùng hướng khi và chỉ khi có một phép
vị tự quay biến AB 7→ A0 B0 .

Nhận xét.

1. Nếu có một phép vị tự quay tâm O biến AB thành A0 B0 thì cũng có một phép vị tự quay
tâm O biến AA0 thành BB0 .
2. Nếu có ∆OAB ∼ ∆OA0 B0 và ∆OAA0 ∼ ∆OBB0 thì có một phép vị tự quay tâm O
biến A 7→ A0 , B 7→ B0 .

9
A0

B0

Định lý 4. Cho các điểm A, A0 , B, B0 sao cho không có 3 điểm nào thẳng hàng.
1. Tồn tại duy nhất một phép vị tự quay S(O; k; α) biến A thành A0 và B thành B0 .
2. E là giao điểm của AB và A0 B0 ; F là giao điểm của AA0 và BB0 . Khi đó O, A, A0 , E và
O, B, B0 , E đồng viên; O, A, B, F và O, A0 , B0 , F đồng viên.

A0

B0

Định lý 5. Cho phép vị tự quay S(O; k; α) biến A thành A0 và B thành B0 . Khi đó giao điểm
thứ hai của đường tròn ngoại tiếp tam giác OAB và OA0 B0 là giao điểm của AA0 và BB0 .

2. VÍ DỤ
Ví dụ 1 (USAMO 2006). Cho tứ giác ABCD. Các điểm E, F thuộc các cạnh AD và BC sao
AE BF
cho = . Đường thẳng EF cắt các đường thẳng AB và CD lần lượt tại S và T. Chứng
AD BC
minh rằng đường tròn ngoại tiếp các tam giác SAE, SBF, TDE, TCF cùng đi qua một điểm.

10
P

S
E

C
T D

Lời giải. Xét phép vị tự quay cùng tâm S( P, k, α = ( AD; BC )) biến A 7→ B, D 7→ C.


AE BF
Mà = nên S( P; k; α) : E 7→ F.
AD BC
Vậy S( P; k; α) : AE 7→ BF, ED 7→ FC.
Ta có S = EF ∩ AB, T = EF ∩ CD nên theo định lý 3 thì các tứ giác PAES, EBFS, PFCT
và PEDT nội tiếp.

Ví dụ 2. Cho tam giác ABC nhọn có các đường cao AD, BE và CF. Chứng minh rằng
đường thẳng euler của các tam giác AEF, BDF và CEF đồng quy.

Lời giải. Xét phép vị tự quay S( E; k; α = ( EA; ED ) biến A 7→ D, F 7→ C. Khi đó trực


tâm tam giác AEF biến thành trực tâm tam giác ECD; tâm ngoại tiếp tam giác AEF
biến thành tâm ngoại tiếp tam giác ECD. Hay Ha 7→ Hc và Ma 7→ Mc .
Gọi P là giao điểm của Ha Ma và Hc Mc , ta có P thuộc ( EMa Mc ). Mà EMa Mc là đường
tròn Euler của tam giác ABC. Nên P thuộc đường tròn Euler của tam giác ABC.

11
A

Ha

E
Ma

Hc Mc

C
B D

Chứng minh tương tự ta có giao điểm của các đường Mb Hb , Mc Hc ; Mb Hb , Ma Ha cũng


thuộc đường tròn Euler của tam giác nên các giao điểm này trùng nhau. Vậy các
đường thẳng Euler của các tam giác AEF, BDF và CDE đồng quy tại một điểm thuộc
đường tròn Euler của tam giác ABC.

Ví dụ 3. Cho hai đường tròn (O) và ( I ) cắt nhau tại hai điểm A và B. Một đường thẳng
thay đổi qua A cắt (O) tại C và cắt ( I ) tại D (A nằm giữa C và D). Tiếp tuyến tại C của (O)
và tiếp tuyến tại D của ( I ) cắt nhau tại điểm E. Chứng minh rằng đường trung trực của BE
luôn tiếp xúc với một đường tròn cố định.

Lời giải. Dễ thấy tứ giác BCED nội tiếp. Ta có ∆BCD ∼ ∆BOI và ∆BCO ∼ ∆BDC.
Xét phép vị tự quay tâm B biến C 7→ O, I 7→ D và E 7→ K.
Khi đó ta cũng có ∆BKO ∼ ∆BEC, suy ra ∠ BKO = ∠ BEC = ∠ BDC = ∠ BIO. Do đó
tứ giác BOKI nội tiếp.

12
E

D
A

I
C O

Hơn nữa KE = KO và ∠EKO = ∠ DIB = 2∠EBD. Suy ra K là tâm ngoại tiếp tứ giác
BCED.
Gọi N là trung điểm BE ta có ∠ NKB = ∠EIB, suy ra NK là tiếp tuyến của ( BOI ) mà
NK là trung trực của BE nên ta có điều cần chứng minh.

Ví dụ 4 (PTNK 2013). Tam giác ABC nhọn có H là trực tâm và P là điểm di động bên trong
tam giác ABC sao cho ∠ BPC = ∠ BHC. Đường thẳng qua B và vuông góc với AB cắt PC tại
M, đường thẳng qua C và vuông góc với AC cắt PB tại N. Chứng minh trung điểm I của MN
luôn thuộc một đường thẳng cố định.

Lời giải. Dễ thấy tứ giác BCED nội tiếp. Ta có ∆BCD ∼ ∆BOI và ∆BCO ∼ ∆BDC.
Xét phép vị tự quay tâm B biến C 7→ O, I 7→ D và E 7→ K.
Khi đó ta cũng có ∆BKO ∼ ∆BEC, suy ra ∠ BKO = ∠ BEC = ∠ BDC = ∠ BIO. Do đó
tứ giác BOKI nội tiếp.

13
A
N

D
B C

D1

X M

Hơn nữa KE = KO và ∠EKO = ∠ DIB = 2∠EBD. Suy ra K là tâm ngoại tiếp tứ giác
BCED.
Gọi N là trung điểm BE ta có ∠ NKB = ∠EIB, suy ra NK là tiếp tuyến của ( BOI ) mà
NK là trung trực của BE nên ta có điều cần chứng minh.

Ví dụ 5 (PTNK 2013). Tam giác ABC nhọn có H là trực tâm và P là điểm di động bên trong
tam giác ABC sao cho ∠ BPC = ∠ BHC. Đường thẳng qua B và vuông góc với AB cắt PC tại
M, đường thẳng qua C và vuông góc với AC cắt PB tại N. Chứng minh trung điểm I của MN
luôn thuộc một đường thẳng cố định.

Lời giải. Ta có tứ giác PDMN nội tiếp, gọi X là giao điểm của đường tròn ngoại tiếp
tứ giác và đường tròn ngoại tiếp tam giác BHC, X khác phía P đối với đường thẳng
BC. Ta chứng minh H, D, X thẳng hàng.

14
A
N

D
B C

D1

X M

Ta có: ∠( XD; XP) = ∠( ND; NP) = ∠( BH; BP) = ∠( XH; XP). Suy ra X, H, P thẳng
hàng. Do đó X cố định. Khi đó ∠( NX; N M ) = ∠( DX; DM ) = ∠( DH; DB) không đổi
và ∠( XN; XM ) = ∠( PM; PN ) = ∠( AC; AB) không đổi. Vậy tam giác XMN đồng
XM XC
dạng với tam giác XCB, suy ra ∠( XM; XI ) = ∠( XC; XK ) và = không đổi.
XI XK
XC
Xét phép vị tự quay tâm X góc quay ∠( XC; EK ) tỉ số biến M thành I, mà M thuộc
XK
đường thẳng DB cố định nên I cũng thuộc một đường thẳng cố định. Kết thúc chứng
minh.

Ví dụ 6 (Việt Nam TST 2013). Cho tứ giác ABCD lồi có các cặp cạnh đối không song song
nội tiếp đường tròn (O; R). Gọi E là giao điểm của AC, BD. Đường phân giác góc ∠ AEB
cắt các đường thẳng AB, BC, CD, DA lần lượt tại M, N, P, Q.

1. Chứng minh rằng các đường tròn ( AQM), ( BMN ), (CPN ), ( DPQ) cùng đi qua một
điểm suy nhất. Gọi điểm đó là K.
2R2
2. Đặt min{ AC, BD } = m. Chứng minh rằng OK ≤ √ .
4R2 − m2

Lời giải. Gọi V là giao điểm của AB và CD, U là giao điểm của AD và BC.

1. Xét phép vị tự quay biến AB thành DC. Gọi K là tâm của phép vị tự quay đó, ta
có K thuộc đường tròn ngoại tiếp các tam giác BCV, ADV, ABU, CDU. (1)

15
MB AM
Mặt khác do M thuộc AB, P thuộc AC và = nên phép vị tự quay trên
PC DP
cũng biến AM thành DP và MB thành PC. Từ đó ta cũng có K thuộc đường
tròn ngoại tiếp các tam giác AMQ, DPQ, MBP, CPN. (2)
Từ (1) và (2) ta có các đường tròn ngoại tiếp các tam giác
AQM, BMN, CNP, DPQ cùng đi qua một điểm K.
2. Ta có ∠ AKU = ∠ ABU = ∠ ADV mà ∠ ADV + ∠ AKV = 1800 nên ∠ AKU +
∠ AKV = 180o . Do đó U, K , V thẳng hàng.
Hơn nữa ∠UKA + ∠VKC = ∠ ABU + ∠CBV = 2∠ ADC = ∠ AOC. .Do đó
AOCK nội tiếp, tương tự thì DOBK nội tiếp. Mà EA.EC = EB.ED nên E thuộc
trục đẳng phương của ( DOBK ) và ( AOCK ), từ đó O, E, K thẳng hàng và
R2
OE.OK = OA2 = R2 , suy ra OK = .
OE
Gọi
r H, T là hình chiếu r của O trên AC và BD. Ta r có OE ≥ OH, OT, mà OH =
AC 2 BD 2 m2 1√ 2
R2 − , OK = R2 − , suy ra OE ≥ R2 − = 4R − m2 .
4 4 4 2
Bài toán kết thúc.

3. BÀI TẬP
Bài 1. Cho hai đường tròn w1 và w2 cắt nhau tại P và K. XY là tiếp tuyến chung của hai
đường tròn với X thuộcw1 và Y thuộc w2 và gần P hơn K. XP cắt w2 tại C và YP cắt w1 tại
B. BX và CY cắt nhau tại A. Đường tròn ngoại tiếp tam giác AXY và ABC cắt nhau tại Q.
Chứng minh rằng ∠QXA = ∠QKP.

Bài 2. Cho hai đường tròn w và w0 cắt nhau tại A và B, đường thẳng qua B cắt w, w0 lần
lượt tại C và D. Tiếp tuyến tại C của w và tiếp tuyến tại D của w0 cắt nhau tại P. Chứng
minh rằng đường trung trực của AP luôn tiếp xúc với một đường tròn cố định.

Bài 3. Cho góc ∠xAy và điểm P cố định nằm bên trong góc. Đường tròn thay đổi qua A và
P cắt Ax và Ay lần lượt tại D và E. Chứng minh rằng trọng tâm của tam giác ADE thuộc
một đường thẳng cố định.

Bài 4. Cho tam giác ABC nội tiếp đường tròn w, X là một điểm thuộc miền trong tam giác.
AX, BX và CX lần lượt cắt w tại A0 , B0 , C 0 . Đường trung trực của XA0 cắt BC tại A1 ; các
điểm B1 , C1 được xác định tương tự. Chứng minh rằng A1 , B1 , C1 thẳng hàng.

Bài 5. Cho tam giác ABC. Về phía ngoài tam giác dựng các tam giác ABD, ACE cân tại D
và E sao cho ∠ ADB = 120◦ , ∠ AEC = 60◦ . Gọi M là trung điểm của BC. Tính số đo các
góc của tam giác DME.

Bài 6. Cho tam giác ABC. Các điểm D và E lần lượt thay đổi trên các cạnh AB, AC sao cho
AD = CE. Chứng minh rằng đường tròn ngoại tiếp tam giác ADE luôn đi qua một điểm cố
định khác A.

16
Bài 7. Cho tứ giác ABCD với BC = DA và BC không song song với DA. Cho hai điểm
thay đổi F, E lần lượt thuộc BC và DA sao cho BF = DE. Gọi P là giao điểm của AC và
BD. EF cắt BD và AC lần lượt tại Q và R. Chứng minh rằng đường tròn ngoại tiếp tam giác
PQR luôn đi qua một điểm cố định khác P.

Bài 8. Cho tam giác ABC nhọn. Tìm điểm M trong tam giác sao cho MA + MB + MC đạt
giá trị nhỏ nhất.

Bài 9. Cho hai hình vuông ABCD và AB0 C 0 D 0 cùng hướng (B 6= B0 ). Chứng minh rằng
các đường thẳng BB0 , CC 0 và DD 0 đồng quy.

Bài 10. Cho lục giác ABCDEF nội tiếp đường tròn sao cho AB = CD = EF. Gọi M, N, P
là trung điểm của AB, CD, EF. Chứng minh tam giác MNP đều.

Bài 11. Cho tam giác ABC và đường tròn tâm I nội tiếp tam giác tiếp xúc với các cạnh
BC, AC và AB tại D, E, F. Gọi D 0 , E0 , F 0 là hình điểm đối xứng của D, E, F qua I. Gọi
A0 , B0 , C 0 là trung điểm của các đoạn thẳng I A, IB, IC. Chứng minh rằng A0 D 0 , B0 E0 và
C 0 F 0 đồng quy tại một điểm.

Bài 12 (PTNK 2011). Cho tam giác ABC nội tiếp đường tròn (O) với B, C cố định còn A
thay đổi trên (O). Trung trực d của BC cắt AB, AC tại M, N. Gọi P, Q là các điểm đối xứng
của M, N qua O và K là giao điểm của BQ và CP.

1. Chứng minh luôn thuộc một đường tròn cố định.


2. Kết luận trên còn đúng không nếu thay bằng đường thẳng Euler của tam giác ABC.

Bài 13. Cho tam giác ABC cân tại A, nội tiếp đường tròn (O). Gọi (ω ) là đường tròn tiếp
xúc với AB, AC tại D và E và tiếp xúc trong với (O) tại K. Chứng minh rằng DE đi qua tâm
nội tiếp của tam giác ABC.

Bài 14. Cho hai đường tròn (O1 ) và (O2 ) cắt nhau tại A và B. C1 C2 , D1 D2 là tiếp tuyến
chung của (O1 ) và (O2 ). I1 , I2 là giao điểm của C1 D1 và C2 D2 với O1O2 . Chứng minh rằng
∠O1 AO2 = ∠ I1 AI2

Bài 15. Cho tam giác ABC nội tiếp đường tròn (O), gọi A1 , B1 , C1 lần lượt là các chân các
đường cao của tam giác dựng từ A, B, C. Gọi H1 là trực tâm tam giác A1 B1 C1 . Gọi A2 , B2 , C2
lần lượt là trung điểm của B1 C1 , A1 C1 , A1 B1 . Chứng minh rằng trung điểm I của OH1 là
tâm đường tròn nội tiếp tam giác A2 B2 C2 .

Bài 16. Cho tam giác ABC. Đường tròn tâm I nội tiếp tam giác tiếp xúc với ba cạnh BC, AC
và AB lần lượt tại A1 , B1 , C1 . Gọi H là trực tâm của tam giác A1 B1 C1 , O là tâm đường tròn
ngoại tiếp tam giác ABC. Chứng minh rằng H, I, O thẳng hàng.

17
Bài 17. Hai đường tròn (O1 ), (O2 ) tiếp xúc ngoài nhau tại C và tiếp xúc trong với (O) tại
D và E. Gọi (d) là tiếp tuyến chung của (O1 ) và (O2 ) tại C. AB là đường kính của (O) sao
cho A, D, O1 cùng phía đối với (d). Chứng minh rằng AO1 , BO2 và DE đồng quy.

Bài 18 (IMO 2008). Cho tứ giác lồi ABCD (AB khác BC). Gọi đường tròn nội tiếp của các
tam giác ABC và ADC lần lượt là (ω1 ) và (ω2 ). Giả sử tồn tại đường tròn (ω ) tiếp xúc
với tia BA về hướng A và tia BC về hướng C và tiếp xúc với các đường thẳng AD và CD.
Chứng minh rằng tiếp tuyến chung ngoài của các đường tròn (w1 ) và (w2 ) cắt nhau tại một
điểm thuộc đường tròn (C ).

Bài 19 (IMO shortlist 1998). Cho tam giác ABC. Gọi H là trực tâm và O là tâm đường
tròn ngoại tiếp tam giác. Gọi D, E, F lần lượt là điểm đối xứng của A qua BC, B qua CA và
của C qua AB. Chứng minh rằng D, E, F thẳng hàng khi và chỉ khi OH = 2R, với R là bán
kính đường tròn ngoại tiếp tam giác.

Bài 20. Cho tam giác ABC nội tiếp đường tròn tâm O và ngoại tiếp đường tròn tâm I. Gọi
A0 , B0 , C 0 lần lượt là giao điểm của AI, BI, CI với (O). ( I ) tiếp xúc với các cạnh BC, AC và
AB lần lượt tại A1 , B1 , C1 .

1. Chứng minh rằng đường thẳng Euler của tam giác A0 B0 C 0 đi qua điểm I.
2. Chứng minh rằng đường thẳng Euler của tam giác A1 B1 C1 đi qua điểm O.

Bài 21. Cho tam giác ABC nhọn có trực tâm H và tâm đường tròn ngoại tiếp là O. Đường
trung trực của AH cắt cạnh AB và AC tại D và E. Chứng minh rằng OA là phân giác của
góc ∠ DOE.

d Đường tròn thay


Bài 22 (PTNK 2009). Cho góc xbOy và điểm P cố định nằm trong góc xOy.
đổi qua O và P cắt Ox và Oy tại M, N.

1. Tìm quỹ tích trọng tâm của tam giác OMN.


2. Tìm quỹ tích trực tâm của tam giác OMN.

Bài 23. Cho tam giác nhọn ABC với AB khác AC. Gọi H là trực tâm của tam giác ABC,
M là trung điểm cạnh BC. Lấy D trên cạnh AB và E trên cạnh AC sao cho AD = AE và
D, H, E thẳng hàng. Chứng minh rằng đường thẳng MH vuông góc với dây cung chung của
đường tròn ngoại tiếp các tam giác ADE và ABC.

Bài 24 (Mối quan hệ giữa phép vị tự và nghịch đảo). Cho tam giác ABC, đường tròn
tâm I nội tiếp tam giác và tiếp xúc với các cạnh BC, AC và AB tại D, E, F. Chứng minh rằng
trực tâm của tam giác DEF thuộc đường thẳng nối tâm nội tiếp I và tâm ngoại tiếp O của
tam giác ABC.

Bài 25. Cho hình bình hành ABCD và hai điểm A1 , C1 trên các cạnh AB, BC tương ứng.
Các đường thẳng CA1 và AC1 cắt nhau tại P. Giả sử đường tròn ngoại tiếp tam giác AA1 P
và CC1 P giao nhau tại P trong tam giác ACD. Chứng minh rằng ∠ PDA = ∠QBA.

18
19
SUY LUẬN PHẢN CHỨNG TRONG HÌNH HỌC
Phạm Tuấn Huy
Lớp chuyên Toán khoá 2011 - 2014

GIỚI THIỆU. Bài viết được hoàn thành sau khi tác giả đạt huy chương vàng tại
Olympic Toán Quốc tế năm 2013 và từng được sử dụng để giảng dạy cho đội dự
tuyển trường PTNK. Dù được viết đã lâu, BBT vẫn cho rằng đây là bài viết giá trị về
hình học, thể hiện cách tiếp cận vấn đề tự nhiên và lý thú.

1. Cơ sở lý luận
Phản chứng là một phương pháp thú vị và một tư tưởng đẹp. Trong tổ hợp, đại
số và số học, phản chứng cho phép ta tạo thêm được điều kiện và chọn một đường
hướng chứng minh đơn giản hơn so với việc đi theo chiều xuôi từ giả thiết đến kết
luận của một bài toán. Trong hình học, ngoài những bài toán thể hiện rõ phương
pháp của phản chứng (chẳng hạn như bất đẳng thức hình học), thì tư tưởng và hình
ảnh của phép suy luận phản chứng vẫn xuất hiện trong nhiều bài toán với hình thức
khá đa đạng và thú vị. Đặc thù của phương pháp này cho phép ta "ứng biến" linh
hoạt giữa giả thiết và kết luận đồng thời có được cái nhìn rộng hơn về những mối
quan hệ đó. Do đó, trong nhiều bài toán hình học, phản chứng có thể không phải là
lời giải cuối cùng nhưng là đường lối hỗ trợ trong việc tìm ra lời giải.
Một trong những hình ảnh thường gặp nhất của suy luận phản chứng trong hình
học là việc sử dụng "điểm trùng nhau". Tình huống thường gặp nhất, ta cần chứng
minh tính chất hay sự tồn tại của một số đối tượng hình học, chẳng hạn như giao
điểm của một số đường thẳng. Khi đó, gọi hai hay một số giao điểm (dĩ nhiên tồn
tại) của một số cặp hay một số đối tượng. Sau đó, ta sẽ chứng minh các giao điểm
(đối tượng) mà ta vừa dựng là trùng nhau. Đôi khi để thực hiện điều này, ta cũng cần
gọi thêm một số đối tượng khác cùng đi qua điểm đang xét rồi xét sự đồng quy của
chúng với các đối tượng gọi thêm nhằm có thêm tính chất của các điểm mà ta cần
chứng minh trùng nhau.
Suy luận này còn được gặp ở dạng sau: giả thiết cho biết điểm A là một điểm
có các tính chất (1), (2),... mà từ những điều kiện này ta suy ra duy nhất cách dựng
điểm A. Ngoài ra, từ những tính chất trên, dựa vào kết luận cần chứng minh suy luận
ngược về hay qua trực quan hình vẽ, ta đoán được A còn có các tính chất (10 ).(20 ),...
mà từ đó có thể kết nối được giữa (1), (2),... và kết luận. Khi đó, ta bỏ đi một số tính
chất theo cách xác định gốc của điểm A và thay vào đó là một số tính chất ta đoán

21
được, sao cho từ hệ các tính chất trên xây dựng được duy nhất một điểm A0 . Bằng các
lý luận hình học ta sẽ chứng minh A và A0 trùng nhau để có cùng lúc hệ điều kiện
(1), (2),... và (10 ), (20 ),...
Suy luận phản chứng ở đây thực chất biểu hiện ở việc ta xác định một điểm A0
theo cách khác A. Giả sử A0 không trùng A, bằng suy luận hình học thông thường sẽ
dẫn đến điều vô lý. Phần phản chứng này đôi khi rất quen thuộc hoặc trực quan hiển
nhiên nên ta bỏ qua.
Trong cách làm này, một tiêu chí rất quan trọng là cách dựng hình và dựng điểm:
những điều kiện mà ta chọn để xác định điểm A0 phải bảo đảm tính "không thừa",
tức là từ đó ta luôn dựng được ít nhất và tốt nhất là đúng một điểm thoả mãn. Đây là
một chi tiết cần được thực hiện cẩn thận vì quá trình dựng điểm mới và phản chứng
dễ xảy ra ngộ nhận. Cách dựng hình cũng là một phần quan trọng giúp ta thực hiện
dễ dàng và thành công những suy luận hình học, đặc biệt là những suy luận trùng
nhau giữa các điểm.
Những tính chất mà ta chọn thêm có thể là những điều kiện ta cần trong kết luận,
những điều ta suy ra được bằng phép tương đương từ kết luận, hay là những chi tiết
không gắn một cách tương đương với kết luận nhưng lại luôn có và từ đó có thể suy
ra được kết luận.
Việc đảo liên tục và linh động giữa giả thiết và kết luận của bài toán hay những
mệnh đề con cho phép ta thay đổi cách xác định điểm để tìm cách liên kết rõ và dễ
nhất giữa giả thiết và kết luận. Việc hoán vị các bộ điều kiện ta có, ta cần và ta suy
đoán cho ta sự lựa chọn đa dạng về hướng đi và cách làm để có thể chọn bộ điều kiện
phù hợp nhất mà từ đó theo sở trường có thể suy ra tất cả những điều kiện còn lại,
biến đổi chi tiết khó liên hệ thành dễ liên hệ thông qua một trung gian. Cách xác định
một điểm rõ ràng và cụ thể hơn chỉ ra hướng đi, hoán đổi nhiệm vụ cần chứng minh,
ví dụ giữa đồng quy, thẳng hàng, đồng viên,... Đặc biệt trong nhiều bài toán sử dụng
phương pháp đại số hoá (tính toán), việc hoán đổi điều kiện là rất quan trọng nhằm
tìm một cách xác định và dựng điểm thuận lợi cho tính toán. Chú ý đây là phương
pháp mạnh với những công cụ tỉ lệ, tỉ số kép, các định lý về tỉ lệ như Menelaus, Ceva
và phương pháp diện tích, phương pháp vector,...
Việc có thể lý luận được các điểm trùng nhau thường được dựa trên những lý
luận cơ bản nhất sau đây:

Định lý 1. Về giao điểm của các đối tượng hình học:

1. Hai đường thẳng có nhiều nhất 1 giao điểm.


2. Hai đường tròn có nhiều nhất 2 giao điểm.
3. Một đường thẳng và một đường tròn có nhiều nhất 2 giao điểm.
4. Một tia có gốc nằm trong đường tròn và đường tròn đó có nhiều nhất 1 giao điểm.

Định lý 2. Về tính duy nhất của một số đối tượng hình học:

1. Tồn tại duy nhất 1 điểm chia trong hay chia ngoài đoạn thẳng theo tỉ số k cho trước.

22
2. Tồn tại duy nhất 1 điểm thuộc đường thẳng AB mà MA2 − MB2 = k cho trước.
3. Tồn tại duy nhất 1 điểm trên đường tròn chia 1 cung cho trước theo tỉ số k cho trước.
4. Tồn tại duy nhất 1 điểm nằm trên đường thẳng đi qua 3 điểm A, B, C phân biệt sao cho
tỉ số kép của hàng định ra bởi 4 điểm đó là k 6= 1 cho trước.

Trong tình huống này, có thể mở rộng ra nhiều tính chất phức tạp hơn bằng một
số công cụ đại số, chẳng hạn như tồn tại nhiều nhất 1 điểm H nằm trên đường thẳng
đi qua 4 điểm A, B, C, D phân biệt sao cho H A.HB = HC.HD. Một phương pháp
hiệu quả để xác định sự duy nhất này là lắp các điểm vào một trục toạ độ và quy về
số nghiệm của một phương trình đại số đơn giản.
Việc chứng minh các định lý trên là khá đơn giản, xin dành cho bạn đọc. Ta sẽ đi
sâu hơn vào một số ví dụ cho phương pháp trên.

2. Một số ví dụ

2.1. Các bài toán liên quan đến sự đồng quy, thẳng hàng, đồng viên hay việc xác định
giao điểm một số đối tượng
Ví dụ 1. Cho tam giác ABC có các đường cao AA1 , BB1 , CC1 và trực tâm H. Chứng minh
rằng đường thẳng Euler của các tam giác AB1 C1 , BC1 A1 , CA1 B1 đồng quy.

Phân tích. Một hướng đi thú vị và thường gặp khi xét đến sự đồng quy của 3 hay nhiều
đường thẳng là việc xét giao điểm của từng cặp đường thẳng trong đó, chẳng hạn ở đây, ta lấy
một đường thẳng làm gốc để xét các tỉ lệ trên đó nhằm chứng minh tính đồng quy, các đối
tượng xác định khá quen thuộc và thuận lợi cho tính toán nên hướng làm trên khả thi.
Ở bài toán này, ý tưởng trên gần như tương tự tuy nhiên hướng tiếp cận lại khá thú vị: ta xét
thêm tính chất của giao điểm các cặp đường thẳng nhằm chứng minh các điểm trùng nhau,
chẳng hạn xét thêm một đường thẳng hay một đường tròn chứa giao điểm ấy.

Lời giải. Gọi A2 , B2 , C2 là trung điểm H A, HB, HC và Ha , Hb , Hc là trực tâm các tam
giác AB1 C1 , BC1 A1 , CA1 B1 thì A2 , B2 , C2 ∈ ( A1 B1 C1 ) và A2 , B2 , C2 lần lượt là tâm
đường tròn ngoại tiếp các tam giác AB1 C1 , BC1 A1 , CA1 B1 .

23
A

A2

Ha

B1

C1
H
Hc

Hb
B2 C2

B A1 C

Mặt khác lại chú ý rằng các tam giác AB1 C1 , BC1 A1 , CA1 B1 , ABC đồng dạng cùng
hướng, ta có ( Ha A2 , Hb B2 ) ≡ ( AB1 , A1 B) ≡ (CA, CB) ≡ (C2 A2 , C2 B2 ) (mod π)
nên Ha A2 , Hb B2 cắt nhau tại Tc ∈ ( A1 B1 C1 ). Tương tự thì Ha A2 , Hc C2 cắt nhau tại
Tb ∈ ( A1 B1 C1 ), hơn nữa Ha A2 và ( A1 B1 C1 ) chỉ có duy nhất 1 giao điểm khác A2 nên
Tc ≡ Tb hay Ha A2 , Hb B2 , Hc C2 đồng quy. Vậy đường thẳng Euler của các tam giác
AB1 C1 , BC1 A1 , CA1 B1 đồng quy. Bài toán kết thúc.

Ví dụ 2 (USA TST 2013). Cho tam giác ABC nhọn có trực tâm H. HB, H A giao CA, CB
lần lượt tại E, F. Gọi N, L là giao điểm của BE và đường tròn đường kính CA sao cho L nằm
giữa B và N; M, K là giao điểm của AF và đường tròn đường kính CB sao cho K nằm giữa
A và M. Chứng minh rằng AB, NK, ML đồng quy.

Lời giải. Gọi T là giao điểm của ML và AB. Theo định lý Menelaus thì:

TA LB MH
. . =1
TB LH MA
TA LH MA
Dẫn đến = . . Ta có FH.FA = FB.FC = FM2 nên dễ dàng có được:
TB LB MH

MH = FM − FH = FH.FA − FH

MA = FA − FM = FA − FH.FA

24
C K

N
E H

L
M

T A B

Từ đó dẫn đến √ r
MA FA − FH.FA FA
=√ = .
MH FH.FA − FH FH
Ta cũng đồng thời có EA.EC = EH.EB = EL2 nên:

LH = EL − EH = EH.EB − EH

LB = EB − EL = EB − EH.EB
√ r
LH EH.EB − EH EH
Do đó = √ = , nghĩa là
LB EB − EH.EB EB
r
TA LH MA HE.FA
= . = .
TB LB MH HF.EB

Chú ý rằng A, B có vai trò như nhau đối với biểur


thức ở vế phải. Tương tự, nếu gọi
0
TA HE.FA TA
T 0 là giao điểm của KN, AB ta cũng có 0 = = . Vậy T ≡ T 0 hay
TB HF.EB TB
ML, NK, AB đồng quy.

Ví dụ 3. (Vietnam TST 2013) Cho đường tròn (O, R) và điểm A cố định nằm trên (O).
B, C là các điểm thay đổi trên (O) sao cho ∠ BAC = α không đổi. Trên các tia BA, CA lần
lượt lấy E, F sao cho BE = BC = CF. Gọi P, Q lần lượt là trung điểm cung AB không chứa
C và cung AC không chứa B của (O). Đường thẳng qua O vuông góc với EF cắt AB, AC tại
M, N. Chứng minh rằng giao điểm của PN và QM thuộc đường tròn cố định.

Phân tích. Các bài toán liên quan đến định lý Pascal và sự đồng quy trên đường tròn sử
dụng trong nhiều suy luận khi ta có kết luận và cần chứng minh giả thiết. Ta đã biết định lý
Pascal với 6 điểm trên đường tròn tạo ra các giao điểm thẳng hàng. Nhưng nếu ta có các giao
điểm thẳng hàng thì sẽ có thể dẫn đến sự đồng viên của các điểm.

25
Cụ thể, nếu ta có AC cắt BF tại M, CE cắt FD tại N, AD cắt BE tại P, 3 điểm M, N, P
thẳng hàng và A, B, C, D, F đồng viên thì E ∈ ( ABCDF ). Gọi E0 là giao điểm của BP và
( ABCDF ), CE0 cắt FD tại N 0 thì M, N 0 , P thẳng hàng dẫn đến N 0 ≡ N, E0 ≡ E.

E
P

Q M

X
O

B C

Lời giải: Gọi I là tâm nội tiếp tam giác ABC. Ta sẽ chứng minh rằng OI ⊥ EF hay là
OE2 − OF2 = IE2 − IF2 . Gọi ( I ) tiếp xúc với CA, AB tại X, Y thì:

IE2 − IF2 = EY 2 − FX 2 = ( a − p + b)2 − ( a − p + c)2 = (b − c) a

Lại có EA.EB = R2 − OE2 và FA.FC = R2 − OF2 nên:

OE2 − OF2 = FA.FC − EA.EB = (b − a) a − (c − a) a = (b − c) a

Do đó IE2 − IF2 = OE2 − OF2 hay OI ⊥ EF. Từ đó M, I, N thẳng hàng. Gọi PM cắt
QN tại T. Ta có BQ cắt CP tại I, AB cắt PT tại M, AC cắt QT tại N và M, I, N thẳng
hàng nên T ∈ (O). Vậy T luôn thuộc một đường tròn cố định.

Ví dụ 4. Cho tam giác ABC nhọn nội tiếp (O) có trực tâm H và đường cao AD, BE, CF. P
bất kỳ trên đường thẳng Euler của tam giác ABC. PA, PB, PC cắt (O) tại M, N, K. Gọi A0
đối xứng với M qua D, B0 đối xứng với N qua E, C 0 đối xứng với K qua F.

26
a) Chứng minh rằng H, A0 , B0 , C 0 thuộc cùng một đường tròn.
b) Đường thẳng qua C 0 vuông góc HC 0 và đường thẳng qua B0 vuông góc HB0 lần lượt
cắt HK, HN tại V, T. Chứng minh rằng VT k NK.

N A10
A

B1

E
O30

C1 C0 O C10
F

K O10
P H

0
A0 B

B D C
M
B10

A1
X

Lời giải. Gọi A1 , B1 , C1 là giao điểm thứ hai của H A, HB, HC với (O).

• Ta chứng minh rằng đường thẳng qua M vuông góc với MA1 , gọi là m, và các
đường thẳng xác định tương tự đồng quy trên đường thẳng OH. Thật vậy, gọi
A10 , B10 , C10 là các điểm xuyên tâm đối của A1 , B1 , C1 với (O). Theo định lý Pascal,
gọi NB10 cắt KC10 tại O10 và BC10 cắt CB10 tại X thì P, X, O10 thẳng hàng. Cũng theo
định lý Pascal thì X, O, H thẳng hàng nên X, O, O10 , H, P thẳng hàng hay n, k cắt
nhau tại O10 ∈ OH. Tương tự thì m, n cắt nhau tại O20 ∈ OH nên O10 ≡ O20 . Do đó
m, n, k đồng quy trên đường thẳng Euler của tam giác ABC.
• Bây giờ, ta chứng minh rằng đường thẳng qua A1 vuông góc với MA1 và các
đường thẳng xác định tương tự đồng quy trên đường thẳng OH. Thật vây, qua
phép đối xứng tâm O thì 3 đường thẳng trên biến thành m, n, k đồng quy tại
O10 ∈ OH nên 3 đường thẳng ban đầu đồng quy tại O30 đối xứng với O10 qua O.
1
• Xét phép vị tự tâm H tỉ số , ta có đường thẳng qua D, E, F lần lượt vuông góc
2
với MA1 , NB1 , PC1 , gọi là d, e, f , đồng quy tại O100 trên OH. Dễ thấy rằng đường

27
thẳng qua A0 vuông góc với H A0 và các đường thẳng xác định tương tự đồng
quy tại O1 đối xứng với O10 qua O100 nên ∠ H A0 O1 = ∠ HB0 O1 = ∠ HC 0 O1 = 90◦ .
1
Qua phép vị tự tâm H tỉ số thì O1 biến thành O2 là tâm của ( H A0 B0 C 0 ).
2
• Gọi f cắt HK tại F 0 và k cắt HN tại E0 thì F 0 , E0 là trung điểm HV, HT. Do đó để
chứng minh rằng VT//KN thì ta chỉ cần E0 F 0 //KN. Nhưng thật vậy, điều này
E0 H F0 H HO100
đúng theo định lý Thales đảo do tỉ lệ 0 = 0 = 00 0 .
EN FK O1 O1
Bài toán được chứng minh hoàn toàn.

Nhận xét. Để liên hệ được các góc trong tam giác A0 B0 C 0 là một việc khá phức tạp, cùng với
việc phát hiện tâm của đường tròn ( H A0 B0 C 0 ) nằm trên OH nên ta nghĩ đến một cách chứng
minh nội tiếp khá đặc biệt: chứng minh tâm hoặc chân đường kính của các đường tròn chứa
một số trong các điểm H, A0 , B0 , C 0 trùng nhau. Từ đó ta quy về việc sử dụng định lý Pascal
để chứng minh các đường thẳng đồng quy. Việc xét thêm đường thẳng OH trong sự đồng
quy là một ý tương tự với ví dụ 1 nhằm giới hạn số giao điểm.

Ví dụ 5 (IMO 2013). Đường tròn bàng tiếp góc A của tam giác ABC tiếp xúc với đoạn BC
tại A1 . Các điểm B1 trên CA và điểm C1 trên AB được xác định tương tự, bằng cách lần
lượt xét đường tròn bàng tiếp góc B và góc C, tương ứng. Giả sử tâm đường tròn ngoại tiếp
tam giác A1 B1 C1 nằm trên đường tròn ngoại tiếp tam giác ABC. Chứng minh rằng tam giác
ABC là tam giác vuông.

Phân tích. Thông thường, tâm đường tròn ngoại tiếp của tam giác sẽ được xác định bởi
giao điểm đường trung trực của các cạnh. Tuy nhiên ở đây, việc xác định trung trực của
A1 B1 , B1 C1 , C1 A1 là khá phức tạp. Dù vậy, khi giải quyết chiều thuận của bài toán, ta thấy
tâm O1 của ( A1 B1 C1 ) là điểm chính giữa cung chứa góc vuông. Từ đó, với trực quan, ta
có thể suy đoán tâm của ( A1 B1 C1 ), nếu thuộc ( ABC ) thì là điểm chính giữa của 1 trong 3
_ _ _ _
cung BAC, ACB, CBA, giả sử là BAC. Sau khi lấy M là điểm chính giữa cung này, ta có thể
chứng minh đơn giản rằng MB1 = MC1 , từ đó M thuộc trung trực B1 C1 . Từ điều này, ta
chỉ cần lí luận sao cho M và O1 thuộc cùng một cung chắn BC thì sẽ có ngay M ≡ O1 .

28
M O1
A

C1

B1
O
B A1 C

Lời giải. Gọi O là tâm ngoại tiếp tam giác ABC và M là trung điểm cung BC chứa
A của (O). Từ giả thiết ta có BC1 = B1 C, MB = MC và ∠ MCB1 = ∠ MBC1 nên
∆MBC1 = ∆MCB1 (c.g.c). Từ đây MB1 = MC1 hay M thuộc trung trực B1 C1 . Do đó
M là một trong hai giao điểm của trung trực B1 C1 và (O).
Theo phân tích trên, ta sẽ lí luận để M ≡ O1 . Thật vậy, do A1 , B1 , C1 nằm trong các
cạnh của tam giác ABC, còn theo giả thiết thì O1 ∈ ( ABC ) nằm ở miền ngoài tam
giác ABC nên O1 nằm ở miền ngoài tam giác A1 B1 C1 hay tam giác A1 B1 C1 tù. Có thể
giả sử rằng ∠ B1 A1 C1 > 90◦ để ta có O1 thuộc miền trong góc B1 A1 C1 . Điều này dẫn
_
đến O1 thuộc cung BAC. Hệ quả là ta có M ≡ O1 . Đến đây, ta có thể xử lí phần còn
lại theo nhiều cách khác nhau, chẳng hạn so sánh O1 A1 và O1 B1 như sau:
Đặt BC = a, CA = b, AB = c và 2p = a + b + c. Gọi A3 là trung điểm BC thì:

a2 ( b − c )2
O1 A21 = O1 A23 + A1 A23 = O1 B2 − + .
4 4

Nhận thấy rằng ∆MB1 C1 = ∆MCB (c.g.c) nên ta cũng đồng thời có được:

B1 C12 O1 B2 h i
O1 B12 = O1 B2 · = · ( p − b ) 2
+ ( p − c ) 2
− 2 ( p − b ) . ( p − c ) cos A
BC2 a2

Từ các đẳng thức trên, nhờ O là tâm ngoại tiếp tam giác ABC, ta có được:

a2 a2
O1 B2 = =
4 sin2 A
2
2(1 − cos A)

29
b + c2 − a2
Kết hợp với cos A = và O1 A1 = O1 B1 , bằng biến đổi tương đương, ta nhận
2bc
được a2 = b2 + c2 . Theo định lý Pythagore, tam giác ABC vuông tại A.

2.2. Các bài toán liên quan đến tính duy nhất của đối tượng: dựng đối tượng thoả
mãn trước, sau đó chứng minh tính duy nhất của đối tượng này
Ví dụ 6. (Sharygin 2013) Đường tròn k đi qua hai đỉnh B, C của tam giác nhọn (không cân)
ABC. k cắt các tia AB, AC kéo dài về phía B, C theo thứ tự tại P, Q. Gọi A1 là chân đường
cao kẻ từ A đến BC. Giả sử A1 P = A1 Q. Chứng minh rằng ∠ PA1 Q = 2∠ BAC.

Phân tích. Rõ ràng việc khai thác "giả sử A1 P = A1 Q" là phức tạp vì ta chưa thể xác định
rõ P, Q từ đó. Ta sẽ đi ngược lại từ kết luận để xác định P, Q thỏa mãn. Sau đó chứng minh
trường hợp của đề bài cũng phải trùng với trường hợp đó.

B C
A1
k

P1

Q1

Lời giải. Dựng ( A1 ; A1 A) cắt các tia AB, AC theo thứ tự tại P1 , Q1 . Ta có ∠ AQ1 P1 =
90◦ − ∠ A1 AB = ∠ ABC nên B, C, P1 , Q1 đồng viên, hơn nữa có A1 P1 = A1 Q1 .
Do B, C, P1 , Q1 đồng viên, ta có PQ k P1 Q1 hoặc PQ ≡ P1 Q1 . Giả sử PQ 6≡ P1 Q1 .
A1 P, A1 Q cắt P1 Q1 thứ tự tại P2 , Q2 . Do PQ k P1 Q1 , A1 P = A1 Q nên tam giác
A1 P2 Q2 cân tại A1 , dẫn đến A1 P1 = A1 Q1 . Từ đây thì ∠ P1 A1 P = ∠Q1 A1 Q. Do đó
∠ A1 Q1 Q = ∠ A1 P1 P, vô lý bởi vì tam giác ABC không cân.
Vậy PQ ≡ P1 Q1 . Do đó ∠ PA1 Q = ∠ P1 A1 Q1 = 2∠ BAC.

Ví dụ 7. (Vietnam TST 2008) Cho tam giác ABC nội tiếp đường tròn (O) có phân giác
AD, BE, CF. Gọi K, M, N lần lượt chia trong AD, BE, CF theo cùng một tỉ số k. Kí hiệu ( X )
là đường tròn qua K, A và tiếp xúc với OA. Các đường tròn (Y ), ( Z ) định nghĩa tương tự.

30
a) Nếu k = 21 , chứng minh rằng ( X ), (Y ), ( Z ) có 2 điểm chung và trọng tâm G của tam
giác ABC thuộc đường nối hai điểm này.
b) Tìm mọi k sao cho ( X ), (Y ), ( Z ) có 2 điểm chung.

A
Ob

Hb
E
K
F
Oa O

H
Hc N
M
C
A0 Ha D

Oc

Lời giải. a) Gọi K 0 là trung điểm của AK, A0 là giao điểm của tiếp tuyến tại A với BC
và Oa là tâm của đường tròn qua A, K và tiếp xúc với OA. Các điểm khác xác định
tương tự. Ta có Oa thuộc trung trực của AK vì Oa A = Oa K, và cũng có Oa ∈ A0 A.
Do ∠ A0 DA = ∠ BCA + ∠BAC 2 = ∠ A0 AD nên A0 D = A0 A. Lại có Oa A = Oa K
nên Oa K k BC. Do đó Oa là trung điểm của A0 A. Tương tự thì (Oa ), (Ob ), (Oc ) theo
thứ tự là các đường tròn đường kính AA0 , BB0 , CC 0 .
Gọi Ha , Hb , Hc theo thứ tự là chân các đường cao hạ từ các đỉnh A, B, C xuống
cạnh đối diện của tam giác ABC thì Ha , Hb , Hc theo thứ tự thuộc (Oa ), (Ob ), (Oc ). Mà
HHa .H A = HHb .HB = HHc .HC và OA2 = OB2 = OC2 nên (Oa ), (Ob ), (Oc ) có trục
đẳng phương chung là OH, nhưng OH chính là đường thẳng Euler của tam giác ABC
nên đi qua trọng tâm G. Từ đây thì Oa Ob Oc ⊥ OG.
b) Tiếp theo ý tưởng của phần trên, gọi Oa0 , Ob0 , Oc0 lần lượt là các điểm chia trong
AA0 , BB0 , CC 0 với cùng một tỉ lệ k 6= 12 . Suy ra Oa0 , Ob0 , Oc0 chia trong (hay ngoài)

31
Oa A0 , Ob B0 , Oc C 0 theo cùng một tỉ lệ k0 (k0 6= 0). Một kết quả quen thuộc là A0 , B0 , C 0
thẳng hàng. Để thuận tiện, ta sẽ ký hiệu là A0 , B0 , C 0 . Ta có:

−−→  −−→ −−→ 1


Oa0 Ob0 = k0 Oa Ob + A0 B0 ·
k0 +1

Và cũng đồng thời có được:

−−→  −−→ −−→ 1


Ob0 Oc0 = k0 Ob Oc + B0 C 0 · .
k0 +1

Vì Oa0 , Ob0 , Oc0 nên ∃µ, k0 để

−−→ −−→  −−→ −−→


O a Ob + A 0 B 0 = µ k 0 Ob Oc + B 0 C 0

−−→ −−→ −−→ −−→ O a Ob


= k0 A B
0 0
Hay là Oa Ob = µk0 Ob Oc và A0 B0 = µ B0 C 0 . Từ đây dẫn đến Ob Oc
. Tuy nhiên,
B0 C 0
nếu điều này đúng thì với k 6= 12 , ta có A, B, C thẳng hàng, vô lý.
Vậy k = 21 là giá trị duy nhất để cho (Oa ), (Ob ), (Oc ) có hai điểm chung.

Nhận xét. Ý tưởng của câu b) là việc chứng minh k = 12 là giá trị duy nhất thỏa mãn điều
kiện, để có điều này, ta thông qua giả sử một trường hợp thỏa mãn khác tồn tại và chứng minh
vô lý (hình ảnh của bổ đề ERIQ gợi nhớ đến phương pháp vector). Việc chứng minh sự tồn tại
duy nhất bằng đại số cũng là phương pháp thú vị trong hình học.

Ví dụ 8. (Iran TST 2013) Trên đường thẳng ` cho A, B, C, D phân biệt theo thứ tự đó. Các
đường tròn (O1 ), (O2 ) qua A, B và (O3 ), (O4 ) qua C, D sao cho (O1 ) tiếp xúc với (O3 ) và
(O2 ) tiếp xúc với (O4 ) (các tiếp điểm ở cùng phía so với `). Chứng minh rằng tiếp tuyến
chung ngoài của (O1 ) và (O4 ), tiếp tuyến chung ngoài của (O2 ) và (O3 ) và ` đồng quy.

32
k3

k4

k2 O3

O4
N
O2
M
k1
O1

Q0 S T
Q A B P C D

Lời giải. Gọi N là tiếp điểm của (O2 ) và (O4 ), M là giao điểm của (O1 ) và (O3 ). Gọi
P0 là giao điểm của tiếp tuyến tại N của (O2 ), (O4 ) và `.
Xét trục ` với A( a), B(b), C (c), D (d), P0 ( p0 ). Ta có P0 A.P0 B = P0 C.P0 D nên khi
quy về toạ độ, ta được ( a − p0 )(b − p0 ) = (c − p0 )(d − p0 ). Sau khi khai triển, đây là
phương trình bậc nhất theo p0 nên có nghiệm duy nhất, dẫn đến tồn tại duy nhất
P0 ∈ ` mà P0 A.P0 B = P0 C.P0 D. Tuy nhiên, nếu gọi P là giao điểm của tiếp tuyến tại
M của (O1 ), (O3 ) và ` thì ta cũng có PA.PB = PC.PD. Do đó P0 ≡ P.
Gọi S, T là trung điểm AB, CD. Đặt khoảng cách từ O1 , O2 , O3 , O4 đến ` lần lượt là
h1 , h2 , h3 , h4 . Trước hết, ta chứng minh đoạn mà tiếp tuyến chung chắn trên (O1 ), (O4 )
và (O2 ), (O3 ) là bằng nhau. Điều này tương đương với:

O1O42 − ( R4 − R1 )2 = O2O32 − ( R2 − R3 )2

Tuy nhiên dễ dàng thấy rằng:

O1O42 − (h1 − h4 )2 = O2O32 − (h2 − h3 )2 = (d(O1O2 , O3O4 ))2

Từ đó ta cần (h1 − h4 )2 − (h2 − h3 )2 = ( R1 − R4 )2 − ( R2 − R3 )2 . Khai triển, ta cần có:

R1 R4 − R2 R3 = h1 h4 − h2 h3 (1)

Dễ dàng chứng minh được rằng:


PA+ PB PC + PD
• x2 + h2i = R2i + h2 (1 ≤ i ≤ 4) với x = 2 , y= 2 và h = PM = PN.
• h1 h3 = h2 h4 = xy, R1 R3 = R2 R4 = h2 .

33
Bình phương hai vế của (1), ta cần chứng minh rằng:

( x2 + h21 − h2 )(y2 + h24 − h2 ) + ( x2 + h22 − h2 )(y2 + h23 − h2 ) − 2h4 = h21 h24 + h22 h23 − 2x2 y2

Chuyển vế và thu gọn, ta được:

4x2 y2 = 2h2 ( x2 + y2 ) + h2 (h21 + h22 + h23 + h24 ) − x2 (h24 + h23 ) − y2 (h21 + h22 ) (2)

Ta có h4 = R21 R23 = (h21 + x2 − h2 )(h23 + y2 − h2 ). Khai triển và thu gọn, ta được

h21 h2 + h23 h2 + h2 ( x2 + y2 ) − y2 h21 − x2 h23 = 2x2 y2 .

Lập các đẳng thức tương tự và cộng lại, ta có (2) đúng, dẫn đến (1) đúng. Như vậy, độ
dài đoạn tiếp tuyến chung chắn trên (O1 ), (O4 ) và (O2 ), (O3 ) bằng nhau, gọi độ dài
đó là k. Gọi Q1 là giao điểm tiếp tuyến chung của (O1 ), (O4 ) và ` thì
p p
Q1 A.Q1 B − Q1 C.Q1 D = k.

Gọi Q10 là giao điểm tiếp tuyến chung của (O2 ), (O3 ) và ` thì
q q
Q10 A.Q10 B − Q10 C.Q10 D = k.

Trên trục `, gọi toạ độ Q1 (q1 ) và Q10 (q10 ). Xét hàm số:
q q
f (q) = ( a − q)(b − q) − (c − q)(d − q) − k (q < a)

Trong đó ta chọn gốc tọa độ thỏa mãn ad = bc. Ta có


!
0 1 c + d − 2q a + b − 2q
f (q) = p −p .
2 (c − q)(d − q) ( a − q)(b − q)
√ p √ p
Đặt a0 = a − q, b0 = b − q, c0 = c − q, d0 = d − q. Ta sẽ chứng minh dấu của
f 0 (q) luôn không đổi khi q < a, tức là ( a0 c0 − b0 d0 )(b0 c0 − a0 d0 ) không đổi dấu. Chú ý
rằng do a < b < c < d nên a0 c0 < b0 d0 , hơn nữa ta có:

(b0 c0 )2 − ( a0 d0 )2 = (b − q)(c − q) − ( a − q)(d − q) = ( a + d − b − c)q

Do q < a < 0 nên (b0 c0 )2 − ( a0 d0 )2 không đổi dấu, từ đó f 0 (q) không đổi dấu. Nhưng
rõ ràng f (q) có nghiệm nên nghiệm đó phải duy nhất. Vậy Q10 ≡ Q1 , hay tiếp tuyến
chung ngoài của (O1 ) và (O4 ), của (O2 ) và (O3 ) cắt nhau trên `.

Nhận xét. Ở bài toán này, tuy việc xây dựng các điểm mới tự nhiên nhưng việc chứng minh
trùng nhau khá phức tạp. Việc sử dụng công cụ đại số mà ở đây là tọa độ trục giúp việc chứng
minh sự tồn tại duy nhất của các điểm thỏa mãn hệ thức đại số chặt chẽ hơn.

34
2.3. Các bài toán liên quan đến việc đảo cách xác định đối tượng, thay bằng những
đối tượng có cách xác định đặc biệt
Ví dụ 9. (Canada MO 2013) Gọi O là tâm đường tròn ngoại tiếp tam giác ABC nhọn.
Lấy P trên AB là điểm thỏa mãn ∠ BOP = ∠ ABC, lấy Q trên AC là điểm thỏa mãn
∠COQ = ∠ ACB. Chứng minh rằng đường thẳng đối xứng với BC qua PQ tiếp xúc với
đường tròn ngoại tiếp tam giác APQ.

Phân tích. Qua quan sát, có thể giao điểm của d (đường đối xứng với BC qua PQ) và
( APQ) đồng thời là giao điểm thứ hai của ( ABC ) và ( APQ). Tiếp điểm này có nhiều tính
chất đặc biệt dễ khai thác, ta sẽ bắt đầu từ đây. Gọi giao điểm thứ hai của ( ABC ) và ( APQ)
là K. Kẻ tiếp tuyến từ K của ( APQ). Hơn nữa, ta có thể thấy AK k BC. Tính chất này gợi
mở cho ta nhiều điều, đồng thời khai thác được d dễ dàng hơn.

K
A

O0

O Q

B K0 M C

Lời giải. Gọi O0 là tâm của ( APQ). Ta có ∠ POQ = ∠ POA + ∠ AOQ = 2∠ BCA −
∠ ABC + 2∠ ABC − ∠ BCA = 180◦ − ∠ BAC nên O ∈ ( APQ). Lại có:

∠O0 OA = 90◦ − ∠ AQO = 90◦ − (90◦ − ∠ ABC + ∠ BCA) = ∠ ABC − ∠ BCA

Gọi E là giao điểm của AO và BC và M là trung điểm BC. Ta có:

∠ MOE = 90◦ − (180◦ − 2∠ ABC + 90◦ − ∠ BAC ) = ∠ ABC − ∠ BCA

Từ đó M, O, O0 thẳng hàng. Do đó K đối xứng với A qua OO0 hay AK k BC. Gọi K 0
đối xứng với K qua PQ thì ∠KQK 0 = 2∠KQP = 2∠KCB, lại có QK = QK 0 nên Q

35
là tâm của (KK 0 C ). Từ đó ∠QK 0 C = ∠QCK 0 . Tương tự thì ∠ PK 0 B = ∠ BPK 0 . Do đó
∠ BK 0 P + ∠ PK 0 Q + ∠QK 0 C = 180◦ hay K 0 ∈ BC.
Do ∠QK 0 C = ∠ BCA = ∠KPQ = ∠K 0 PQ nên BC là tiếp tuyến tại K 0 của (K 0 PQ).
Hơn nữa d đối xứng với BC qua PQ nên d trùng với tiếp tuyến tại K của (KPQ).

Ví dụ 10. (Gặp gỡ Toán học V) Cho tam giác ABC cân tại A, D là trung điểm BC. Lấy E
nằm ngoài tam giác ABC sao cho CE ⊥ AB và BE = BD. Gọi M là trung điểm BE. F nằm
trên cung nhỏ AD của ( ABD ) sao cho MF ⊥ BE. Chứng minh rằng ED ⊥ FD.

F
T
M

L
B D C

Lời giải. Lấy F 0 sao cho DE ⊥ DF 0 và MF 0 ⊥ BE. Khi đó tứ giác MEF 0 D nội tiếp.
Gọi K là giao điểm của ME và DF 0 thì KD.KF 0 = KM.KE. Gọi T là giao điểm của
F 0 M và DE, T và L là các giao điểm của ( TDF 0 ) và ( TMB). Khi đó L ∈ BF 0 do
∠TLF 0 = ∠TLB = 90◦ . Do MEF 0 D nội tiếp nên ∆TDF 0 ∼ ∆TME. Mặt khác ta có
∆TME = ∆TMB nên ∠TLM = ∠TBM = ∠TFD. Sử dụng nhận xét về tính duy nhất
thì L ∈ DM.
Chú ý rằng ∠ F 0 TD = ∠ MTE = 90◦ − ∠ MET = 90◦ − ∠ BDE = ∠ BDK nên
( F 0 TD ) tiếp xúc BC và tâm O0 của ( F 0 TD ) thuộc AD. Dẫn đến ∠ BDL = ∠ BF 0 D. Do
đó ∠ BF 0 D = ∠ BDM = ∠ BAD nên F 0 ∈ ( BAD ). Áp dụng tính chất về tính duy nhất
_
của giao điểm giữa đường thẳng qua M và vuông góc với BE và cung ADB, ta phải
có F 0 ≡ F. Vậy FD ⊥ DE.

36
Nhận xét. 3 mối liên hệ chính của bài toán là

MF ⊥ ME, (3)
AF ⊥ FB, (4)
DE ⊥ DF. (5)

Do đó, ở trường hợp gốc, ta có (3), (4) và cần chứng minh (5). Tuy nhiên việc kết hợp (5), (3)
lại mang đến nhiều tính chất thuận lợi hơn và vì vậy, ta có thể chuyển sang sử dụng bộ điều
kiện (5), (3) để chứng minh (4) như một hướng đi tự nhiên thông qua những suy luận về số
hữu hạn các giao điểm giữa đường thẳng và đường thẳng, đường thẳng và đường tròn.

Ví dụ 11. (Gặp gỡ Toán học V) Cho tứ giác ABCD nội tiếp đường tròn (O). Giả sử AB cắt
CD tại E, AD cắt BC tại F. Gọi M, N lần lượt là trung điểm của AC, BD. Gọi d là trung
trực của MN. Giả sử P, Q là các điểm thuộc d sao cho PE ⊥ FM và FQ ⊥ EM. Gọi K là
tâm đường tròn ngoại tiếp tam giác NEF. Chứng minh rằng tam giác KPQ cân.

B K

Q
M

T P

D C E

Lời giải. Gọi Q0 là giao điểm của đường thẳng qua M vuông góc với MF và FQ. Xác
định P0 tương tự. Gọi T là giao điểm của FQ0 và EP0 . Do tứ giác ABCD nội tiếp nên
dễ dàng có được ∆FBN ∼ ∆FAM và ∆EMA ∼ ∆END, dẫn đến:

∠ FNE = ∠ FNB + ∠ BNE = ∠EMA + ∠EMC = 180◦ − ∠ FME

37
Ta có ∠ FTE = 180◦ − ∠ FME = ∠ FNE nên T ∈ ( ENF ). Để ý rằng ∆Q0 MF ∼ ∆P0 ME
(g.g) và Q0 MP0 T là hình bình hành. Mặt khác do

MF FC EA ME
= = =
NF FD ED NE

Q0 F MF NF
Từ đó dẫn đến = = hay ∆NQ0 F ∼ ∆NP0 E (c.g.c). Do đó
PE
0 ME NE

NQ0 FQ0 MQ0 TP0


= = = . (6)
NP0 EP0 MP0 TQ0

Mặt khác 4 Q0 NP0 ∼ 4 ENF (c.g.c) nên ∠Q0 NP0 = ∠ENF = ∠ETF = ∠Q0 TP0 . Do
đó N ∈ ( Q0 TP0 ). Kết hợp với (6) thì TNQ0 P0 là hình thang cân, dẫn đến

P0 N = QT = MP0

Từ đó P0 thuộc trung trực của MN. Áp dụng tính chất về tính duy nhất, ta có P0 ≡ P.
Tương tự thì Q ≡ Q0 . Do đó:

PE PM QT
= =
QF QM PT

Vậy PE.PT = QT.QF hay KP2 = KQ2 và ta thu được KP = KQ.

Nhận xét. Xuất phát từ tính chất quen thuộc ∠EMF + ∠ENF = 180◦ , EP ⊥ FM, và
FQ ⊥ EM ta liên tưởng đến việc xác định T là giao điểm FQ, EP. Từ đó, qua trực quan, có thể
thấy MQTP là hình bình hành hay MQ ⊥ MF, MP ⊥ ME. Với các điều kiện MP ⊥ ME,
P thuộc trung trực MN, EP ⊥ FM, sau khi đảo lại và sử dụng bộ tính chất vuông góc, ta có
thể khai thác sự kiện đồng dạng một cách hiệu quả. Rõ ràng tính chất thẳng hàng hoặc đồng
viên liên hệ với sự đồng dạng là một kết quả khá thú vị.

Ví dụ 12. (Serbia 2013) Cho tam giác ABC nhọn có O là tâm đường tròn ngoại tiếp. Đường
tròn Euler (ω ) và đường tròn ngoại tiếp tam giác OBC cắt nhau tại P, Q. Chứng minh rằng
AP, AQ là hai tia đối xứng đẳng giác trong ∠ BAC.

Phân tích. Dựa vào trực quan, có thể nhận xét AP.AQ = 12 AB.AC. Rõ ràng với tính chất
trên ta thu được nhiều cặp tam giác đồng dạng, tạo dựng liên hệ rõ ràng hơn giữa hai điểm.
Vì vậy, xuất phát từ đây có thể ta sẽ thu được nhiều kết quả hơn. Như vậy, ta sẽ chọn điểm Q0
có tính chất đẳng giác và hệ thức, chỉ cần chứng minh Q0 lần lượt thuộc (ω ) và ( BOC ).

38
A

K N

O
Q
L
P

B C
M

Lời giải. Gọi Q0 là điểm thuộc tia đẳng giác với AP trong ∠ BAC để

1
AP.AQ0 = AB.AC
2

Gọi M, N, K lần lượt là trung điểm của BC, CA, AB. Từ cách dựng Q0 thì ta có được
∆ANQ0 ∼ ∆APB, ∆AQ0 B ∼ ∆ANP, ∆AKQ0 ∼ ∆APC, ∆AQ0 C ∼ ∆AKP (c.g.c). Kết
hợp với các cặp góc tương ứng thì:

∠ BQ0 C = 360◦ − ∠ AQ0 C − ∠ AQ0 B = 360◦ − ∠ AKP − ∠ ANP

Từ đó ∠ BQ0 C = ∠ BAC + ∠KMN = 2∠ BAC = ∠ BOC, dẫn đến Q0 ∈ ( BOC ). Gọi


T, L lần lượt là các giao điểm của ( BOC ) với AB, AC thì:

∠ NQ0 K = ∠ NQ0 A + ∠ AQ0 K = ∠ PBA + ∠ ACP = ∠TCL = ∠ BAC = ∠ MNK

Do đó Q0 ∈ ( MNK ). Hơn nữa ( MNK ) và ( BOC ) có nhiều nhất hai giao điểm, trong
đó có P, nên Q0 ≡ Q. Vậy AP, AQ là hai đường đối xứng đẳng giác trong ∠ BAC.

Ví dụ 13. Cho tam giác ABC nhọn nội tiếp đường tròn (O). Tiếp tuyến tại A của (O) cắt
BC tại D. E là điểm đối xứng của A qua D. Đường thẳng qua B vuông góc với AB cắt trung
trực của BC tại K. Chứng minh rằng BE ⊥ AK.

39
A

D
B C

Lời giải. Gọi K 0 là điểm sao cho hai tam giác CAB và COK 0 đồng dạng cùng hướng.
Từ đó dễ thấy K 0 thuộc trung trực của BC và:

OK 0 BA DB DB
= = =
R AC DA DE

Lại có DE ⊥ OA và DB ⊥ OK nên ∠EDB = ∠ AOK 0 hay ∆AOK 0 ∼ ∆EDB (c.g.c).


Do đó ∠ DEB = ∠OAK 0 = 90◦ − ∠EAK 0 nên EB ⊥ AK 0 . Hơn nữa có được biến đổi
∠K 0 BC = ∠OAC = 90◦ − ∠ ABC nên AB ⊥ BK 0 . Vậy K 0 ≡ K hay BE ⊥ AK.

Nhận xét. Đi ngược từ kết luận có thể thu được một tính chất rất đẹp đẽ của điểm K là
∆COA ∼ ∆CAB. Từ đây, ta sử dụng việc đảo các điều kiện trong hệ ∆COA ∼ ∆CAB (va
OK ⊥ BC), BK ⊥ BA (và OK ⊥ BC). Trong bài toán hình học, khác với các mảng khác, ta
nhiều lúc không thể xác định được đường đi hay thu được nhiều chi tiết từ giả thiết, hoặc từ
đó chỉ có một số dữ kiện phục vụ được cho kết luận. Nhu cầu đi từ kết luận và đối chiếu với
giả thiết để có định hướng là khá tự nhiên.

Ví dụ 14. (IMO Shortlist 2011) Cho tam giác ABC cân tại A. D là trung điểm của AC.
Đường phân giác ∠ BAC cắt đường tròn ngoại tiếp tam giác BDC tại E (E nằm trong tam
giác ABC). BD cắt đường tròn ngoại tiếp tam giác AEB tại điểm thứ hai F. Gọi I là giao
điểm của AF và BE. K là giao điểm của CI và BD. Chứng minh rằng I là tâm đường tròn
nội tiếp của tam giác KAB.

40
A

D0 E D

I
G

C
B

Lời giải. Gọi K 0 là điểm thuộc đoạn BD sao cho DK 0 .DB = DA2 , D 0 là trung điểm
của AB. L là giao điểm của BE và AC. Vì ED = ED 0 nên BE là phân giác ∠ DBA. Áp
dụng định lí Menelaus cho tam giác LBD, ta có

IL FB AD
· · =1
IB FD AL

IL AL FD DA2 0 DB2 − DA2


Từ đó = · . Lại có DK 0 = , K B = DB − DK 0 = nên:
IB AD FB DB DB

K0 B DB2 − DA2
=
K0 D DA2
CD DA
Dễ dàng có được = . Xét biến đổi:
CL DA + DL

IL K 0 B CD AL FD DB2 − DA2 DA
· 0 · = · · ·
IB K D CL AD FB DA2 DA + DL

AL AL 1 DA
Ta có = = = . Dẫn đến:
DA + DL AL + 2DL DB DA + DB
1+2·
2DA

AL AD DB2 − DA2 DB − DA
· · 2
=
AD AD + DL DA DA

41
EG BG
Gọi G là trọng tâm của tam giác ABC thì GF.GB = GE.GA. Chú ý = nên ta
EA BA
EG BG GA2
có được = . Do đó kết hợp lại thì GF = . Ta có:
AG BA + BG BA + BG

GA2
FD = GD + GF = GD +
AB + BG

GA2
Tương tự thì FB = GB − GF = GB − . Lưu ý rằng:
AB + BG
9 2 1 2 2
 9
2 1 2 2

GA = 4AB − BC , GB = 2BC + AB
4 4 4 4
Từ đó có được:

1 
FB GB2 − GA2 + GB.GA BC2 − AB2 + GB.AB
= = 3
FD 1 1 AB2 1
GB.BG + GB.AB + GA2 + GB.GA
2 2 2 2
3 1
FB GB − AB
Ta chứng minh rằng = 2 2 . Thật vậy, điều này tương đương với:
FD 1
AB
2
   
1 2 2 1 2 
AB BC − AB + AB.GB = AB + AB.GB (3GB − AB)
3 2

Viết lại thành:


   
1 2 2
2 BC − AB + AB.GB = (3GB − AB) ( GB + AB)
3

Đẳng thức cuối cùng đúng sau khi khai triển và thu gọn. Theo định lý Menelaus cho
tam giác BLD thì K 0 , I, C thẳng hàng. Theo tính chất duy nhất, ta có K 0 ≡ K. Từ đây
∠KAD = ∠ DBA = 2∠EBA = 2∠ FAE, do đó AI là phân giác ∠ BAK. Vậy I là tâm
đường tròn nội tiếp tam giác KAB. Bài toán kết thúc.

Nhận xét. Điểm K xác định bởi CI ∩ BD khá rời rạc so với các quan hệ khác. Tuy nhiên, nếu
đi ngược từ kết luận (cụ thể là biến đổi tương đương), ta lại có tính chất thú vị DK.DB = DA2
và có lẽ đây là một khởi đầu thuận lợi hơn, vì sau đó để chứng minh C, I, K thẳng hàng, ta đã
có định lý Menelaus và tính toán đại số hỗ trợ; các quan hệ tỉ lệ độ dài trong bài khá rõ ràng.

Ví dụ 15. (USATST 2013) Cho tam giác ABC vuông tại C có đường cao CH. Lấy X thuộc
CH. L, K lần lượt thuộc AX, BX sao cho AK = AC và BL = BC. Chứng minh rằng khi X
thay đổi trên CH thì ( HKL) luôn đi qua một điểm cố định khác H.

42
P

Q0
C

X
C1
K
L
V A H M B

C2

C0

Lời giải. Ký hiệu ( B) là ( B; BC ) và ( A) là ( A; AC ). Gọi P, Q lần lượt là giao điểm


thứ hai của AX và BX với ( B) và ( A). V là giao điểm của PQ và AB. Ta có được
BK.BQ = BC2 = BA.BH và BC tiếp xúc ( A) nên ( XB, QK ) = −1. Gọi M là giao điểm
PK, AB thì ( AB, V M) = P( XB, QK ) = −1. Tương tự, gọi M0 là giao điểm QL, AB thì
( AB, V M0 ) = −1, dẫn đến M0 ≡ M hay KP, LQ, AB đồng quy. Do CX là trục đẳng
phương của ( A), ( B) nên XR.XQ = XL.XP, từ đó QLKP là tứ giác nội tiếp.
Ta chứng minh kết quả sau: Cho ( A) và ( B) là hai đường tròn trực giao với C và
C là hai giao điểm. Gọi V 0 là tâm vị tự ngoài của ( A), ( B). Lấy Q ∈ ( A) sao cho đoạn
0

VQ không cắt ( A). Đường thẳng VQ cắt ( B) tại P0 , P sao cho P0 thuộc đoạn PQ. Khi
đó các đường thẳng AP, BQ, CC 0 đồng quy.
Thật vậy, gọi X 0 là giao điểm của AP và CC 0 . Bởi ( A), ( B) trực giao nên AC, AC 0
là tiếp tuyến của ( B), dẫn đến PX 0 là đường đối trung của tam giác PCC 0 . Từ đó:
 2
CX 0 PC
=
C0 X 0 PC 0
 2
CX 00 QC
Gọi X 00 là giao điểm của BQ và CC 0 thì tương tự, ta cũng có 0 00 = . Gọi
CX QC 0
Q0 là giao điểm của PQ và ( A). Xét phép vị tự tâm V 0 như sau:

HV 0 : ( B) 7→ ( A), P 7→ Q0 , C 7→ C1 , C 0 7→ C2

43
PC Q0 C1
Từ đây thì = . Hơn nữa xét ( A) cùng các tam giác đồng dạng thì:
PC 0 Q0 C2

QC 0 V0Q V0Q QC
= 0 = 0 = 0
Q C2
0 V C1 V C2 Q C1

Kết hợp các điều kiện trên, ta có được:


 2  2  2
CX 0 PC Q0 C1 QC CX 00
= = = =
C0 X 0 PC 0 Q0 C2 QC 0 C 0 X 00

Các điểm X 0 .X 00 cùng chia trong đoạn CC 0 theo cùng một tỉ số nên X 0 ≡ X 00 hay là
AP, BQ, CC 0 đồng quy. Quay trở lại bài toán.
Gọi T là giao điểm của V 0 Q và ( B). Áp dụng bổ đề trên thì AT, BQ, CC 0 đồng
quy, nhưng điểm đồng quy chính là X nên T ≡ P, dẫn đến V ≡ V 0 . Chú ý rằng:

VA R( A) CA
= =
VB R( B) CB

Từ đó CV là phân giác ngoài của tam giác ABC. Lại có ( AB, V M) = −1 nên CM là
phân giác trong của tam giác ABC. Ta có ∠VPC = ∠VQ0 C1 = ∠VCQ nên

VP.VQ = VC2 = V H.V M

Hơn nữa Q(VL, KA) = A(VL, KQ) = −1 nên LK đi qua V. Do QLKP nội tiếp nên
VL.VK = VP.VQ = V H.V M. Vậy ( HKL) luôn đi qua điểm M cố định.

Nhận xét. Bài toán này hầu như giống bài toán 5 tại IMO 2012 về cách xác định các đối
tượng nên ta có thể nghĩ đến việc dựng ( A), ( B) cùng các giao điểm. Từ sau đó, những suy
luận và quan sát giúp biến đổi linh hoạt trong việc sử dụng các điều kiện và điểm trùng nhau.
chẳng hạn như PK, QL, ( HLK ), AB đồng quy hay V là chân đường phân giác ngoài - tâm vị
tự ngoài (rõ ràng tính chất này cụ thể hơn việc xác định V = PQ ∩ AB). Bài toán cho thấy rõ
việc chứng minh điểm trùng nhau phối hợp giữa các tỉ lệ đại số, hàng điểm điều hòa,...

Ví dụ 16. (Bulgaria 1998 - Vietnam TST 2013) Cho tứ giác ABCD có các cặp cạnh không
song song nội tiếp đường tròn (O; R). Gọi E là giao điểm của AC, BD. Đường phân giác
trong của ∠ AEB lần lượt cắt các đường thẳng AB, BC, CD, DA lần lượt tại M, N, P, Q.

a) Chứng minh rằng các đường tròn ( AQM), ( BMN ), (CPN ), ( DPQ) cùng đi qua một
điểm. Gọi điểm đó là K.
2R2
b) Đặt min{ AC, BD } = m. Chứng minh rằng OK ≤ √ .
4R2 − m2

44
Q

N
K

M B

O
C
D P F

Lời giải. Gọi F là giao điểm của AB và CD và L là giao điểm của AD và BC.
a) Áp dụng định lí Miquel thì ( FMP), ( BMN ), (CNP), ( FBC ) có một điểm chung,
( FMP), ( QMA), ( QPD ), ( FAD ) có một điểm chung; ( BMN ), ( QMA), ( LAB) có một
điểm chung; ( NPC ), ( QPD ), ( LCD ) có một điểm chung. Gọi K 0 là giao điểm của
( FBC ) và ( FAD ) thì K nằm trên ( LAB), ( LCD ). Bởi vì ( FMP) và ( FBC ) chỉ có đúng
một giao điểm ngoài F, nếu ta chứng minh được K 0 ∈ ( FMP) thì ta cũng phải có
( BMN ), (CNP), ( QMA), ( QPD ) có 1 điểm chung. Ta sẽ chứng minh điều này.
Thật vậy, ∆K 0 BC, ∆K 0 AD đồng dạng cùng hướng nên K 0 là tâm của phép vị tự
quay biến A 7→ D, B 7→ C, AB 7→ DC. Lại có tỉ lệ sau do tứ giác ABCD nội tiếp và
tính chất phân giác:
MA EA ED PD
= = =
MB EB EC PC
Nghĩa là qua cùng phép vị tự quay trên , ta có M 7→ P. Do đó:

( MK 0 , MP) ≡ ( BK 0 , BC ) ≡ ( FK 0 , FC ) ≡ ( FK 0 , FP) (mod π )

Từ đây thì K 0 ∈ ( FMP). Vậy theo lập luận, ta phải có K 0 là điểm chung của các đường
tròn ( BMN ), (CNP), ( QMA), ( QPD ), dẫn đến K 0 ≡ K.
b) Gọi T là điểm thuộc tia OE sao cho OE.OT = R2 . Do đó OE.OT = OA2 và
∠OAC = ∠OTA. Tương tự thì ∠OCA = ∠OTC và ∠OBD = ∠OTB. Từ đó thì:

∠ BTC = ∠OCA − ∠OBD = ∠ BCD − ∠CDA = ∠ AFD

45
Điều này dẫn đến T ∈ ( BCF ). Tương tự thì T ∈ ( ADF ). Áp dụng tính chất duy nhất
về giao điểm, ta phải có T ≡ K. Do đó OE.OK = R2 . Vậy ta có:

R2 R2 2R2
OK = ≤r =√
OE m2 4R2 − m2
R2 −
4
Bài toán được chứng minh hoàn toàn..

Nhận xét. Bài toán trên không quá phức tạp nhưng tích hợp được nhiều tính chất thú vị.
Hình ảnh các đường tròn gợi ý đến định lý Miquel, hệ quả là việc xét các giao điểm của các
đường tròn ( FBC ), ( FAD ), ( FMN ). Việc dùng điểm trùng nhau thường hiệu quả trong các
bài toán về sự đồng quy 3 đường tròn hoặc đường thẳng, thông qua việc xét sự đồng quy của 3
đường hình học khác. Việc xác định K thông qua giao điểm của các đường tròn là khá phức tạp
và khó liên hệ với định tính. Ý sau tương tự bài toán Bulgaria 1998, khi có OE.OK = OA2
thì ta có thể thu được các tam giác đồng dạng và nhiều tính chất khác.

3. Một số bài tập tham khảo


Bài toán 1. Cho tam giác ABC nội tiếp đường tròn (O). Đường phân giác trong của ∠ BAC
cắt (O) tại D. Gọi I là tâm đường tròn nội tiếp tam giác ABC. K là điểm đối xứng của I qua
BC. KD cắt (O) tại M. Chứng minh rằng tứ giác IODM nội tiếp.

Bài toán 2. (Iran 1997) Cho tam giác ABC nội tiếp đường tròn (O). M là điểm thay đổi
trên cung BC không chứa A. Gọi I, J lần lượt là tâm đường tròn nội tiếp của các tam giác
ABM và ACM. Chứng minh rằng đường tròn ngoại tiếp của tam giác MI J luôn đi qua một
điểm cố định khi M thay đổi.

Bài toán 3. (Chọn đội tuyển KHTN 2012) Cho tam giác ABC không cân nội tiếp đường
tròn (O). P là điểm bất kì trong tam giác ABC và không trùng với O. AP cắt (O) tại D khác
A. Lấy DE, AF là đường kính của (O). EP, FP lần lượt cắt (O) tại G, H khác E, F. Giả sử
AH cắt DG tại K. Gọi L là hình chiếu của K lên đường thẳng OP.
a) Chứng minh rằng 4 điểm A, L, K, D cùng thuộc một đường tròn (S).
b) Chứng minh rằng đường thẳng OP cắt EF tại điểm T thuộc (S).

Bài toán 4. (Chọn đội tuyển TP.HCM 2013) Cho tam giác ABC cân tại A nội tiếp đường
tròn (O). Điểm M thay đổi trên (O) và M không thuộc AO. Đường thẳng vuông góc với
AM tại M cắt BC tại N. Đường trung trực của MN cắt AB, AC lần lượt tại E, F. Tìm quỹ
tích trọng tâm của tam giác AEF khi M thay đổi.

Bài toán 5. (USAMO 2008) Cho tam giác ABC không cân có M, N, P lần lượt là trung
điểm của BC, CA, AB. Đường trung trực của AB và AC cắt AM lần lượt tại D và E. BD
cắt CE tại F. Chứng minh rằng tứ giác APFN nội tiếp.

46
Bài toán 6. (Chọn đội tuyển Hải Phòng 2013) Cho hai đường tròn (O1 ) và (O2 ) cắt nhau
tại hai điểm phân biệt A và B sao cho O1 , O2 nằm khác phía với đường thẳng AB. Một đường
thẳng thay đổi qua A cắt (O1 ), (O2 ) lần lượt tại C, D khác A (A nằm giữa C và D). Gọi P, Q
lần lượt là hình chiếu vuông góc của B xuống tiếp tuyến tại C của (O1 ) và tiếp tuyến tại D
của (O2 ). Chứng minh rằng đường thẳng PQ luôn tiếp xúc với một đường tròn cố định.

47
CÁC BÀI TOÁN VỀ HỆ SỐ NHỊ THỨC
Nguyễn Nguyễn
Lớp chuyên Toán khoá 2016 - 2019

GIỚI THIỆU. Bài viết khai thác các tính chất của hệ số nhị thức và ứng dụng qua
các bài toán Olympic.

1. MỘT SỐ ĐỊNH LÝ LIÊN QUAN


Định nghĩa 1. Với k, n ∈ N và n > 0, một tổ hợp chập k của n phần tử là số cách chọn  rak
n
phần tử mà không kể đến thứ tự giữa chúng từ n phần tử cho trước. Ký hiệu là Cnk hay .
k
     
n n n! n
Công thức xác định là = . Quy ước rằng với k > n thì = 0.
k  k k!(n − k)! k
n
Để thuận tiện, ký hiệu sẽ được sử dụng trong toàn bộ phạm vi bài viết.
k
Một số định lý liên quan thường được sử dụng:
n  
n n k n−k
Định lý 1 (Khai triển Newton). Với x, y ∈ R và n ∈ N : ( x + y) = ∑
∗ x y .
i =1
k

Định lý 2 (Công thức Pascal). Với n, k ∈ N và n > 0:


         
n+1 n n n n n−1
= + và = .
k+1 k k+1 k k k−1
 
k   
m n m+n
Định lý 3 (Hệ thức Vandermonde). Với k, m, n ∈ N: ∑ = .
i =0
i k − i k

Chứng minh. Xác định hệ số của x k trong khai triển ( x + 1)m+n bằng 2 cách.
 
∗ m
Định lý 4. Với m, n ∈ N và m ≥ n thì m là ước của (m, n).
n
 
p
Hệ quả 1. Với p là số nguyên tố, ta luôn có p là ước của với k ∈ {1, 2, ..., p − 1}.
k

49
 
p−1
Định lý 5. Với p là số nguyên tố, ta luôn có được ≡ (−1)k (mod p) với k ∈
k
{0, 1, ..., p − 1}.

Định lý 6 (De Polignac, Legendre). Cho n ∈ N∗ và p là số nguyên tố, ta có:


∞  
n n − s p (n)
v p (n!) = ∑ pi
=
p−1
.
i =1

Trong đó v p (n) là số mũ lớn nhất của p trong phân tích n ra thừa số nguyên tố, bnc là số
nguyên lớn nhất không vượt quá n và s p (n) là tổng các chữ số của n trong hệ cơ số p.
   
n n
Chứng minh. Trước hết ta có nhận xét rằng có đúng − i+1 số nguyên
pi p 
n
dương t ≤ n mà v p (t) = i với i = 1, 2, .... Hơn nữa khi i đủ lớn thì = 0 nên từ
pi
đó ta có:
∞     ∞  
n n n
v p (n!) = ∑ i
− i +1 =∑ i .
i =1 p p i =1 p
s   s
k n
Đặt n = ∑ nk p với 0 ≤ nk ≤ p − 1. Khi đó với j = 1, 2, ..., ta có j
= ∑ ni pi − j .
k =0 p i= j
Do đó:
!
∞   s s s i −1 s
n i i− j j pi − 1 n − s p (n)
∑ pj = ∑∑ n p = ∑ i ∑n p = ∑ i p−1 = p−1 .
n .
j =1 j =1 i = j i =1 j =0 i =1

∞  
n n − s p (n)
Vậy ta đã chứng minh được v p (n!) = ∑ pi
=
p−1
.
i =1


 lý 7 (Kummer). Cho k, n ∈ N với 1 ≤ k ≤ n và p là số nguyên tố. Khi đó ta có
Định
n
vp đúng bằng số lần nhớ của phép cộng k và n − k trong hệ cơ số p.
k

Chứng minh. Phép chứng minh khá dài nên không trình bày lại tại đây, có thể tham
khảo trong bài viết của tác giả Trần Quốc Anh tại [1].
 
n
Hệ quả 2. Cho k, n ∈ N∗ với 1 ≤ k ≤ n và p là số nguyên tố. Khi đó ta có v p đúng
k
bằng số lần nhớ của phép trừ n cho k trong hệ cơ số p.

Định lý 8 (Lucas). Cho p là số nguyên tố và m, n ∈ N ∗ , m ≤ n. Lần lượt gọi m =


m1 m2 ...mk ( p) và n = n1 n2 ...nk ( p) là biểu diễn của m và n trong hệ cơ số p. Khi đó ta có:
      
n n1 n2 n
≡ ... k (mod p).
m m1 m2 mk

50
 i
p
Chứng minh. Ta dễ dàng có p là ước của với i ∈ N∗ và 0 ≤ k ≤ pi − 1. Từ nhận
k
i i
xét trên thì (1 + x ) p ≡ 1 + x p (mod p). Do đó:

k
n
i i
( 1 + x ) n = ( 1 + x ) ∑ i =0 p n i ≡ ∏ ( 1 + x p ) n i (mod p)
i =0
 
n
Ta xác định hệ số của xm
ở 2 vế của đồng dư thức trên. Hệ số của xm ở vế trái là .
m
k  
k
p im ni
m ∑
Hệ số của x ở vế phải là của x i=0 i và là ∏ .
i = 0
mi
      
n n1 n2 n
Vậy ta có được ≡ ... k (mod p).
m m1 m2 mk

Định lý 9 (Wolstenholme). Cho p ≥ 5 là số nguyên tố. Khi đó ta có:


   
2p 3 2p − 1
≡ 2 (mod p ) hay ≡ 1 (mod p3 ).
p p−1

Chứng minh. Theo đẳng thức Vandermonde, ta có:


   
p  p −1   2
2p p p p
=∑ = 2+ ∑
p i =0
i p−i i =1
i

Hơn nữa lại có được

p −1  2 p −1   p −1
p p2 p − 1 2 2 1
∑ i
= ∑ i 2 i−1
= p ∑ 2.
i
i =1 i =1 i =1

Do đó ta chỉ cần chứng minh rằng:

p −1
1
∑ i2
≡ 0 (mod p).
i =1
 
1 
Nhưng 2 với i ∈ {1, 2, ..., p − 1} chính là 12 , 22 ..., ( p − 1)2 với p ≥ 5 nên:
i

p −1 p −1
1 p( p − 1)(2p − 1)
∑ i2
= ∑ p2 = 6
≡ 0 (mod p).
i =1 i =1
   
2p 3 2p − 1
Vậy ≡ 2 (mod p ). Biến đổi dễ có ≡ 1 (mod p3 ).
p p−1

51
Định lý 10 (Babbage). Cho p là số nguyên tố và a, b ∈ N∗ . Khi đó
       
ap a ap 2 a
≡ (mod p ). ≡ (mod p2 ).
bp b bp b

Chứng minh. Có thể tham khảo ở nhiều tài liệu, không chứng minh lại tại đây.

1.1. Chứng minh các đẳng thức tổ hợp


Đối với các bài toán chứng minh các đẳng thức tổ hợp, ngoài việc khai triển trực
tiếp thì ta còn một số cách tiếp cận khác như tính bằng 2 cách, xây dựng song ánh
giữa các đối tượng tổ hợp liên quan đến đẳng thức cần chứng minh, so sánh số mũ
của 2 vế hay thậm chí là so sánh từng hạng tử xuất hiện trong 2 vế của đẳng thức...
2n  2  
i 2n n 2n
Bài toán 1. Chứng minh rằng ∑ (−1) i
= (−1)
n
.
i =0

Lời giải. Ta xác định hệ số của x2n trong khai triển (1 + x )2n (1 − x )2n bằng 2 cách.
2n  
2n 2n 2 2n 2n
Ta có (1 + x ) (1 − x ) = (1 − x ) = ∑ (−1)i x2i nên hệ số của x2n là
i =0
i
 
2n
(−1)n . Mặt khác lại có:
n

2n   ! 2n   !
2n i 2n
(1 + x )2n (1 − x )2n = ∑ x ∑ (−1) j x j .
i =0
i j =0
j

Do đó hệ số của x2n là:


   2n    2n  2
j 2n 2n i 2n 2n i 2n
∑ (−1)
i j
= ∑ (−1) i 2n − i
= ∑ (−1) i
.
i + j=2n i =0 i =0

2n  2  
i 2n n 2n
Vậy ta kết luận được rằng ∑ (−1) i
= (−1)
n
.
i =0

n     
n −i 4n 2n + 2i 8n
Bài toán 2. Chứng minh rằng ∑4 2n + 2i i
=
2n
.
i =0
 8n
1
Lời giải. Ta xác định hệ số của x4n
trong khai triển của bằng 2 cách. x+
x
  8n     8n    
1 8n 8n 8n−i 1 i 8n 8n−2i 8n
Do x + =∑ x =∑ x nên hệ số của x4n là .
x i =0
i x i =0
i 2n

52
Mặt khác lại có:
 8n  4n 4n  4n−i
1 2 1 2 1
x+ = x + 2 +2 = ∑ x + 2 2i
x x i =0
x

4n 4n−i   
4n 4n − i i 8n−2i−4j
= ∑∑ i j
2x .
i =0 j =0

Để tính được hệ số của x4n , chọn i = 2n − 2j thì ta có được:


  4n 4n−i    n   
8n 2n−2j 4n 2n + 2j n −i 4n 2n + 2i
2n
= ∑ ∑ 2
2n − 2j j
= ∑4 2n + 2i i
.
i =2n−2j j=0 i =0

n     
n −i 4n 2n + 2i 8n
Vậy ta kết luận được rằng ∑4 2n + 2i i
=
2n
.
i =0
  n
(−1)i−1 n 1 1 1
Bài toán 3. Chứng minh rằng ∑ = 1 + + + ... + .
i =1
i i 2 3 n
  n
(−1)i−1 n
Lời giải. Đặt un = ∑ . Rõ ràng u1 = 1. Ta có:
i =1
i i

n +1   n  
(−1)i−1 n + 1 (−1)n (−1)i−1 n + 1 n
u n +1 = ∑ = +∑
i =1
i i n+1 i =1
i n+1−i i
   
n
(−1)i−1 n 1 n +1 i −1 n + 1
=∑
n + 1 i∑
+ (−1)
i =1
i i =1
i
  !
n +1
1 n+1
= un − ∑ (−1)k −1
n + 1 i =0 k
1 1
(0 − 1) = u n +
= un − .
n+1 n+1
n  
(−1)i−1 n 1 1 1
Từ đó bằng phép quy nạp thì ∑ = 1 + + + ... + .
i =1
i i 2 3 n

1.2. Các bài toán liên quan đến tính chia hết
Trong các bài toán sau, việc sử dụng các tính chất số học của hệ số nhị thức để so
sánh số mũ đúng giữa 2 vế hoặc sử dụng các đẳng thức tổ hợp để chứng minh các bài
toán chia hết, hơn nữa sử dụng tính chia hết để chứng minh các đẳng thức tỏ ra rất
hiệu quả.
     
2n 2n 2n
Bài toán 4. Tính giá trị của A = gcd , , ..., .
1 3 2n − 1

53
n −1  
2n .
Lời giải. Ta có đẳng thức ∑ = 22n−1 nên 22n−1 .. A hay A = 2m với m ∈ N.
i =0
2i − 1
 
2n . .
Hơn nữa ta có = 2n .. 2m nên n .. 2m−1 hay m ≤ v2 (n) + 1 = p. Do đó A ≤ 2 p .
1    
2n 2n 2n − 1 .. p
Mặt khác với i ∈ {0, 1, ..., n − 1} thì = . 2 . Vậy A = 2 p .
2i + 1 2i + 1 2i
m n
Bài toán 5 (IMO Shortlist 2012). Tìm tất cả m ∈ N∗ để với n ∈ N∗ thoả mãn ≤n≤
  3 2
n
thì ta luôn có n là ước của .
m − 2n

Lời giải. Ta chứng minh rằng m = p với p là số nguyên tố là tất cả các giá trị cần tìm.
Với p = 2 thì ta có n = 1 và dễ dàng kiểm tra được trường hợp này thoả mãn.
p−1
Xét p ≥ 3. Khi đó với n ∈ N∗ mà n ≤ thì ta luôn có được
2
   
n n−1 ..
( p − 2n) =n . n.
p − 2n p − 2n − 1
 
n ..
Lại có ( p − 2n, n) = ( p, n) = 1 nên . n. Do đó m = p thoả mãn bài toán.
p − 2n
Ta chứng minh rằng với m là hợp số thì không thoả mãn bài toán. Xét các khả năng:
   
∗ n n
• Với m = 2k (k ∈ N ) thì ta chọn n = k, ta có = = 1 không là
m − 2n 0
bội của n.
• Với m là số lẻ thì m có ước nguyên tố p là số lẻ. Đặt m = p(2k + 1) (k ∈ N∗ ).
m
Chọn n = pk thoả mãn 3 ≤ n ≤ n2 , ta có:
   
1 n 1 pk ( pk − 1)( pk − 2)...( pk − ( p − 1))
= = .
n m − 2n pk p p!

Biểu thức trên không là số nguyên do tử thức không là bội của p.

Vậy tất cả các giá trị m thoả mãn bài toán là m = p với p là số nguyên tố.

Bài toán 6 (IMO Shortlist 2011). Cho p là số nguyên tố lẻ. Với mỗi a ∈ Z, ta đặt Sa =
p −1
ak m
∑ k . Gọi m, n ∈ Z thoả mãn đẳng thức S3 + S4 − 3S2 = n . Chứng minh rằng p là ước
k =1
của m.

Lời giải. Ta có
   
(−1)k−1 1 k −1 1 p−1 1 p
= (−1) ≡ = (mod p).
k k k k−1 p k

54
Từ đó:
p −1
ak
Sa = ∑ k
k =1
p −1  p −1 
(− a)k (−1)k−1 −1 p
=−∑
k ∑ k (−a)k

p
k =1 k =1
p  !
−1 p ( a − 1) p − a p − 1
= −1 − (− a) p + ∑ (− a)k ≡ (mod p).
p k =0
k p

Điều này dẫn đến

3 p − 4 p − 1 + 2 p − 3 p − 1 − 3(1 − 2 p − 1) −(2 p − 2)2


S4 + S3 − 3S2 ≡ = ≡0 (mod p),
p p

trong đó đồng dư thức cuối cùng đúng theo định lý Fermat nhỏ.
Vậy m là bội của p.
n  
3i 2n + 1
Bài toán 7 (IMO 1974). Chứng minh rằng f (n) = ∑ 2 không là bội của 5.
i =0
2i + 1

Lời giải. Ta lập hệ thức truy hồi cho f (n) bằng các biến đổi sau:
n   n 
3i 2n + 1 √ 2n + 1
2i
∑2 2i + 1
=∑ 8
2i + 1
i =0 i =0
n √  
1 2i +1 2n + 1
= √ ·2∑ 8
2 8 i =0 2i + 1
!
2n+1 √   2n+1 
√ i 2n + 1
1 i 2n + 1
= √ ∑ 8
2 8 i =0 i
− ∑ − 8
i
i =0
 
1 √ 2n+1  √ 2n+1
= √ 1+ 8 − 1− 8 .
2 8

Từ đây ta được hệ thức truy hồi


(
f (0) = 1. f (1) = 11
.
f (n + 2) = 18 f (n + 1) − 49 f (n) ∀n ∈ N

Bằng tính tuần hoàn của dãy số dư ta dễ dàng có f (n) không là bội của 5, ∀n ∈ N.

Bài toán 8. Cho số nguyên tố p và n = nk nk−1 ...n1 n0 ( p) là biểu diễn trong cơ số p của n ∈
     
n n n
N∗ . Chứng minh rằng trong các số , , ... có đúng (nk + 1)(nk−1 + 1)...(n0 + 1)
0 1 n
số không là bội của p.

55
Lời giải. Với s ∈ N và s ≤ n, đặt s = sk sk−1 ...s1 s0 ( p) . Theo định lý Lucas, ta có:
      
n nk n k −1 n0
≡ ... (mod p).
s sk s k −1 s0
 
n
Do đó không là bội của p khi và chỉ khi 0 ≤ si ≤ ni với 0 ≤ i ≤ k.
s
Từ đây ta nhận thấy rằng có ni + 1 cách chọn si . Hơn nữa các bước lựa chọn là độc lập
với nhau nên có đúng (nk + 1)(nk−1 + 1)...(n0 + 1) số không là bội của p.

1.3. Về các bài toán đồng dư theo modulo


p   
p−2 p+2
Bài toán 9. Chứng minh rằng ∑ k ≡ p + 2 (mod p3 ).
k =2
k−2 k

p     
p−2 p+2 2p − 1
Lời giải. Ta sẽ chứng minh rằng ∑ k = ( p + 2) . Ta có:
k =2
k−2 k p−1

p    p   
p−2 p+2 p−2 p+1 p+2
∑ k k−2 k
= ∑k
k−2 k−1 k
k =2 k =2
p   
p−2 p+1
= ( p + 2) ∑ .
k =2
k−2 k−1

p −2     
p−2 p+1 2p − 1
Do đó ta chỉ cần chứng minh rằng ∑ k k+1
=
p−1
. Thật vậy, ta có:
k =0

p −2    p −2     
p−2 p+1 p−2 p+1 2p − 1
∑ k k+1
= ∑ k p−k
=
p−1
.
k =0 k =0
 
2p − 1
Theo định lý Wolstenholme thì ≡ 1 (mod p3 ) nên có kết luận bài toán.
p−1

Bài toán 10 (Putnam 1991). Cho p là số nguyên tố. Chứng minh rằng:
p   
p p+i
∑ i ≡ 2p + 1 (mod p2 ).
i =0
i

  p  p−1     
p p+i p p+i 2p
Lời giải. Ta có biến đổi ∑ = ∑ +1+ .
i =0
i i i = 1
i i p
   
2p 2
Theo định lý Babbage, ta có 1 + ≡ 1+ = 3 (mod p2 ).
p 1

56
Do đó ta chỉ cần chứng minh:

p −1   
p p+i
∑ ≡ 2 p − 2 (mod p2 ).
i =1
i i

Với i ∈ {1, 2, ..., p − 1}, ta có:


 
p+i ( p + 1)( p + 2)...( p + i )
=
i i!
 
p2 .A + p.i! 1 + 21 + ... + 1i + i!
= ( A ∈ Z)
 i! 
2
p .A 1 1
= + p 1 + + ... + +1
i! 2 i
 
1 1
≡ p 1 + + ... + +1 (mod p2 ).
2 i

Do đó ta có được:

p −1    p −1     
p p+i p 1 1
∑ i i
≡ ∑
i
p 1 + + ... +
2 i
+1
i =1 i =1
p −1     p −1  
p 1 1 p
≡ ∑p 1 + + ... + +∑
i =1
i 2 i i =1
i
p −1 2  
p p−1 B
≡ ∑ + 2 p − 2 ≡ 2 p − 2 (mod p2 ) ( B ∈ Z).
i =1
i i − 1 i!
   p
p p+i
Vậy ta đã chứng minh được ∑ ≡ 2 p + 1 (mod p2 ). Bài toán kết thúc.
i =0
i i
 
2p
Bài toán 11. Cho số nguyên tố p ≥ 5 và k = . Chứng minh rằng:
3

k  
p
∑ i ≡1 (mod p2 ).
i =0

Lời giải. Do p ≥ 5 nên ta có p = 6j + 1 hoặc p = 6j − 1 ( j ∈ N).


Ta chứng minh bài toán với p = 6j + 1, trường hợp còn lại làm tương tự. Ta có:

k   k   k
p p−1 p (−1)i−1
∑ i = ∑ i−1 i ≡ p ∑ i (mod p2).
i =0 i =0 i =0

57
k
(−1)i−1
Do đó ta chỉ cần chứng minh ∑ i
≡ 0 (mod p). Với p = 6j + 1 thì k = 4j nên:
i =0

4j 4j 2j  
(−1)i−1 1 1 1 1
∑ i = ∑ i = 2 ∑ 2j + i + 4j + 1 − i
i =0 i =2j+1 i =1
2j
1 p
= ∑ .
2 i=1 (2j + i )(4j + i − 1)

Mặt khác lại có gcd( p, (2j + i )(4j + i − 1)) = 1 với i ∈ {1, 2, ..., 2j} nên:

2j
1 p

2 i=1 (2j + i )(4j + i − 1)
≡ 0 (mod p).

k    
p 2p
Vậy ta kết luận được ∑ ≡ 1 (mod p2 ) với k = .
i =0
i 3

Bài toán 12 (Vietnam 2017). Chứng minh rằng:


1008    
2017 2
1. ∑ k ≡0 mod 2017 .
k =1
k
504    
k 2017 2016 2
2. ∑ (−1) ≡ 3(2 − 1) mod 2017 .
k =1
k

Lời giải. Trong lời giải sau, ta sẽ thay 2017 bởi số nguyên tố p ≡ 1 (mod 4).
   
p p−1
1. Với 1 ≤ k ≤ p − 1, ta có k =p ≡ p.(−1)k−1 (mod p2 ). Từ đó:
k k−1
p −1   p −3
2
p 2

∑ k k ≡ p ∑ (−1)k ≡ 0 (mod p2).


k =1 k =0

p−1
2. Với k ∈ N và 1 ≤ k ≤ , ta có biến đổi sau:
2
   
k p k p p−1 p
(−1) = (−1) . ≡ 2.(−1)2k−1
k k k−1 2k
   
p−1 p p
≡2 =2 (mod p2 ).
2k − 1 2k 2k

58
Do đó ta có được:
p −1   p −1   p −1   p −1  
4
k p 4
p 2
p 2
k p
∑ (−1) k ≡ ∑ 2 2k = ∑ k + ∑ (−1) k
k =1 k =1 k =1 k =1
p −1  p−2 1  
2
p p
≡ ∑ +∑2
k =1
k k =1
2k
p −1  p −1   p −1  
2
p p k p
≡ ∑ +∑ + ∑ (−1)
k =1
k k =1
k k =1
k
≡ 2 p−1 − 1 + 2 p − 2 + 0 = 3(2 p−1 − 1) (mod p2 ).

Bài toán kết thúc hoàn toàn.

Bài toán 13 (Vietnam TST 2010). Đặt Sn là tổng các bình phương hệ số trong khai triển
của (1 + x )n . Chứng minh rằng S2n + 1 không là bội của 3.
n  2    
n 2n 4n
Lời giải. Ta có Sn = ∑ = . Do đó S2n = .
k =0
k n 2n
Đặt cách viết trong hệ cơ số 3 là 4n = am am−1 ...a1 a0 (3) và 2n = bm bm−1 ...b1 b0 (3) với
0 ≤ ai , bi ≤ 2 và i ∈ {0, 1, ...,
 m }.
m  
4n a
Theo định lý Lucas, ta có + 1 ≡ ∏ i + 1 (mod 3). Xét các khả năng:
2n i =0
bi
   
aj 1
• Nếu tồn tại j để b j = 2 thì a j = 1 nên = = 0. Do đó:
bj 2
  m  
4n a
2n
+1 ≡ ∏ bii + 1 ≡ 1 (mod 3).
i =0

• Nếu ai ∈ {0, 1} (0 ≤ i ≤ m) thì trong các ai có 2k số 1 (2k ≤ m). Khi đó:


   2k
4n m−2k 2
+1 ≡ 1 . + 1 ≡ 2 (mod 3).
2n 1

Vậy trong mọi trường hợp, ta luôn có S2n + 1 không là bội của 3. Bài toán được chứng
minh.
1.4. Một số bài toán khác
Bài toán 14 (Iran TST 2012). Tìm tất cả n ∈ N∗ sao cho với mọi 0 ≤ i, j ≤ n thì:
   
n n
+ ≡ i + j (mod 2).
i j

59
 
n
Lời giải. Bài toán tương đương với tìm n ∈ N∗ mà + 1 ≡ i (mod 2) với 0 ≤ i ≤ n.
i  
n
Đặt n = am am−1 ...a1 a0 (2) và i = bm bm−1 ...b1 b0 (2) . Chọn i chẵn thì ≡ 1 (mod 2).
i
Nếu tồn tại j ∈ N (1 ≤ j ≤ m) sao cho a j = 0, ta chọn i = 2 j . Khi đó n − i là phép trừ
có nhớ.  
n
Theo định lý Kummer thì ≡ 0 (mod 2), vô lý. Do đó phải có b j = 1 với j ∈
i
{1, 2, ..., m}.
Chọn i lẻ, ta có (ni) ≡ 0 (mod 2) hay n − i phải là phép trừ có nhớ. Từ đó b0 = 0.
Điều này dẫn đến n = 2m − 2 với m ∈ N∗ . Thử lại, các số n trên thoả mãn bài toán.
 
2012
Bài toán 15 (CGMO 2012). Có bao nhiêu số tự nhiên k ∈ {0, 1, ..., 2012} mà là
k
bội của 2012 ?
 
2012 ..
Lời giải. Ta có 2012 = 40(503) nên theo định lý Kummer, 6 . 503 khi và chỉ khi
  k
. 2012
k .. 503. Ta sẽ đếm các số k để không là bội của 4.
k
Lại có 2012 = 11111011100(2) nên ta đếm k mà phép toán 2012 − k có không quá 1 lần
nhớ. Xét các khả năng sau:

• Nếu phép trừ không nhớ: có 28 số k bằng việc ta thay bit 0 trong n bởi bit 1.
• Nếu phép trừ có nhớ: có đúng 2 vị trí ở n để thay bit 0 bằng bit 1 và bit ở ngay
phía trước sẽ phải là bit 0; các bit 1 còn lại sẽ không bị ảnh hưởng nên ta có 2.27
số k.
 
8 2012
Do đó có 2013 − 2.2 = 1501 số k để là bội của 4. Hơn nữa
      k
2012 2012 2012
, , là bội của 4 nên tổng cộng có 1501 − 3 = 1498 số thoả
503 1006 1539
mãn bài toán.

Bài toán 16 (Vietnam  TST 1996). Với mỗi n ∈ N∗ , ta đặt f (n) =


b n−2 1 c  
k n
v2  ∑ 3 . Xác định tất cả các giá trị n để ta có f (n) = 1996.
k =0
2k + 1

60
b n−2 1 c  
k n
Lời giải. Đặt an = ∑ 3
2k + 1
. Ta có biến đổi sau:
k =0

b n−2 1 c 
b n−2 1 c √  
1 k n 2k+1 n
an = ∑ 3 = √ ∑ ( 3)
k =0 3 k =0 2k + 1 2k + 1
  !
1 n √
k n
n √ k n
= √ ∑ ( 3) k − ∑ (− 3) k
2 3 k =0 k =0
√ n √ n
(1 + 3) − (1 − 3)
= √ .
2 3

Xét các khả năng sau:


√ √
(2+ 3) k √
−(2− 3)k
1. Nếu n = 2k, ta viết an = 2k .b k với bk = .
2 3
(
b1 = 1, b2 = 4
Dễ dàng có được hệ thức truy hồi sau:
bk+2 = 4bk+1 − bk ∀k ∈ N∗
Từ đây bằng phép quy nạp nhận thấy bk lẻ khi và chỉ khi k lẻ. Đặt g(k ) = v2 (bk )
thì ta có:
f (n) = f (2k ) = k + g(k ).
Đến đây lại có 2 khả năng sau xảy ra:

• Nếu k là số lẻ thì f (n) = f (2k ) = k. Để f (n) = 1996 thì k = 1996 là số


chẵn, loại.
• Nếu k là số chẵn, đặt k = 2s .l với s ≥ 1 và l lẻ thì ta dễ chứng minh được
điều sau: (
g(2m) = g(m) + 1 nếu m là số chẵn
.
g(2m) = g(m) + 2 nếu m là số lẻ
Từ đó ta có được f (n) = f (2k) = k + g(2s .l ) = k + s + 1.
Do n = 2k = 2s+1 .l và kết hợp với giả thiết, ta được 1996 = f (n) =
2s .l + s + 1.
Điều trên tương đương với 2s .l + s = 1995. Nhận xét rằng s phải lẻ và
1 ≤ s ≤ 9.
Đến đây thử trực tiếp nhận n ∈ {3984, 3988}.
(
a1 = 1, a2 = 2
2. Nếu n = 2k + 1, ta có nhận xét rằng: .
an+2 = 2an+1 + 2an ∀n ∈ N∗
Từ đó kết hợp với việc v2 ( a2k ) ≥ k thì bằng phép quy nạp, ta được f (n) =
v2 ( a2k+1 ) = k.
Đến đây do f (n) = 1996 nên ta tìm được n = 3993.

61
Vậy các giá trị n thoả mãn bài toán là n ∈ {3984, 3993, 3988}.
 
2n
Bài toán 17 (Vietnam TST 2017). Với mỗi số nguyên dương n, ta đặt xn = .
n

2017k .
1. Chứng minh rằng nếu như < n < 2017k với k ∈ N∗ thì xn .. 2017.
2
2. Tìm tất cả h ∈ N∗ để tồn tại các số nguyên dương N, T thoả mãn: ∀n > N thì ( xn ) là
dãy số tuần hoàn theo modulo h với chu kỳ T.

Lời giải. Trong lời giải sau, ta sẽ thay 2017 bằng số nguyên tố p ≥ 3 bất kỳ.

pk
1. Theo giả thiết thì pk−1 < < n < pk nên n = ak ak−1 ...a2 a1 ( p) hay n có đúng k
2
chữ số trong hệ cơ số p. Giả sử rằng với mỗi i thoả mãn 1 ≤ i ≤ k, ta luôn có
p −1
ai ≤ 2 thì:

k
i −1 p − 1 k i −1 p − 1 pk − 1 pk − 1 pk
n= ∑ ai p ≤
2 i∑
p =
2
.
p−1
=
2
< .
2
i =1 =1

p−1
Điều này trái với giả thiết nên tồn tại chỉ số i thoả mãn 1 ≤ i ≤ k mà ai > .
2
p+1
Do đó ai ≥ nên phép cộng n + n trong hệ cơ số p là có nhớ khi ta cộng 2
2
giá trị ai . Vậy theo định lý Kummer thì xn là bội của p.
2. Giả sử rằng số nguyên dương h thoả mãn bài toán có ước nguyên tố lẻ là q.
pk
Ta chọn k đủ lớn sao cho > 2T thì tồn tại T số hạng liên tiếp của xn nằm
! 2
pk k
trong , p là bội của q hay tồn tại N để xn là bội của q với mọi n > N.
2

p t −1 pt − 1
Ta chọn t > k sao cho 2 > N. Đến đây chọn n =
thì xn không phải
2
là bội của q. Nhưng điều này trái với chứng minh trên. Do đó h không có ước
nguyên tố lẻ hay h = 2r .
(2n)! n − s p (n)
Giả sử rằng r > 1. Để ý rằng xn = và v p (n!) = với s p (n) là tổng
n!.n! p−1
các chữ số của n trong hệ cơ số p, ta có được

v2 ( xn ) = 2n − s2 (2n) − 2(n − s2 (n)) = 2s2 (n) − s2 (2n)

r −1
Ta sẽ chọn n = ∑ 2ai với 1 ≤ a1 < a2 < ... < ar−1 và 2a1 > max { N, T }. Bởi vì
i =1
cách chọn, dễ dàng thấy rằng v2 ( xn ) = r − 1 nên xn ≡ 2r−1 (mod h).
Mặt khác do T < 2a1 nên tổng n + T trong hệ cơ số 2 có ít nhất thêm 1 bit 1.

62
Từ đây dễ dàng nhận thấy v2 ( xn+T ) ≥ r hay xn+T ≡ 0 (mod h).
Nhưng do n > N và dãy tuần hoàn chu kỳ T nên xn ≡ xn+T ≡ 0 (mod h). Điều
này vô lý. Do đó r = 1 hay h = 2. Khi h = 2, dễ dàng kiểm tra rằng xn ≡ 0 (mod
2) ∀n ∈ N∗ nên T = 1. Vậy h = 2 là số nguyên dương duy nhất thoả mãn.

Bài toán kết thúc.

2. BÀI TẬP
Bài tập 1. Chứng minh các đẳng thức sau:
n
1 n + 1 n + 1 2i
1. ∑ ( n ) 2n +1 ∑ i .
=
i =0 i
n  2  i =1 
n 2n − 1
2. ∑ i =n .
i =0
i n−1
n  
(−1)i n 1
3. ∑ = .
i =0
i+1 i n+1
b n2 c    2  
n n 1 2n
4. ∑ − = .
i =0
k k−1 n+1 n
(
F0 = F1 = 1
Bài tập 2. Cho dãy số Fibonacci xác định như sau: . Chứng
F = Fn+1 + Fn ∀n ∈ N
j k n +2
n  n+k 
2
minh rằng ta luôn có đẳng thức Fn+1 = ∑ .
k =0 k

Bài tập 3 (China TST 2017). Chứng minh rằng:

58    29  
2017 + k 2075 − k 4091 − 2k
∑ 58 − k k
= ∑ 58 − 2k
k =0 k =0
     
n n n
Bài tập 4. Tìm tất cả n ∈ N và n ≥ 3 thoả mãn 22000 là bội của 1 + + + .
1 2 3

Bài tập 5. Xác định các giá trị sau:


     
n n+1 n+k
1. A = gcd , , ..., .
 k  k  k

n n n
2. B = gcd , , ..., .
1 2 n−1
 
2n
Bài tập 6. Chứng minh rằng với n ∈ N∗ thì lcm(1, 2, ..., 2n) là bội của .
n

Bài tập 7. Tồn tại hay không bộ k số ( a1 , a2 , ..., ak ) mà ai ∈ {0, 1} với i ∈ {1, 2, ..., k } mà:

63
k  
i k
1. ∑ 3 .ai = 4k .
i =0
i
k  
k
2. ∑ 5i .ai = 6k −1 .
i =0
i

Bài tập 8 (Romania TST 2004). Cho m, n ∈ N∗ và m là số lẻ. Chứng minh rằng:
m  
3m .
∑ (3n − 1)k .. n.3m .
k =0
3k

Bài tập 9 (IMO Shortlist 2007). Chứng minh rằng với k ∈ N và k ≥ 2 thì ta có:
   !
2k +1 2k
v2 − = 3k.
2k 2k −1

Bài tập 10 (China TST 2011). Cho trước số nguyên dương


 n. Gọi k là tổng các chữ số trong
n 
2n .
biểu diễn nhị phân của n. Chứng minh rằng ∑ i2r .. 22n−k với r ∈ N∗ .
i =−n
n+i
 
2n .
Bài tập 11 (China 2015). Tìm tất cả k ∈ N sao cho tồn tại vô hạn n ∈ N để 6 .. n + k.
n

Bài tập 12. Cho p là số nguyên tố và 0 ≤ k ≤ n ≤ p − 1. Chứng minh rằng:


   
n n+k p − k − 1
≡ (−1) (mod p).
k p−n−1
   
np n
Bài tập 13. Cho số nguyên tố p ≥ 5 và n, k ∈ N∗. Chứng mỉnh ≡ (mod p3 ).
kp k
   
2pn 2pn−1
Bài tập 14. Cho số nguyên tố p ≥ 5, n ∈ N ∗ . Chứng minh ≡ (mod
pn p n −1
p3n ).

Bài tập 15 (Định lý Morley). Cho số nguyên tố p ≥ 5. Chứng minh rằng:


 
p −1 p−1
(−1) 2
p −1 ≡ 4 p−1 (mod p3 ).
2

  p −1
p 2
k
Bài tập 16. Cho p là số nguyên tố. Chứng minh rằng ∑ 3 ≡ 2 p − 1 (mod p2 ).
k =0
k

64
Bài tập 17 (USATST 2010). Liệu có tồn tại hay không số nguyên tố p = 6k + 1 thoả mãn
 
3k
≡ 1 (mod p).
k
  2
p3 p
Bài tập 18. Cho số nguyên tố p ≥ 5. Chứng minh rằng ≡ (mod p8 ).
p2 p
 
2016
Bài tập 19. Có bao nhiêu k ∈ {0, 1, , 2, ..., 2016} để không là bội của 2016 ?.
k

Bài tập 20 (Trường xuân 2017). Cho dãy số ( xn ) xác định bởi công thức:

 x0 = 1
.
 xn = −2017 · x0 + x1 + ... + xn−1 ∀n ∈ N, n ≤ 2017
n

Hỏi có bao nhiêu chỉ số i ∈ {0, 1, 2, ..., 2017} sao cho | xi | là bội của 3 ?.

Bài tập 21. Cho p là số nguyên tố và p ≥ 5. Chứng minh rằng tồn tại a ∈ {1, 2, ..., p − 2}
sao cho a p−1 − 1 và ( a + 1) p−1 − 1 không là bội của p2 .
n
Bài tập 22 (Saudi Arabia TST 2015). Với n ∈ N∗ , ta đặt S(n) = ∑ ri trong đó ri là số dư
  i =0
n
của khi chia cho 3. Tìm tất cả n để S(n) ≥ n.
i

Bài tập 23. Cho trước hằng số k ∈  N∗ .Chứng minh rằng tồn tại N ∈ N∗ và chỉ xác định
n
phụ thuộc vào k sao cho ∀n ≥ N thì có ít nhất k ước nguyên tố phân biệt.
k
   
∗ n n
Bài tập 24. Chứng minh rằng không tồn tại n, k ∈ N và n ≥ k + 3 để các số , ,
    k k + 1
n n
, lập thành cấp số cộng.
k+2 k+3
     2 
n 2n n
Bài tập 25. Cho tập hợp A = , , ..., với n ∈ N∗ và n là hợp số.
n n n

1. Chứng minh rằng tồn tại vô hạn n để A là hệ thặng dư đầy đủ modulo n.


2. Chứng minh rằng tồn tại vô hạn n để A không là hệ thặng dư đầy đủ modulo n.

Bài tập 26 (Bulgaria 2011). Tồn tại hay không n, k ∈ N và 1 ≤ k ≤ n − 2 thoả mãn
 2  2  4
n n n
+ = .
k k+1 k+2

65
  n
n i
Bài tập 27. Cho p là số nguyên tố và dãy vô hạn các số a0 , a1 ... thoả mãn ∑ p a =0
i =0
i i
với vô số giá trị của n. Chứng minh rằng a0 = a1 = ... = 0.

Bài tập 28 (Romania 2010). Cho dãy số thực dương ( an ) thoả mãn:
n  
n
∑ k ak an−k = a2n ∀n ∈ N.
i =0

Chứng minh rằng ( an ) là một cấp số nhân.

Bài tập 29 (IMO Shortlist 2008). Cho n ∈ N∗ . Chứng minh rằng các số dư của:
   n   n 
2n − 1 2 −1 2 −1
A= , , ..., n
1 3 2 −1

khi chia cho 2n là toàn bộ các phần tử của {1, 3, ..., 2n − 1}.
 
2n
Bài tập 30 (China TST 2012). Với n ∈ N∗ , đặt xn = . Chứng minh rằng tồn tại vô
n
hạn các tập hợp hữu hạn A, B 6= ∅ và A, B ⊂ N ∗ thoả mãn:

∏ xi
i∈ A
= 2012.
∏ xj
j∈ B

TÀI LIỆU THAM KHẢO


1. Trần Nam Dũng (chủ biên), Các phương pháp giải toán qua các kỳ thi Olympic, 2014.
2. Trần Nam Dũng (chủ biên), Các phương pháp giải toán qua các kỳ thi Olympic, 2015.
3. Đề bài và lời giải Việt Nam TST 1989-2004.
4. Community - Art of Problem Solving: https://artofproblemsolving.com/community
5. Diễn đàn toán học Việt Nam - VMF: https://diendantoanhoc.net

66
MỘT SỐ ĐỊNH HƯỚNG TIẾP CẬN LỜI GIẢI
ĐỐI VỚI CÁC BÀI TOÁN OLYMPIC
Nguyễn Tiến Hoàng
Lớp chuyên Toán 2016 - 2019

1. GIỚI THIỆU
Theo quan sát, trong vài năm gần đây, các bài toán khó của kỳ thi HSG Quốc gia
(VMO) và chọn đội tuyển tham dự Olympic Toán Quốc tế (Việt Nam TST) đều có mối
liên hệ đến các định lý hoặc dựa trên những bài toán đã có sẵn. Chẳng hạn như bài
toán số 2 của Việt Nam TST 2017:
 
2n
Bài toán 1 (Việt Nam TST 2017). Với mỗi số nguyên dương n, đặt xn = .
n

2017k
1. Chứng minh rằng nếu tồn tại k ∈ N∗ để < n < 2017k thì xn là bội của 2017.
2
2. Tìm tất cả số nguyên dương h > 1 để tồn tại các số nguyên dương N, T sao cho
∀n > N thì { xn } là dãy số tuần hoàn theo modulo h với chu kỳ T.

Đây là một bài toán có sự liên quan mật thiết đến các định lý liên quan đến tính
chất số học của hệ số nhị thức như Legendre hay Babbage. Ví dụ gần đây nhất là bài
toán số 6 của VMO 2018 với tính chất của dãy số Lucas:

Bài toán 2 (Việt Nam 2018). Cho { xn } là dãy số xác định bởi
(
x0 = 2, x1 = 1
.
x n +2 = x n +1 + x n ∀ n ∈ N

1. Với n > 1 là số nguyên dương, chứng minh rằng nếu xn là số nguyên tố thì n là số
nguyên tố hoặc n không có ước nguyên tố lẻ.
2. Tìm tất cả các cặp số nguyên dương (m, n) để xn là bội của xm .

Hai bài toán trên là các ví dụ điển hình cho điều sau: nếu học sinh thiếu kinh
nghiệm về những bài toán thuộc về chủ đề liên quan, việc tìm ra lời giải trong điều
kiện thời gian hạn chế là một điều rất khó xảy ra. Từ đó dẫn đến câu hỏi: Liệu có thể

67
giải được trong thời gian ngắn những bài toán ở mức độ trên theo một cách "ngây thơ" và tự
nhiên nhất, với việc biết rất ít (hoặc hoàn toàn không biết) các vấn đề liên quan ?
Với tinh thần của câu hỏi trên, bài viết này sẽ quan tâm chủ yếu đến những bài
toán "phi truyền thống", nghĩa là không quá liên quan đến một kết quả nhất định
nào, đặc biệt là các kết quả khó tiếp cận với số đông. Các bài toán ấy sẽ được tiếp cận
theo góc nhìn tự nhiên nhất có thể, và hơn nữa, lời giải chỉ dựa trên sự quan sát kỹ
càng vấn đề, kết hợp với những tư duy quen thuộc.

2. Những định hướng tiếp cận điển hình

2.1. Quy nạp và thác triển từ trường hợp riêng


Nguyên lý quy nạp là một chủ đề quen thuộc với các bài toán Olympic, xin không
nhắc lại các bước thực hiện cụ thể tại đây. Về bản chất, đây là một cách tiếp cận theo
hướng xây dựng: nếu mệnh đề A(n) đúng thì các mệnh đề A( f 1 (n)), A( f 2 (n)), ...
cũng sẽ đúng, với f 1 , f 2 , ... là các hàm số theo biến n, có tập xác định và tập giá trị là Z.
Sở dĩ lấy tập hợp Z vì trong một số tình huống, ta thực hiện "quy nạp lùi", chẳng hạn
như giải phương trình hàm Cauchy trên Z. Trong phép quy nạp thông thường thì
f 1 (n) = n + 1. Với phép quy nạp kiểu Cauchy thì ta chọn f 1 (n) = 2n, f 2 (n) = n − 1.
Và còn nhiều kiểu quy nạp khác nữa tuỳ tình huống cụ thể.
Quy nạp có mối liên hệ mật thiết với thứ tự của các số nguyên. Do tính "rời rạc"
của tập hợp số nguyên, ta có thể bắt đầu từ một giá trị khởi tạo nào đó để tiến hành
quy nạp mà yên tâm rằng có thể đi qua được hết các giá trị của yêu cầu bài toán. Từ
đó dẫn đến hai ý tưởng sau:

• Nên tiến hành quy nạp với các bài toán liên quan đến số nguyên mà việc giải
trực tiếp bài toán gặp nhiều khó khăn (biến đổi đại số cồng kềnh, vấn đề trừu
tượng,...)
• Trong trường hợp bài toán không có dấu hiệu nào rõ ràng, hãy sử dụng ý tưởng
học được của phép quy nạp: giải quyết trường hợp cơ sở, quan sát, sắp xếp các
đại lượng để làm gọn vấn đề, thậm chí là đoán nhận quy luật tổng quát.

Ta bắt đầu với bài toán sau:

Bài toán 3. Chứng minh rằng với n > 3, luôn tồn tại hoán vị ( a1 , a2 , ..., an ) của (1, 2, ..., n)
sao cho ai + ak 6= 2a j với mọi i, j, k mà 1 ≤ i < j < k ≤ n.

Hướng dẫn. Ta bắt đầu từ những trường hợp nhỏ, chẳng hạn như n = 4, n = 5.
Với n = 4, ta có thể có hoán vị (4, 2, 1, 3). Với n = 5 thì ta có một hoán vị là (5, 1, 3, 2, 4).
Để ý một chút, hai hoán vị này có gì lạ ? Các số lẻ "nằm ở một bên" và các số chẵn ở
bên còn lại. Một hoán vị với đặc điểm này sẽ thoả mãn được điều kiện ai + ak 6= 2a j
nếu ai , ak được lấy ở "hai bên" của hoán vị, vì tổng nhận được luôn là số lẻ trong khi
2a j là số chẵn.

68
Vậy ta chỉ còn phải kiểm tra tính chất của bài toán đối với mỗi bên của hoán vị, mà ta
sẽ gọi mỗi bên là một hoán vị con. Một nhận xét như sau:

Nhận xét. Nếu (b1 , b2 , ..., bk ) là một hoán vị k phần tử thoả mãn yêu cầu bài toán thì
(2b1 , 2b2 , ..., 2bk ) và (2b1 − 1, 2b2 − 1, ..., 2bk − 1) cũng là các hoán vị thoả mãn.

Đây là nhận xét dễ kiểm chứng. Tại sao lại nghĩ đến điều này ? Vì trong mỗi hoán vị
con đều chứa toàn bộ phần tử là số chẵn hoặc số lẻ. Cần có xu hướng giảm độ lớn của
các phần tử trong hoán vị, nên với phần tử chẵn thì ta chia đôi, còn phần tử lẻ thì biến
đổi thành chẵn rồi chia đôi. Từ đó dễ dàng dẫn ra lại được nhận xét trên.
Dễ thấy rằng ý tưởng giảm số phần tử như trên chính là một kiểu quy nạp lùi. Hơn nữa
nếu có hoán vị thoả mãn đề bài của bộ (1, 2, ..., n) thì cũng có hoán vị thoả mãn với
bộ (1, 2, ..., n − 1) như sau: ta chỉ cần loại n là xong. Gọi A(n) là mệnh đề cần chứng
minh thì ta có các bước sau:

• Chứng minh rằng A(k ) đúng, với k nào đó chọn trước.


• Tạo ra hoán vị thoả mãn của (2, 4, ..., 2k) và (1, 3, ..., 2k − 1).
• Ghép hoán vị và lần lượt bỏ đi phần tử lớn nhất trong mỗi thao tác.

Việc trình bày lại những phân tích trên thành một lời giải hoàn chỉnh xin dành cho
bạn đọc.

Bài toán 4. Cho lưới nguyên 100 × 100. Cần ít nhất bao nhiêu đường thẳng để mỗi điểm
của lưới nguyên trên đều nằm trên ít nhất 1 đường thẳng, trừ điểm ở góc dưới cùng bên trái ?

Lời giải. Ta chứng minh bằng quy nạp rằng với lưới nguyên m × n (m, n ∈ N∗ ) như
điều kiện bài toán, số đường thẳng ít nhất là m + n − 2. Thật vậy, trước hết là có cấu
hình cho m + n − 2 đường thẳng với m − 1 đường nằm ngang và n − 1 đường nằm
dọc.
Dĩ nhiên với m = 1, số đường thẳng ít nhất là n − 1 vì không thể có đường nằm
ngang. Giả sử với m, n > 1, bài toán đúng với mọi lưới nguyên có kích thước nhỏ hơn
m × n.

• Nếu có một đường nằm ngang (hoặc nằm dọc), ta loại bỏ đường đó đi để thu
được một lưới nhỏ hơn. Theo giả thiết quy nạp, số đường thẳng cần sử dụng ít
nhất cho lưới nhỏ hơn mà ta vừa xây dựng là m + n − 1 − 2 đường thẳng, nên
kết hợp với đường thẳng mà ta vừa loại thì cần ít nhất m + n − 2 đường thẳng.
• Ngược lại, xét 2m + 2n − 5 điểm ở ngoài cùng của lưới nguyên (do đã loại bỏ
điểm ở góc dưới cùng bên trái), mà ta sẽ gọi là "đường bao". Một đường thẳng
bất kỳ chỉ cắt tối đa 2 điểm ở đường bao, nên số đường thẳng tối thiểu sẽ là:
 
2m + 2n − 5
+ 1 = m + n − 2.
2

69
Theo nguyên lý quy nạp, với m, n ∈ N∗ thì số đường thẳng cần sử dụng ít nhất cho
lưới nguyên m × n là m + n − 2. Áp dụng với m = n = 100 thì ta cần ít nhất 198
đường thẳng.

Đôi khi việc thực hiện quy nạp phải thông qua nhiều biến đổi đại số như hai bài
toán sau:

Bài toán 5. Cho số nguyên dương k. Xét hàm số f : N∗ → N∗ được xác định như sau:
(
f (n) = 1 ∀n ≤ k + 1
f (n) = f ( f (n − 1)) + f (n − f (n − 1))

với mọi n > k + 1. Chứng minh rằng ∀n ∈ N∗ , { f −1 (n)} là tập hữu hạn chỉ chứa các số
nguyên dương liên tiếp.

Lời giải. Trước hết ta chứng minh rằng f (n) − f (n − 1) ∈ {0; 1} ∀n ∈ N∗ .


Giả sử rằng điều này đúng với mọi số nguyên dương nhỏ hơn n + 1 với n ≥ k. Chú ý
rằng điều này dẫn đến f (m) < m với 2 ≤ m ≤ n. Có hai khả năng sau:

• Nếu f (n) − f (n − 1) = 0 thì ta có:

f (n + 1) − f (n) = f ( f (n)) + f (n + 1 − f (n)) − f ( f (n − 1)) − f (n − f (n − 1))

Viết gọn lại thì f (n + 1) − f (n) = f (n − f (n) + 1) − f (n − f (n)) ∈ {0; 1}.


• Nếu f (n) − f (n − 1) = 1 thì biến đổi tương tự, ta sẽ có được:

f (n + 1) − f (n) = f ( f (n − 1) + 1) − f ( f (n − 1)) ∈ {0; 1}

Theo nguyên lý quy nạp thì f (n) − f (n − 1) ∈ {0; 1} ∀n ∈ N∗ . Từ đó f (n) lập thành
một dãy không giảm các số tự nhiên "gần liên tiếp". Bây giờ ta chỉ cần chứng minh f
không bị chặn trên.
Thật vậy, giả sử rằng tồn tại a, b ∈ N∗ để f (n) = b ∀n ≥ a ≥ k + 1. Thay n = a + b thì:

f ( a + b) = f ( f ( a + b − 1)) + f ( a + b − f ( a + b + 1)) = f (b) + f ( a + b − b) = f (b) + b > b

Đây là điều vô lý. Từ đó f không bị chặn trên. Phép chứng minh kết thúc.

Bài toán 6 (Trường xuân miền Nam 2016). Chứng minh rằng với mỗi n ∈ N∗ , n ≥ 2, ta
có thể phân hoạch 2n số tự nhiên đầu tiên thành các tập hợp An , Bn để điều sau được thoả
mãn:
∑ x k = ∑ x k ( k ∈ N, k ≤ n − 1 )
x ∈ An x ∈ Bn

70
Lời giải. Rõ ràng tập hợp 2n số tự nhiên đầu tiên chính là Sn = {0; 1, ...; 2n − 1}. Ta
phân hoạch tập hợp này thành 2 phần là An , Bn sao cho tại An , tổng các chữ số khi
viết trong hệ cơ số 2 của mỗi phần tử là số chẵn, còn ở Bn là phần còn lại.
Dễ thấy rằng mỗi phần tử của Sn sẽ tương ứng được với một dãy nhị phân có không
quá n bit. Hơn nữa, tồn tại một song ánh từ An vào Bn bằng cách thay đổi chữ số cuối
cùng trong hệ cơ số 2 của mỗi phần tử của An : bit 0 thành bit 1 và ngược lại. Do đó
| An | = | Bn | = 2n−1 .
Ta chứng minh bằng quy nạp theo n rằng cách phân hoạch này thoả mãn yêu cầu
bài toán. Thật vậy, dễ kiểm tra điều này với n = 2. Giả sử bài toán đã đúng với
n = l (l ≥ 2). Ta chứng minh rằng bài toán cũng đúng với k = l + 1.
Để thuận tiện về mặt ký hiệu, ta quy ước với A là một tập hợp số thì A + c =
{ a + c | a ∈ A} và cA = {ca | a ∈ A}. Ta nhận thấy được hai điều sau:
(
Al +1 = 2Al ∪ ( Bl + 2l )
Bl +1 = 2Bl ∪ ( Al + 2l )

Từ đây với k ≤ l và 0 ≤ i ≤ k, ta có được:


 
k k l k k k k li k−i
∑ x = ∑ (2x) + ∑ (x + 2 ) =2 ∑ x + ∑ i
2 x
x ∈ A l +1 x ∈ Al x ∈ Bl x ∈ Al x ∈ Bl

  !
k
k li
Viết lại thành ∑ x k = 2k ∑ xk + ∑ 2 ∑ x k−i . Tương tự, ta có:
x ∈ A l +1 x ∈ Al i =0
i x∈B l

  !
k
k ni
∑ x k = 2k ∑ xk + ∑ 2 ∑ x k −i
x ∈ Bl +1 x ∈ Bl i =0
i x∈ A l

Do giả thiết quy nạp mà với k ≤ l − 1 thì ∑ xk = ∑ x k . Điều này cũng đúng khi ta
x ∈ Al x ∈ Bl
nhân thêm hằng số vào các tổng, nên dẫn đến việc ∑ xk = ∑ x k với k ≤ l − 1.
x ∈ A l +1 x ∈ Bl +1
Chỉ còn trường hợp k = l. Viết lại rõ hơn các đẳng thức trên:
  !  !
l l

 l l li l
2

 ∑ xl + ∑ 2 ∑ x l − i = 2l ∑ x l + 2l ∑ x l + ∑ 2ni ∑ x l −i
x ∈ Al i =0
i x ∈ Bl x ∈ Al x ∈ Bl i =1
i x ∈ Bl
 !  !
 l l
 l l li l

2
 ∑ xl + ∑ 2 ∑ x l − i = 2l ∑ x l + 2l ∑ x l + ∑ 2ni ∑ x l −i
x ∈ Bl i =0
i x∈ A x∈B x∈ A i =1
i x∈B
l l l l

Rõ ràng biểu thức sau cùng nhận được của cả hai đẳng thức là bằng nhau. Từ đó:

∑ xl = ∑ xl
x ∈ A l +1 x ∈ Bl +1

71
Bài toán đúng với n = l + 1. Theo nguyên lý quy nạp thì bài toán đúng với mọi
n ∈ N∗ .

Để kết thúc, ta xem xét một bài toán từ tạp chí Pi số tháng 1/2018.

Bài toán 7 (Tạp chí Pi, P137). Với mỗi n ∈ N∗ , gọi Qn là tập hợp tất cả các số hữu tỉ có
m
dạng với m ∈ Z. Tìm tất cả các hàm số f : Q → R và tăng ngặt thoả mãn:
n

∀n ∈ N∗ , ∀ x, y ∈ Qn , ∃z ∈ Qn : f (z) = f ( x ) − f (y)

Lời giải. Do f tăng ngặt nên f là đơn ánh. Để ý rằng Q1 = Z ⊂ Qn ∀n ∈ N∗ .


Theo giả thiết, tồn tại t ∈ Z để f (z) = f ( x ) − f ( x ) = 0 với x ∈ Z. Do f tăng ngặt nên
ta có f (t + 1) > 0, lại nhận thấy nếu f là một hàm số thoả mãn bài toán thì k · f cũng
thế (k ∈ R+ ) nên không mất tính tổng quát, giả sử rằng f (t + 1) = 1.
Cũng từ f tăng ngặt mà f ( x ) > 0 ∀ x > t, x ∈ Q. Theo giả thiết, tồn tại t1 ∈ Z sao cho

f ( t1 ) = f ( t + 2) − f ( t + 1) > 0 = f ( t )

Dễ dàng nhận thấy các bất đẳng thức sau:


(
f ( t + 2) > f ( t + 2) − f ( t + 1) > 0 = f ( t )
f ( t + 2) > f ( t + 1) > f ( t )

Do đó mà f (t + 2) − f (t + 1) = f (t + 1) hay f (t + 2) = 2 f (t + 1) = 2.
Bây giờ ta chứng minh bằng quy nạp rằng f (t + n) = n ∀n ∈ N∗ . Giả sử rằng điều
này đã đúng đến n = k (k ≥ 2). Theo giả thiết, tồn tại z1 , z2 , ..., zk ∈ Z sao cho:

f (zi ) = f (t + k + 1) − f (t + i ) ∀i ∈ {1; 2; ...; k}

Ta có hai dãy bất đẳng thức sau:


(
f (t + k + 1) > f (z1 ) > f (z2 ) > ... > f (zk ) > f (t)
f (t + k + 1) > f (t + k ) > f (t + k − 1) > ... > f (t + 1) > f (t)

Do đó mà f (z1 ) = f (t + k ) hay f (t + k + 1) − f (t + 1) = f (t + k ), dẫn đến f (t + k +


1) = k + 1.
Theo nguyên lý quy nạp thì f (t + n) = n ∀n ∈ N∗ . Để ý rằng f (t − 1) < f (t) = 0.
Bằng việc thiết lập các dãy bất đẳng thức theo chiều ngược lại và quy nạp tương tự,
ta có được:
f ( t − n ) = n f ( t − 1 ) ∀ n ∈ N∗
Bây giờ ta xác định f (t − 1). Tồn tại z ∈ Z sao cho:

f ( z ) = f ( t + 1) − f ( t − 1) = 1 − f ( t − 1) > 1

72
Do đó f (z) ∈ Z, từ đó thì f (t − 1) ∈ Z. Tồn tại l ∈ N∗ sao cho:

l f ( t − 1) = f ( t − l ) = f ( t − 1) − f ( t + 1) < 0

Từ đây f (t + 1) là bội của f (t − 1), mà f (t − 1) < 0 nên f (t −


 1) = 
−1.
1 1 1
Lập các dãy tương tự, ta sẽ có f (t + 1) = n f t + hay f t + = ∀ n ∈ N∗ .
n n n
Bây giờ lại tiếp tục quy nạp với mỗi n ∈ N∗ thì f ( x ) = x − t ∀ x ∈ Q.
Thử lại thì hàm số này thoả mãn. Bởi nhận xét về việc sai khác một hằng số ở phép
nhân, tất cả các hàm số cần tìm là f ( x ) = k ( x − t) ∀ x ∈ Q (k ∈ R+ , t ∈ Z).

2.2. Điều chỉnh địa phương


Điều chỉnh địa phương, đúng như tên gọi, có thể được hiểu rộng là việc xử lý
những điểm nhỏ, lẻ tẻ mà ta gặp phải trong quá trình giải toán. Đó có thể là việc
chứng minh một kết quả bất ngờ phát sinh, hay là cố gắng đưa bài toán về một trường
hợp "gần tốt" nhất có thể - chẳng hạn như việc tìm cực trị của các biểu thức bậc hai
mà ta vẫn gặp trong các bài toán đếm bằng hai cách.
Nguyên tắc nằm trong việc "sai ở đâu, sửa ở đó". Một ví dụ rất tốt là bài toán 5 ở
mục trên. Khi ta thực hiện bước quy nạp, việc giải quyết rẽ nhánh để cuối cùng tất cả
các trường hợp đều thuận lợi chính là minh hoạ của xử lý dữ kiện ở các bước nhỏ để
kết quả tốt nhất.
Ta bắt đầu với các bài toán cực trị.

Bài toán 8. Cho 100 số nguyên dương a1 , a2 , ..., a100 có tổng bằng 2017. Tìm giá trị nhỏ
nhất:
A = a21 + a22 + ... + a2100 .

Lời giải. Không mất tính tổng quát, giả sử a1 ≤ a2 ≤ ... ≤ a100 .
Ta sẽ chứng minh rằng, để A đạt giá trị nhỏ nhất thì a100 − a1 ≤ 1. Thật vậy, gọi
( a1 , a2 , ..., a100 ) là bộ số mà a100 − a1 ≥ 2 để A = a21 + a22 + ... + a2100 đạt giá trị nhỏ
nhất thì xét bộ số sau:

(b1 , b2 , ..., b100 ) = ( a1 + 1, a2 , ..., a100 − 1)

Khi đó thì b1 + b2 + ... + b100 = 2017. Với B = b12 + b22 + ... + b100
2 thì:

B − A = ( a100 − 1)2 + ( a1 + 1)2 − a2100 − a21 = 2 − 2( a100 − a1 ) < 0

Điều này vô lý với tính nhỏ nhất, nên điều giả sử là sai. Do đó bộ số làm cho A nhỏ
nhất phải thoả mãn a100 − a1 ≤ 1. Đến đây, kết hợp với việc tổng của 100 số là 2017
để có được 20 < a100 < 22, ta sẽ dẫn đến:

A ≥ 83 · 202 + 17 · 212 = 40697

73
Đẳng thức xảy ra khi a1 = a2 = ... = a83 = 20 và a84 = a85 = ... = a100 = 21.

Bài toán 9. Cho n ≥ 3 là hằng số. Tìm số nguyên dương k nhỏ nhất sao cho với mọi đồ thị
G có n đỉnh và ít nhất k cạnh thì G là đồ thị liên thông.
 
n−1
Lời giải. Đặt r = + 1. Ta sẽ chứng minh đây là giá trị k cần tìm.
2
Trước hết, tồn tại một đồ thị có n đỉnh và r − 1 cạnh mà không liên thông. Thật vậy, ta
xét G là hợp của một đồ thị đầy đủ có n − 1 đỉnh với một đỉnh treo. Dĩ nhiên G không
liên thông, từ đó ta phải có k ≥ r. Bây giờ ta chứng minh mọi đồ thị có n đỉnh và ít
nhất r cạnh đều liên thông.
Giả sử rằng tồn tại một đồ thị n đỉnh và ít nhất r cạnh mà không liên thông. Gọi V là
tập hợp các đỉnh, E là tập hợp các cạnh và deg(v) là bậc của đỉnh v ∈ V. Kết quả sau
là quen thuộc:
∑ deg(v) = 2r
v ∈V

Do đó với v ∈ V bất kỳ thì:

2r (n − 1)(n − 2) + 1
max deg(v) ≥ = > n−3
n n

Gọi đỉnh có bậc lớn nhất là u thì có các khả năng sau xảy ra:
• Nếu deg(u) = n − 1 thì G liên thông, trái với điều giả sử.
• Nếu deg(u) = n − 2, gọi w là đỉnh không kề với u. Rõ ràng tất cả các đỉnh còn
lại của G đều kề với u, và nếu w kề với bất kỳ một đỉnh nào khác thì G sẽ liên
thông, trái với điều giả sử. Từ đó w phải là một đỉnh treo, dẫn đến số cạnh tối
đa của G là:    
n−1 n−1
r= + 1 ≤ | E| ≤
2 2
Đây cũng là một điều vô lý.
 
n−1
Tóm lại điều giả sử là sai hay G phải liên thông. Vậy k = r = là giá trị cần
2
tìm.
Hai ví dụ tiếp theo sẽ lạ mắt hơn khi các thao tác dựa trên lập luận số học:

Bài toán 10. Với mỗi số nguyên dương m, đặt S(m) là tổng các chữ số của m khi viết trong
hệ thập phân. Tìm tất cả n ∈ N∗ để với mỗi k ≥ n, tồn tại bội số m của n mà S(m) = k.

Lời giải. Với mỗi k ≥ n, đặt m = f (k ) · n. Ta có:

k = S(m) ≡ m ≡ f (k) · n (mod 9)

Do k nhận mọi giá trị lớn tuỳ ý nên k sẽ đi qua một hệ thặng dư đầy đủ modulo 9. Từ
đây thấy ngay rằng (n, 9) = 1 hay (n, 3) = 1 chính là điều kiện cần.

74
Ta sẽ chứng minh rằng đây cũng là điều kiện đủ bằng cách xây dựng m.
Trước hết ta giải bài toán với (n, 10) = 1. Theo định lý Euler, ta có:

10sϕ(n) ≡ 1 (mod n) ∀s ∈ N∗

Từ đây dễ có được 10sϕ(n)+1 ≡ 10 (mod n) ∀s ∈ N∗ .


Ta sẽ xây dựng m có toàn các chữ số 1 sao cho S(m) = k. Nghĩa là ta chọn ra k số hạng
phân biệt gồm các dạng 10sϕ(n) , 10sϕ(n)+1 để cộng lại với nhau, tạo thành m. Do s có
thể lấy tuỳ ý trên N∗ nên tính phân biệt là hiển nhiên. Ta chỉ cần phải kiểm tra điều
kiện chia hết.
Gọi số các số hạng mỗi loại lần lượt là a, b ≥ 0 thì ta có hệ phương trình sau:
(
a+b = k ≥ n
a + 10b ≡ 0 (mod n)

Nếu k là bội của n thì chọn a = k, b = 0. Ngược lại, ta có 9b ≡ −k (mod n) và


(n, 9) = 1 nên b là nghịch đảo của 9 theo modulo n. Dĩ nhiên chọn được b ≤ n nên
tồn tại a ≥ 0 thoả mãn.
Từ đó ta đã xây dựng được m để S(m) = k và m là bội của n với (n, 10) = 1. Nếu
(n, 10) > 1, viết n = 2α · 5β · p với ( p, 10) = 1. Xây dựng m thoả mãn với p và có
S(m) = k ≥ n ≥ p, rồi chọn m0 = 10α+ β m thì số m0 có S(m0 ) = S(m) = k ≥ n và m0
là bội của n.
Vậy tất cả các số nguyên dương n thoả mãn bài toán là (n, 3) = 1. Bài toán kết
thúc.

Bài toán 11. Trên mặt phẳng toạ độ có một đa giác lồi n đỉnh (n > 2) và toạ độ các đỉnh đều
là số hữu tỉ. Giả sử rằng đa giác có tất cả các cạnh bằng nhau. Chứng minh rằng n là số chẵn.

Lời giải. Đầu tiên, xét bài toán với trường hợp các đỉnh có toạ độ nguyên. Gọi một
đỉnh nào đó là A1 ( x1 , y1 ) và gọi các đỉnh lần lượt là A2 , A3 , ..., An theo chiều kim đồng
hồ. Bởi độ dài các cạnh bằng nhau, mà ta sẽ tạm gọi là ` > 0, theo công thức tính
khoảng cách thì:

`2 = ( x1 − x2 )2 + (y1 − y2 )2 = ... = ( xn−1 − xn )2 + (yn−1 − yn )2 = ( xn − x1 )2 + (yn − y1 )2

Dĩ nhiên `2 là một số nguyên và là tổng của hai số chính phương.


Xét đẳng thức trên theo modulo 4. Quy ước xn+1 = x1 , yn+1 = y1 . Có 3 khả năng sau
xảy ra:

• Nếu `2 ≡ 2 (mod 4) thì ta có xi − xi+1 ≡ yi − yi+1 ≡ 1 (mod 2) (1 ≤ i ≤ n). Từ


đó:
x1 ≡ x2 + 1 ≡ ... ≡ x1 + n (mod 2)
Điều này dẫn đến n là số chẵn.

75
• Nếu `2 ≡ 1 (mod 4) thì ta có xi − xi+1 + yi − yi+1 ≡ 1 (mod 2) (1 ≤ i ≤ n). Từ
đó:
n n
n≡ ∑ (xi − xi+1 ) + ∑ (yi − yi+1 ) = 0 (mod 2)
i =1 i =1

• Nếu `2 ≡ 0 (mod 4) thì xi − xi+1 ≡ yi − yi+1 ≡ 0 (mod 2). Điều này nghĩa là các
giá trị ( x1 , x2 , ..., xn ) và (y1 , y2 , ..., yn ) lập thành 2 bộ số mà mỗi bộ đều chẵn hoặc
lẻ. Bây giờ ta tịnh tiến đa giác theo một vector ~v để x1 , x2 , .., xn , y1 , y2 , ..., yn đều
chẵn. Vị tự tâm O là gốc toạ độ để có đa giác mới A10 A20 ...A0n với 2xi0 = xi , 2yi0 = yi
(1 ≤ i ≤ n ).
Nếu đa giác mới có toạ độ thoả mãn 1 trong 2 trường hợp đầu thì bài toán sẽ
xong. Ngược lại, nếu luôn rơi vào trường hợp 3, ta sẽ luôn thu được đa giác mới
có toạ độ nguyên độ dài cạnh mà mỗi lần giảm ít nhất một nửa so với đa giác
ban đầu. Nhưng điều này là không thể vì khoảng cách nhỏ nhất giữa 2 điểm
nguyên luôn là 1 đơn vị.

Tóm lại, nếu các đỉnh có toạ độ nguyên thì n là số chẵn. Với trường hợp các đỉnh có
toạ độ hữu tỉ, vị tự với tâm là gốc toạ độ để thu được đa giác có toạ độ nguyên. Bài
toán kết thúc.

Nhận xét. Số nguyên dương n duy nhất thoả mãn bài toán là n = 4. Điều này có thể được
chứng minh bởi định lý Pick, nhưng do vượt quá khuôn khổ bài viết nên không trình bày tại
đây.

Rõ ràng ở bài toán trên, ta đã thực hiện được hai thao tác là đưa về trường hợp
riêng và điều chỉnh các trường hợp con phát sinh. Cụ thể hơn, việc lùi "vô hạn" sẽ dẫn
đến điều vô lý, mà lập luận thường gặp nhất dựa trên một dãy số nguyên dương. Dĩ
nhiên trong nhiều bài toán khác, ta cũng làm việc với các tình huống tương tự, nhưng
sẽ phức tạp hơn hẳn.
Bài toán IMO Shortlist 2017 A7 tiếp theo sẽ cho thấy rõ hơn sự phức tạp đó.

Bài toán 12 (IMO Shortlist 2017). Cho a0 , a1 , a2 , ... là dãy vô hạn các số nguyên và
b0 , b1 , b2 , ... là dãy vô hạn các số nguyên dương, đồng thời thoả mãn điều kiện sau:
(
an bn + an−1 nếu bn−1 = 1
a0 = 0, a1 = 1, an+1 = ∀ n ∈ N∗
an bn − an−1 nếu bn−1 > 1

Chứng minh rằng ít nhất một trong hai số a2017 , a2018 không nhỏ hơn 2017.

Lời giải. Với giả thiết, do a0 = 0 nên a2 = a1 b1 + a0 = b1 ≥ 1 ∀b0 ∈ N∗ .

• Nếu b1 = 1 thì a3 = a2 b2 + a1 = a2 + a1 ≥ 2
• Nếu b1 ≥ 2 thì a2 = b1 ≥ 2 và a3 = a2 b2 − a1 ≥ 2 − 1 = 1

Cả hai trường hợp thì ta đều có max{ a2 , a3 } ≥ 2. Cụ thể hơn thì a1 , a2 , a3 ∈ N∗ .

76
Bây giờ ta chứng minh rằng an ∈ N∗ ∀n ≥ 2 bằng nguyên lý quy nạp. Giả sử rằng
điều này đã đúng đến n = l (n ≥ 3). Có các khả năng sau xảy ra:

• Nếu bl −1 = 1 thì al +1 = al bl + al −1 ∈ N∗ .
• Nếu bl −1 ≥ 2 thì al +1 = al bl − al −1 . Đến đây ta lại rẽ nhánh thành các khả năng
sau:

– Nếu bl −2 = 1 thì al = al −1 bl −1 + al −2 ∈ N∗ .
Hơn nữa al ≥ al −1 + 1 nên al +1 = al bl − al −1 ≥ al − al −1 = 1 hay al +1 ∈ N∗
– Nếu bl −2 ≥ 2 thì al = al −1 bl −1 − al −2 . Đến đây ta lại rẽ nhánh...

Quá trình rẽ nhánh luôn tạo thành 2 khả năng: một khả năng giải quyết được trọn
vẹn, và khả năng còn lại là có chỉ số s để bs ≥ 2 với s giảm ngặt. Dãy chỉ số là một dãy
nguyên, không âm và giảm ngặt nên phải có lúc dừng. Trường hợp tệ nhất chính là
khi b0 ≥ 2. Lúc đó thì:
min{b0 , b1 , ..., bl −1 } ≥ 2
Chú ý rằng a2 = b1 ≥ 2 ≥ a1 + 1 nên thực hiện quá trình "truy ngược" lên, ta sẽ có:

ai ≥ ai−1 bi−1 − ai−2 ≥ 2ai−1 − ai−2 ≥ ai−1 + 1 ∀i ∈ {2; ...; l + 1}

Nói riêng thì al +1 ≥ al + 1 và al +1 ∈ Z nên al +1 ∈ N∗ .


Từ đây theo nguyên lý quy nạp thì an ∈ N∗ ∀n ≥ 2. Chú ý ở quá trình quy nạp, ta đã
có:

"Nếu có các chỉ số s, l để bs = 1 và bs+1 , ..., bs+l ≥ 2 thì as+1 < as+2 < ... <
a s + l +2 "

Bây giờ ta chứng minh rằng max{ an+2 , an+3 } ≥ max{ an , an+1 } + 2 ∀n ≥ 1. Có các
khả năng:
(
an+2 = an+1 bn+1 + an ≥ 1 + max{ an , an+1 }
• Nếu bn = bn+1 = 1 thì
an+3 = an+2 bn+2 + an+1 ≥ 1 + an+2 ≥ 2 + max{ an , an+1 }
• Nếu bn = 1 và bn+1 ≥ 2 thì:

max{ an+2 , an+3 } ≥ an+2 = an+1 bn+1 + an ≥ 2an+1 + an ≥ 2 + max{ an , an+1 }

• Nếu bn , bn+1 ≥ 2 thì theo phần chứng minh trên, ta có:

– Nếu bn−1 = 1 thì an < an+1 < an+2 < an+3 , do đó thì:

max{ an+2 , an+3 } ≥ 2 + an+1 = 2 + max{ an , an+1 }

– Nếu bn−1 ≥ 2 thì ta cũng có điều tương tự.


• Nếu bn ≥ 2 và bn+1 = 1, ta có các khả năng sau phải xem xét:

77
– Nếu bn−1 = 1 thì do bn ≥ 2 nên an < an+1 < an+2 . Từ đó thì:

a n + 3 = a n + 2 bn + 2 + a n + 1 > a n + 2

Từ đó an < an+1 < an+2 < an+3 nên max{ an+2 , an+3 } ≥ max{ an , an+1 } +
2.
– Nếu bn−1 ≥ 2 thì ta cũng có an < an+1 < an+2 < an+3 nên cũng xong.

Tóm lại ta đã chứng minh được max{ an+2 , an+3 } ≥ max{ an , an+1 } + 2 ∀n ≥ 1.
Áp dụng liên tiếp thì max{ a2017 , a2018 } ≥ max{ a1 , a2 } + 2016 ≥ 2017. Vậy phải có ít
nhất một trong hai số a2017 , a2018 không nhỏ hơn 2017. Phép chứng minh kết thúc.
Những ví dụ được giới thiệu đều có điểm chung là ta cố gắng tìm một (hoặc một
vài) yếu tố hữu hạn được ẩn giấu. Để kết thúc mục này, bạn đọc hãy tự mình xem xét
bài tập 8, đó là một bài toán hay, đòi hỏi sự lao động tỉ mỉ và có nhiều thao tác "ở vô
cùng".

2.3. Tham lam


Ý tưởng tham lam thường gợi các suy nghĩ về thuật toán. Thật ra không hẳn như
thế, đơn giản chỉ là việc chọn thật nhiều các đối tượng thoả mãn bài toán, để việc
chọn là tối đa theo tiêu chí nào đó. Một ví dụ dễ hiểu là nguyên lý cực hạn. Cụ thể
hơn một chút, trong lĩnh vực đồ thị hữu hạn, để chứng minh các định lý về đường đi
và chu trình Hamilton thì ta thường đề cập đến đường đi dài nhất. Nhưng ý tưởng
tham lam không chỉ dừng lại ở cực hạn và các biến thể.
Để có thể hình dung rõ hơn, ta xét bài toán sau:

Bài toán 13. Trong các số có dạng k! (1 ≤ k ≤ 50), cần phải loại bỏ ít nhất bao nhiêu số để
các số còn lại có tích là số chính phương?

Lời giải. Xét tích sau:

50 25 25
∏ i! = ∏(2i − 1)! · (2i)! = 225 · 25! · ∏((2i − 1)!)2
i =1 i =1 i =1

Bằng một vài tính toán, ta có 2 · 25! · k! phải là số chính phương với 1 ≤ k ≤ 50 nếu
chỉ loại đi một số là k!, nhưng đây là điều vô lý, nên ta phải loại bỏ ít nhất hai số. Hơn
nữa khi loại bỏ 2! và 25! thì tích còn lại là số chính phương. Vậy cần loại bỏ ít nhất hai
số.

Bài toán 14. Cho A = {1; 2; ...; 2018}. Cần phải loại khỏi A ít nhất bao nhiêu phần tử đề
trong phần còn lại, không có phần tử nào bằng tích 2 phần tử khác?

Lời giải. Ta chứng minh rằng cần loại bỏ ít nhất 43 phần tử. Một ví dụ là loại bỏ
{2; 3; ...; 44} để thu được tập hợp {1; 45; 46; ...; 2018}. Rõ ràng tích của hai phần tử bất
kỳ hoặc là bằng chính nó, hoặc là vượt quá 452 = 2025 > 2018.

78
Ta chứng minh rằng mọi cách loại bỏ 42 phần tử đều không thoả mãn yêu cầu. Xét
các bộ ba có dạng (k; 89 − k; k (89 − k)) với 2 ≤ k ≤ 44. Các phần tử được liệt kê đều
là phân biệt, và có 43 bộ ba, nên nếu loại bỏ 42 phần tử khỏi A thì vẫn còn ít nhất một
bộ ba, dẫn đến điều mâu thuẫn.
Vậy số phần tử cần loại bỏ ít nhất là 43 phần tử.

Trong lời giải trên, sự chọn lựa 43 phần tử là một việc khá liều lĩnh, bởi vì số này
rất nhỏ so với kích thước của tập hợp A. Việc nhìn nhận theo hướng tối đại (hoặc tối
thiểu) sẽ thường dẫn đến những kết luận không ngờ, và việc kiểm chứng tính đúng
đắn phụ thuộc vào kinh nghiệm của người giải. Bài toán tiếp theo, rất giống với bài 1
của Việt Nam TST 2015, sẽ cho thấy điều đó:

Bài toán 15 (Nga 2014). Kho bạc của nước "Cộng hoà toán học" chọn một số thực α > 2
rồi phát hành các đồng xu có mệnh giá là 1 rub và αk rub ∀k ∈ N∗ , hơn nữa αk là số vô tỉ
∀k ∈ N∗ . Liệu có xảy ra việc sau hay không: ∀n ∈ N∗ , ta luôn chọn được một số đồng xu có
tổng mệnh giá bằng n và mỗi loại được chọn không quá 6 đồng?

Lời giải. Rất ngạc


√ nhiên, câu trả lời ở đây là khẳng định.
−1 + 29
Xét α = > 2 là nghiệm dương của phương trình x2 + x = 7. Dễ thấy α là
2
số vô tỉ. Bây giờ ta chứng minh rằng αk là số vô ∗
√ tỉ ∀k ∈ N .
1 + 29
Giả sử rằng tồn tại k để αk ∈ Q. Đặt β = thì αβ = 7 hay αk βk = 7k . Từ việc
2
αk ∈ Q thì βk ∈ Q. Hơn nữa do βk > 0 nên βk ∈ Q+ . Với m, n, p, q ∈ Q+ , xét phân
tích sau:
√ √ √
(m + n 29)( p + q 29) = (mp + 29nq) + (np + mq) 29 6∈ Q+

Áp dụng liên tục điều này thì βk 6∈ Q+ , trái với điều giả sử. Do đó điều giả sử là sai,
nghĩa là αk 6∈ Q ∀k ∈ N∗ .
Giờ ta chỉ ra thuật toán chọn bộ xu có tổng mệnh giá là n bằng nguyên lý quy nạp. Dễ
thấy với n ≤ 7 thì ta chọn được bộ xu. Giả sử rằng chọn được bộ xu với tổng mệnh
giá k ≥ 7, ta xây dựng bộ xu với tổng mệnh giá k + 7 như sau:

• Bổ sung 7 đồng 1 rub vào bộ xu có tổng mệnh giá bằng k.


• Nếu có mệnh giá αl (l ∈ N) có ít nhất 7 đồng xu, do đẳng thức 7αl = (α2 + α)αl ,
ta thay thế 7 đồng xu αl trong bộ xu bởi 2 đồng xu là αl +2 + αl +1 .

Do số đồng xu giảm ngặt nên thuật toán dừng sau hữu hạn bước. Sau cùng, ta sẽ thu
được bộ xu thoả mãn với tổng mệnh giá k + 7. Theo nguyên lý quy nạp, ∀n ∈ N∗ , ta
luôn chọn được một số đồng xu có tổng mệnh giá bằng n, mỗi loại được chọn không
quá 6 đồng. Bài toán kết thúc.

Bài toán 16 (Trung Quốc TST 2015). Cho X 6= ∅ là tập hữu hạn. Xét các tập hợp
A1 , A2 , ..., Ak ⊂ X sao cho | Ai | ≤ 3 ∀i ∈ {1; 2; ...; k }. Giả sử rằng mỗi phần tử của X thuộc

79
 
3k
ít nhất 4 tập hợp con. Chứng minh rằng có thể chọn ra được tập con trong bộ trên mà
7
có hợp là X.

Lời giải. Ta sẽ chọn các tập hợp bởi thuật toán sau:

• Chọn họ tập hợp cực đại từ X gồm các tập hợp mà mỗi lượt ta thêm một tập
vào họ, số phần tử của hợp các tập hợp đó tăng thêm 3 đơn vị. Không mất tính
m
[
tổng quát, xem như đó là các tập hợp A1 , ..., Am . Đặt X3 = Ai .
i =1
Từ đó với i > m thì | Ai ∩ ( X \ X3 )| ≤ 2 bởi tính cực đại.
• Tiếp theo, chọn họ tập hợp cực đại từ X \ X3 gồm các tập hợp mà mỗi lượt ta
thêm một tập vào họ, số phần tử của hợp các tập hợp đó tăng thêm 2 đơn
vị. Không mất tính tổng quát, xem như đó là các tập hợp Am+1 , ..., Am+n . Đặt
m[
+n
X2 = Ai .
i = m +1
Từ đó với i > m + n thì | Ai ∩ ( X \ X3 \ X2 )| ≤ 1 bởi tính cực đại.
• Cuối cùng, chọn các tập hợp từ X \ X3 \ X2 gồm các tập hợp mà mỗi lượt ta thêm
một tập vào họ, số phần tử của hợp các tập hợp đó tăng thêm 1 đơn vị. Chọn
đến khi hợp tất cả các tập hợp bằng X. Giả sử ta đã chọn p tập hợp. Gọi hợp của
họ này là X1 .
3k
Ta cần chứng minh rằng m + n + p ≤ , chú ý là 3m + 2n + p = | X |.
7
• Mỗi phần tử của X1 thuộc ít nhất 4 tập hợp, mà | Ai ∩ ( X \ X3 \ X2 )| ≤ 1 (i >
m + n) nên:
k ≥ m + n + 4p

• Mỗi phần tử của X1 ∪ X2 thuộc ít nhất 4 tập hợp, mà | Ai ∩ ( X \ X3 )| ≤ 2 (i > m)


nên:
4(2n + p)
k ≥ m+
2
• Mỗi phần tử của X thuộc ít nhất 4 tập hợp, mà | Ai | ≤ 3 với 1 ≤ i ≤ k nên:

4(3m + 2n + p)
k≥
3

Từ đây ta có:
 
4(2n + p) 4(3m + 2n + p)
59k ≥ 20(m + n + 4p) + 12 m + + 27 ·
2 3
 
59k 3k 3k
Hay là m + n + p ≤ < . Vậy ta đã chọn được tập con mà có hợp là X.
140 7 7

80
Ta kết thúc bằng một bài toán rất khó từ tạp chí KöMaL của Hungary:

Bài toán 17 (KöMaL, A.713). Dãy số vô hạn phần tử a1 , a2 , ... được gọi là tốt nếu như:

1
∀ j ∈ N∗ , ∀ i < j : | a i − a j | ≥
j

Tìm tất cả các số thực c sao cho tồn tại một dãy số tốt trong đoạn [0, c].

Lời giải. Dễ thấy các số hạng là phân biệt. Với mỗi n ∈ N∗ , ta đặt dãy số

0 ≤ ai1 < ai2 < ... < ain

là hoán vị được xếp theo thứ tự tăng dần của a1 , a2 , ..., an với các chỉ số là hoán vị của
(1, 2, .., n).
Ta có bất đẳng thức sau:

n −1 n −1
1
c ≥ a i n − a i1 = ∑ ( a i k +1 − a i k ) ≥ ∑ min{ik , ik+1 }
k =1 k =1

Mỗi số nguyên dương trong {1; 2; ...; n} chỉ xuất hiện tối đa 2 lần trong tất cả các mẫu
số, nên để tổng thu được cuối cùng là nhỏ nhất thì mỗi số trong {1; 2; ...; n} phải xuất
hiện 2 lần, theo thứ tự ưu tiên từ số lớn nhất đến số nhỏ nhất.
Với n chẵn, đặt n = 2t (t ∈ N∗ ) thì:

2t
1
c≥2 ∑ k
k = t +2

Cho t → +∞ thì ta có c ≥ 2 ln 2. Với n lẻ, tương tự ta cũng có c ≥ 2 ln 2.


Bây giờ ta chỉ cần xây dựng dãy số với c = 2 ln 2. Ý tưởng xây dựng như sau:

• Đầu tiên, chọn a1 = 0, a2 = 2 ln 2. Ta có 1 khoảng với đầu mút là a1 , a2 .


• Chọn a3 nằm giữa a1 , a2 . Ta có 2 khoảng rời nhau, nên khi xếp a4 , a5 vào đó lần
lượt từ trái qua thì sẽ thu được 4 khoảng. Tương tự, khi có 2k + 1 số hạng đầu
tiên của dãy số xếp để tạo thành đầu mút của 2k khoảng, ta tiếp tục xếp 2k phần
tử a2k +2 , ..., a2k+1 +1 vào các khoảng theo thứ tự từ trái qua.
• Bây giờ chọn giá trị các số hạng để cách xây dựng thoả mãn. Chẳng hạn như:

2k −1 −1
1
a3 = 2 · lim
k→+∞
∑ 2l + 2 + l
l =1

Giới hạn này dễ kiểm chứng rằng có tồn tại.

Việc xây dựng các số hạng khá phức tạp nên sẽ được để lại như bài tập.
Tóm lại, tất cả các số thực c thoả mãn bài toán là c ≥ 2 ln 2. Bài toán kết thúc.

81
3. BÀI TẬP
Bài tập 1. Giải lại bài toán 1 khi các số được xếp trên một vòng tròn.
Bài tập 2 (Bulgaria TST 2003). Liệu với mọi hoán vị ( a1 , a2 , ..., a2002 ) của (1, 2, ..., 2002),
có luôn tồn tại 1 ≤ m < n ≤ 2002 để m, n có cùng tính chẵn lẻ và 2a m+n = am + an ?
2
Bài tập 3. Một đa giác lồi được chia thành các tam giác cân bởi các đường chéo mà
đôi một không có điểm trong chung. Chứng minh rằng đa giác có ít nhất hai cạnh
bằng nhau.
Bài tập 4. Chứng minh rằng với a1 , a2 , ..., an là các số nguyên dương phân biệt thì:
!2
n n
∑ ( a7i + a5i ) ≥2 ∑ a3i
i =1 i =1

Bài tập 5 (Nga 2017). Cho các số thực dương a1 , a2 , ..., a2017 . Chứng minh rằng ta có
thể tìm được các số thực dương b1 , b2 , ..., b2017 thoả mãn đồng thời các điều kiện sau:
i. bi ≥ ai ∀i ∈ {1; 2; ...; 2017}
bi b j
ii. Với 1 ≤ i < j ≤ 2017 thì ít nhất một trong hai số , là số nguyên.
b j bi
iii. b1 b2 ...b2017 ≤ 21008 a1 a2 ...a2017
Bài tập 6 (RMM 2015). Cho đa giác đều n đỉnh với n ≥ 5. Ban đầu có 3 tấm thẻ được
đặt trên 3 đỉnh kề nhau của đa giác. Có 2 người cùng trò chơi sau, luân phiên mỗi
người 1 lượt: mỗi người đến lượt đi của mình dịch chuyển tấm thẻ dọc theo các đỉnh
của đa giác, không được nhảy qua bất kỳ một tấm thẻ nào sao cho diện tích của tam
giác có 3 đỉnh ở vị trí 3 tấm thẻ luôn tăng thật sự so với lượt chơi trước. Ai không thực
hiện được nước đi ấy nữa sẽ thua cuộc. Xác định tất cả các giá trị n để người chơi đầu
tiên luôn có chiến thuật thắng.
Bài tập 7 (Định lý Cauchy - Davenport). Với p là số nguyên tố lẻ, đặt Z p = {0, 1, ..., p −
1}. Gọi A, B là hai tập hợp con bất kỳ của Z p . Khi đó:

| A + B| ≥ min{ p, | A| + | B| − 1}

Với quy ước A + B = { a + b | a ∈ A, b ∈ B} và các phần tử của A + B được lấy từ Z p .


Bài tập 8 (Bulgaria 2003). Tìm tất cả các đa thức hệ số nguyên P( x ) thoả mãn điều
kiện sau: với mỗi số nguyên dương n, tồn tại số nguyên x để P( x ) = 2n .
Bài tập 9. Cho số nguyên dương m > 1. Chứng minh rằng tồn tại đa thức hệ số
nguyên P( x ), bất khả quy trên Z[ x ], có bậc 2018 và một nghiệm thực là r thoả mãn
m | 1 + b r n c ∀ n ∈ N∗ .
Bài tập 10. Cho trước số nguyên dương n ∈ N∗ . Tính số lượng các đa thức có bậc
không vượt quá n, có tất cả các hệ số thuộc {0; 1; ...; 8} và f (3) = n.
Bài tập 11. Cho bảng ô vuông n × n. Tô màu đen một số ô vuông trong bảng.
1. Xét n là số chẵn và mỗi ô vuông trong bảng đều có cạnh chung với ít nhất 1 ô
đen. Hãy tìm số lượng ô đen nhỏ nhất có thể trên bảng ô vuông.

82
2. Xét n ≥ 5 và mỗi ô vuông không được tô màu đều có cạnh chung với ít nhất 1 ô
đen. Gọi f (n) là số ô đen nhỏ nhất. Đặt n + 2 ≡ r (mod 5) với 1 ≤ r ≤ 5. Chứng
minh rằng:
 2  2  2 !
n−3 n+2 n−3
f (n) ≤ 5 +r − −4
5 5 5

Liệu bất đẳng thức này có phải là đánh giá tốt nhất không ?

Bài tập 12. Ta biết rằng tồn tại đẳng thức sau:

1 π2
∑ n2 6 =
n =1

π2   [ h π2 
Cho q là số hữu tỉ và q ∈ −1 0, 1, . Chứng minh rằng tồn tại các số
6 6
nguyên dương phân biệt q1 < q2 < ... < qk để đẳng thức sau được thoả mãn:

1 1 1
q= 2
+ 2 + ... + 2
q1 q2 qk

Bài tập 13. Cho 100 điểm nằm trong hình vuông đơn vị. Chứng minh rằng ta có thể
nối chúng bởi 1 đường gấp khúc có độ dài không vượt quá 20.
Bài tập 14. Cho n điểm A1 , A2 , ..., An nằm trong hình vuông đơn vị. Gọi d1 , d2 , ..., dn
lần lượt là khoảng cách từ mỗi điểm A1 , A2 , ..., An đến điểm gần nhất. Chứng minh
rằng:
d21 + d22 + ... + d2n ≤ 4
Bài tập 15 (USA TSTST 2018). Chứng minh rằng tồn tại một hằng số c < 1 sao cho với
mọi đa giác P có diện tích bằng 1, ta có thể tịnh tiến P theo một chiều xác định nào
đó để thu được đa giác Q mà diện tích chung giữa P , Q không vượt quá c.

Lời kết
Bài viết đã giới thiệu một số bài toán điển hình cho những hướng tiếp cận tổng
quát nhất khi giải toán Olympic. Dĩ nhiên để giải quyết những bài toán phức tạp, ta
không thể phủ nhận vai trò của các kiến thức bổ sung, nhưng việc rèn luyện với các
vấn đề tổng quát nhất sẽ giúp tư duy được trong sáng và sắc bén hơn. Rất mong nhận
được sự góp ý của bạn đọc về nội dung.
Cuối cùng, tác giả gửi lời cảm ơn đến thầy Lê Phúc Lữ đã đọc lại bản thảo và đưa
ra những góp ý xác đáng để bài viết được hoàn thiện hơn.

TÀI LIỆU THAM KHẢO


1. Các bài toán ôn thi HSG Quốc gia năm 2018 của đội tuyển PTNK.
2. Dongryul Kim, Problems of Week for SSHS Korea 2014, 2015.

83
3. Trần Nam Dũng, Các bài toán ôn tập đội tuyển IMO 2016.
4. IMO Shortlist 2017.
5. Tạp chí Kvant các năm 2014-2017.
6. Tạp chí Pi số 1/2018, chuyên mục Thách thức toán học.
7. Diễn đàn Mathscope, topic Hướng tới Việt Nam TST 2018: http://mathscope.org/showthread.php?t=51642

84
VỀ BÀI TOÁN TỔ HỢP TRONG KỲ THI
CHỌN ĐỘI DỰ TUYỂN PTNK NĂM 2017
Phan Quốc Vượng
Lớp chuyên Toán khoá 2016-2019

LỜI BAN BIÊN TẬP. Bài viết được hoàn thành vào tháng 4/2017 trước khi tác giả
phải chuyển trường bởi lý do cá nhân. Được sự đồng ý của tác giả, ban biên tập
chỉnh sửa và đăng bài viết này dựa trên sự tổng hợp hai bản thảo: bản thảo đầu
tiên rất vắn tắt và có độ dài... một nửa mặt giấy A5, bản thảo còn lại giải thích rõ
hơn những ý tưởng chính với độ dài một mặt giấy A4. Có thể xem đây là một kỷ
niệm đẹp của tác giả với những người bạn: "Một lần là Năng Khiếu, mãi mãi là Năng
Khiếu".

1. BÀI TOÁN GỐC


Trong kỳ thi chọn đội dự tuyển khối 10 trường PTNK năm 2017 xuất hiện bài
toán sau ở vị trí bài toán số 4, cũng là bài toán cuối cùng.

Bài toán 1. Cho S là tập hợp khác rỗng và A1 , A2 , ..., Am (m ≥ 2) là m tập con của S. Gọi
T là tập hợp gồm tất cả các tập Ai 4 A j (1 ≤ i, j ≤ m). Chứng minh rằng | T | ≥ m

Lời giải sau được đề nghị trong kỳ thi bởi Nguyễn Nguyễn.

Lời giải thứ nhất. Trước hết ta chứng minh rằng, nếu M4 N = M 4 P thì N = P. Giả
sử rằng N 6= P. Không mất tính tổng quát, gọi x là phần tử sao cho x ∈ N và x 6∈ P.
Có hai khả năng sau xảy ra:

• Nếu x ∈ M, khi đó ta có x 6∈ M\ N và x 6∈ N \ M, do đó x 6∈ M 4 N, nhưng lại


có x ∈ M4 P nên M4 N 6= M4 P, trái với giả thiết.
• Nếu x 6∈ M, khi đó tương tự ta có x ∈ M4 N và x 6∈ M 4 P, dẫn đến M 4 N 6=
M4 P, trái với giả thiết.

Do đó N = P. Bổ đề được chứng minh. Quay trở lại bài toán ban đầu. Ta sẽ thực hiện
chứng minh bằng quy nạp theo m. Với m = 2, dễ thấy rằng T = {∅, A1 4 A2 } hay
| T | = 2. Bây giờ giả sử rằng bài toán đã đúng đến m = k (k ∈ N ∗ , k ≥ 2), ta sẽ chứng
minh rằng bài toán cũng đúng với m = k + 1. Nếu như với m = k mà ta có | T | = k + 1
thì bài toán hiển nhiên đúng, nên ta chỉ xét trường hợp | T | = k.

85
Xét dãy tập hợp Bi = Ak+1 4 Ai (1 ≤ i ≤ k + 1). Ta bổ sung vào T các tập hợp Bi . Rõ
ràng | T | là đại lượng không giảm. Giả sử rằng vẫn xảy ra | T | = k, do trong dãy tập
hợp vừa thêm có k + 1 tập nên phải tồn tại chỉ số p, q (1 ≤ p < q ≤ k + 1) sao cho
B p = Bq . Từ đó theo bổ đề dẫn đến A p = Aq , trái với giả thiết các tập hợp Ai là đôi
một phân biệt. Do đó điều đã giả sử là sai, hay là | T | ≥ k + 1. Tóm lại ta luôn chứng
minh được | T | ≥ k + 1, quy nạp thành công.
Vậy bài toán được giải quyết hoàn toàn.

Lời giải trên có thể làm gọn hơn hẳn lại như sau mà không cần quy nạp:

Lời giải thứ hai. Ta cũng cần bổ đề ở lời giải thứ nhất.
Xét tập hợp A1 và các tập hợp A1 4 Ai với 1 ≤ i ≤ m. Theo bổ đề trên thì m tập hợp
này đôi một phân biệt nên | T | có ít nhất m phần tử hay | T | ≥ m.

Qua những lời giải trên, ta nhận thấy ý tưởng giải chung đều khá đẹp và trong
sáng, do đó đây không thể xem là bài toán khó. Tuy nhiên với thời gian làm bài ngắn
cùng việc 3 bài toán đầu tiên không hề dễ dàng nên dẫn đến việc rất ít học sinh làm
được bài toán này. Với cách chứng minh trên, ta cũng đồng thời thấy được đánh giá
| T | ≥ m là không chặt, ta sẽ tập trung khai thác phần này.

2. KHAI THÁC BÀI TOÁN GỐC


Một hướng khai thác rất tự nhiên cho bài toán 1 chính là tìm ra một chặn dưới tốt
hơn cho | T |. Điều này nghĩa là ta sẽ tìm biểu thức A = f (m) mà có được đánh giá:

| T | ≥ A = f ( m ) ≥ m ∀ m ∈ N∗ , m ≥ 2

Để dễ dàng, ta xét một trường hợp nhỏ và dễ kiểm soát như S = {1, 2, 3, 4, 5}.
Khi đó, bằng việc thử các giá trị m một cách cụ thể, ta có được các kết quả sau:

• Với m = 2 thì | T | = 2
• Với m ∈ {3, 4} thì | T | ≥ 4
• Với m ∈ {5, 6, 7, 8} thì | T | ≥ 8
• Với m ∈ {9, 10, 11, ..., 16} thì | T | ≥ 16

Từ đây ta mạnh dạn nghĩ đến đánh giá | T | ≥ 2r với r = dlog2 (m)e. Rất thú vị,
đây lại là một nhận xét đúng và chúng ta sẽ chứng minh bài toán sau:

Bài toán 2. Cho S là tập hợp khác rỗng và A1 , A2 , ..., Am (m ≥ 2) là m tập con của S. Gọi
T là họ gồm tất cả các tập Ai 4 A j (1 ≤ i, j ≤ m). Tìm giá trị lớn nhất và nhỏ nhất của | T |.

Lời giải. Nếu không gây bất kỳ nhầm lẫn, ta quy ước XY = X 4 Y. Cách viết này
hoàn toàn hợp lý trong phạm vi bài toán do dễ dàng kiểm chứng được rằng phép lấy
hiệu đối xứng có tính chất giao hoán và kết hợp.

86
Rõ ràng Ai Ai = ∅ (1 ≤ i ≤ m). Ta có Cm 2 cách chọn ra 2 tập hợp phân biệt từ m tập
2 hiệu đối xứng tạo thành
hợp Ai (1 ≤ i ≤ m) cho trước, do đó để | T | lớn nhất thì Cm
từ các cặp phải đôi một phân biệt, từ đó ta có đánh giá:

2 m ( m − 1)
| T | ≤ Cm +1 = +1
2

Đẳng thức xảy ra chẳng hạn khi S = N∗ và Ai = {i } (1 ≤ i ≤ m).


Bây giờ ta tìm giá trị nhỏ nhất của | T |. Với m tập hợp A1 , A2 , ..., Am , ta định nghĩa họ
tập hợp (C1 , C2 , ..., Cl ) là một hệ cơ sở của m tập hợp trên nếu như chúng thoả mãn
đồng thời các điều kiện sau đây:
• Không tồn tại x, y1 , y2 , ..., y j (1 ≤ x, yi ≤ l, 1 ≤ i ≤ j ≤ l − 1) thoả mãn:

Cx = Cy1 Cy2 ...Cyt

• Với mỗi tập hợp Ai (1 ≤ i ≤ m), tồn tại xi1 , xi2 , ..., xi j (1 ≤ j ≤ l ) thoả mãn:

Ai = Cxi Cxi ...Cxi


1 2 j

Nói cách khác, biểu diễn của mỗi tập hợp Ai theo hệ cơ sở trên là duy nhất. Ta định
nghĩa rằng số phần tử của một hệ cơ sở chính là số tập hợp của hệ đó.
Ta khẳng định sự tồn tại của ít nhất một hệ cơ sở với mỗi bộ m tập hợp mà ta chọn.
m
[
Thật vậy, xét tập hợp Ai có các phần tử là a1 , a2 , ..., an thì ta chọn n tập hợp:
i =1

Ci = { ai } (1 ≤ i ≤ n).

Dễ thấy rằng các tập hợp này thoả mãn những tính chất của hệ cơ sở. Do đó sự tồn
tại của hệ cơ sở được khẳng định. Hơn nữa số phần tử của hệ dĩ nhiên là số nguyên
dương nên phải tồn tại một hệ cơ sở có số phần tử nhỏ nhất. Gọi hệ cơ sở có số phần
tử nhỏ nhất đó là (C1 , C2 , ..., Ck ).
Ta sẽ xét hệ cơ sở nhỏ nhất trong toàn bộ phần sau của lời giải.
Nhận xét rằng để biểu diễn được một tập hợp Ai (1 ≤ i ≤ m), mỗi tập hợp Cj
(1 ≤ j ≤ k) có đúng 2 trạng thái là xuất hiện hay không xuất hiện trong biểu diễn.
Mặt khác hệ cơ sở nhỏ nhất có đúng k phần tử nên m ≤ 2k hay k ≥ dlog2 (m)e.
Có 2 khả năng sau có thể xảy ra:
• k = dlog2 (m)e. Điều này nghĩa là 2m > 2k . Ta chứng minh rằng mỗi tập hợp C
trong hệ cơ sở đều có thể biểu diễn được dưới dạng Ai A j (1 ≤ i, j ≤ m). Đến
đây có 2 khả năng:

– Nếu như C = ∅ thì rõ ràng C = Ai Ai với 1 ≤ i ≤ m.


– Nếu như C 6= ∅ thì ta xét các tập hợp Ai và Ai C (1 ≤ i ≤ m). Số tập hợp
vừa chọn là 2n > 2k nên tồn tại 2 tập hợp vừa chọn có cùng biểu diễn trong

87
hệ cơ sở nhỏ nhất. Ta dễ dàng loại các khả năng Ai = A j , Ai C = A j C (do
bổ đề ở bài toán 1) nên tồn tại 2 chỉ số i, j mà 1 ≤ i < j ≤ m mà Ai = A j C.
Từ đó do tính giao hoán:

Ai A j = A j CA j = ( A j A j )C = C

Mặt khác Ai A j ∈ T nên ta có | T | = 2k với k = dlog2 (m)e.


• k > dlog2 (m)e = r. Điều này nghĩa là m < 2k . Giả sử rằng | T | < 2r .
Gọi X, Y lần lượt là họ các tập hợp Ai (1 ≤ i ≤ m) mà Ai chứa Ck và không chứa
Ck trong biểu diễn theo hệ cơ sở. Đặt Q = { Ai A j |1 ≤ i, j ≤ m, Ai ∈ X, A j ∈ Y }.
Rõ ràng Q ⊂ T nên | Q| ≤ | T | < 2r . Gọi R là họ các tập hợp mà có thể biểu diễn
được dưới dạng Cx1 Cx2 ...Cxi (1 ≤ x1 < x2 < ... < xi ≤ k − 1). Tương tự như
trên, ta có | R| = 2k−1 . Mà lại có được:

|{ Ai A j Ck |1 ≤ i, j ≤ m, Ai ∈ X, A j ∈ Y }| = | Q| < 2k−1

Hơn nữa Ai A j Ck biểu diễn được trong hệ cơ sở k phần tử nên tồn tại tập hợp
Z ∈ R mà không thuộc { Ai A j Ck |1 ≤ i, j ≤ m, Ai ∈ X, A j ∈ Y }.
Ta sẽ lập các tập hợp mới bằng cách giữ nguyên các phần tử của Y, các phần tử
của X sẽ được lấy phép hiệu đối xứng với ZCk . Ta dễ thấy rằng các tập hợp mới
có hệ cơ sở là (C1 , C2 , ..., Ck−1 ) và hơn nữa chúng đôi một phân biệt. Bây giờ ta
chứng minh rằng các tập hợp mới đôi một phân biệt với các tập hợp trong họ Y.
Giả sử rằng ta có Ai ZCk = A j với Ai ∈ X và A j ∈ Y. Khi đó:

Ai A j Ck = Ai ( Ai ZCk )Ck = Z

Do đó Z ∈ { Ai A j Ck |1 ≤ i, j ≤ m, Ai ∈ X, A j ∈ Y }, trái với điều đã khẳng định.


Điều này dẫn đến các tập hợp mới tạo ra từ X và các tập hợp thuộc Y đôi một
phân biệt.
Gọi X 0 , Y 0 lần lượt là họ các tập hợp Ai A j (1 ≤ i, j ≤ m), Ai A j chứa và không
chứa Ck trong cách biểu diễn theo hệ cơ sở. Ta nhận thấy rằng, sau khi lập các
tập hợp mới thì | X 0 |, |Y 0 | không đổi. Theo nguyên lý bù trừ, ta lại có:

| T | = | X 0 ∪ Y 0 | = | X 0 | + |Y 0 | − | X 0 ∩ Y 0 |

Hơn nữa ban đầu | X 0 ∩ Y 0 | = 0 nên qua phép biến đổi thì | T | không tăng.
Mặt khác sau phép biến đổi trên, họ các tập hợp lúc này có hệ cơ sở là
(C1 , C2 , ..., Ck−1 ) có số phần tử nhỏ hơn hệ cơ sở ban đầu nên điều này là vô lý.
Vậy việc giả sử | T | < 2r là sai hay | T | ≥ 2r .
Tóm lại, ta luôn có | T | ≥ 2r với r = dlog2 (m)e. Việc chỉ ra trường hợp xảy ra đẳng
thức không hề khó, xin dành lại cho bạn đọc.

88
3. LỜI KẾT
Bài viết trên đã trình bày một hướng khai thác của tác giả về bài toán 4 trong kỳ
thi chọn đội dự tuyển trường PTNK năm 2017. Rất mong nhận được ý kiến đóng góp
của bạn đọc để tác giả có thể chỉnh sửa lại bài viết cho hoàn thiện hơn.
Cuối cùng, tác giả cảm ơn các bạn học sinh cùng lớp 10 Toán PTNK niên khoá
2016-2019 đã đưa ra những ý tưởng thú vị đưa đến bài toán mở rộng và giúp đỡ biên
tập bản thảo bài viết này.

89
VỀ BÀI TOÁN XÁC ĐỊNH HẰNG SỐ
TRONG GIẢI PHƯƠNG TRÌNH HÀM
Nguyễn Mạc Nam Trung
Lớp chuyên Toán khoá 2017 - 2020

GIỚI THIỆU. Đa số các bài toán phương trình hàm trong các kỳ thi học sinh giỏi
đều có nghiệm hàm dễ đoán định như hàm hằng hoặc hàm đa thức,... Tuy nhiên,
nếu đưa tham số vào bài toán hoặc thay đổi các biến thì ta thu được những bài toán
mới có thể có nhiều nghiệm lạ hơn hoặc vô nghiệm. Bài viết này sẽ đề cập một cách
tổng quan về vấn đề xác định tham số để phương trình hàm có nghiệm hoặc một lớp
nghiệm mong muốn. Định hướng chung là ta sẽ dự đoán tham số và cố gắng chứng
minh bởi các kĩ thuật giải phương trình hàm thông thường, tuy nhiên cũng có đôi
khi chúng ta cần phải linh hoạt hơn, như hai bài toán trong phần 3 của bài viết.

1. MỘT SỐ BÀI TOÁN KHỞI ĐỘNG


Ví dụ 1. Tìm tất cả các số nguyên dương a để tồn tại hàm số f : N∗ → N∗ thỏa mãn

f ( x + y) = a( f ( x ) + f (y)) ∀ x, y ∈ N∗ .

Phân tích. Nếu a = 1 thì dễ thấy f ( x ) = x f (1) là nghiệm của phương trình hàm. Ta sẽ thử
áp dụng kĩ thuật trong trường hợp a = 1 để giải quyết bài toán tổng quát, đó chính là quy
nạp. Ta tính thử vài giá trị ban đầu để làm cơ sở như sau:

f (2) = a( f (1) + f (1)) = 2a f (1)


f (3) = a( f (1) + f (2)) = a(2a + 1) f (1)
f (4) = a( f (1) + f (3)) = a( f (1) + a(2a + 1) f (1)) = a(2a2 + a + 1) f (1)

Nếu tiếp tục tính các giá trị sau như f (5), f (6), ... thì sẽ khá phức tạp nhưng nếu quan sát kỹ
thì ta thấy có thể tính f (4) theo cách khác

f (4) = f (2 + 2) = 2a f (2) = 4a2 f (1)

Đến đây việc còn lại là giải phương trình để tìm a.

91
Lời giải. Ta sẽ tính f (4) bằng hai cách. Trước hết tính f (2) và f (3):

f (2) = a( f (1) + f (1)) = 2a f (1)


f (3) = a( f (2) + f (1)) = a(2a + 1) f (1)
f (4) = a( f (1) + f (3)) = 2a f (2)

Từ đây f (1) + f (3) = 2 f (2) hay (2a2 + a + 1) f (1) = 4a f (1), nghĩa là:

f (1)(2a2 − 3a + 1) = 0

1
Giải phương trình, ta được các nghiệm là a = 1 hoặc a = .
2
Nhưng a ∈ N∗ nên ta nhận a = 1. Đây cũng là giá trị cần tìm do tồn tại hàm f
thỏa mãn bài toán khi a = 1, chẳng hạn như f (n) = n.

Nhận xét. Khi giải quyết bài toán ta phát hiện được việc tính bằng hai cách khá thú vị tách
ra khỏi ý tưởng quy nạp. Đây là một kỹ thuật hiệu quả với các bài toán có tham số. Nếu a là số
1
thực thì ta vẫn có thể nhận a = do lúc đó có thể chọn f là hàm hằng.
2
Ví dụ 2. Tìm tất cả các số nguyên dương p sao cho tồn tại hàm số f : N∗ → N∗ thỏa mãn

f ( f (n)) = n + p ∀n ∈ N∗

Phân tích. Dự đoán rằng hàm số thoả mãn bài toán nên là một đa thức, Khi đó deg f = 1 và
sau khi đồng nhất hệ số, ta được
p
f (n) = n +
2
Điều này chỉ có nghĩa khi p là số chẵn, nên ta chỉ còn tìm định hướng cho trường hợp p là
số lẻ, mà thường gặp nhất là chứng minh không tồn tại. Để khai thác được tốt, ta sẽ sử dụng
phép thế và các tính chất của ánh xạ.

Lời giải. Từ giả thiết thì f là đơn ánh. Xét phép đặt các tập hợp sau:

A = { k ∈ N∗ |@ n ∈ N∗ : f ( n ) = k }

B = {k ∈ N∗ |@ n ∈ N∗ : f ( f (n)) = k}
Dễ thấy A ⊂ B và |B| = p. Với phần tử x ∈ B bất kỳ, ta có các khả năng sau

• Nếu x ∈ A thì f ( x ) ∈ B . Thật vậy, nếu f ( x ) 6∈ B thì tồn tại n nguyên dương sao
cho f ( f (n)) = f ( x ). Bởi tính chất đơn ánh mà f (n) = x, vô lý.
• Nếu x 6∈ A thì tồn tại duy nhất số nguyên dương k để f (k ) = x. Khi đó k ∈ A
vì nếu không thì có n nguyên dương để f (n) = k hay x = f ( f (n)), vô lý.

92
Xét ánh xạ g : A → B\A với phép tương ứng x 7→ f ( x ). Từ những lập luận trên thì
dễ thấy g là song ánh, vậy |A| = |B\A| hay |B| = 2|A|, dẫn đến p là số chẵn. Đặt
p = 2c. Khi đó dễ thấy hàm số f (n) = n + c thỏa mãn bài toán. Vậy p là số chẵn.

Ví dụ 3. (Vietnam TST 2004) Tìm tất cả các số thực a sao cho tồn tại duy nhất một hàm số
f : R → R thỏa mãn:

f ( x2 + y + f (y)) = f 2 ( x ) + ay ∀ x, y ∈ R.

Phân tích. Dự đoán f ( x ) = x là nghiệm hàm sẽ dẫn đến a = 2 và mong muốn chứng minh
đây là hằng số duy nhất. Giả thiết nhắc đến điều kiện duy nhất của nghiệm hàm để loại trường
hợp a = 0, vì lúc ấy có ít nhất 2 nghiệm hàm là f ≡ 0 và f ≡ 1. Vậy ta chỉ cần giải bài toán
với a 6= 0. Đây là phương trình hàm có biến tự do nên đến đây việc giải quyết dựa vào vận
dụng khéo léo tính chất ánh xạ và các phép thế biến.

Lời giải. Nếu a = 0 thì có ít nhất hai hàm số thỏa mãn bài toán là f ≡ 0 và f ≡ 1. Do
đó a 6= 0. Thay x bởi 0 vào giả thiết, ta được:

f (y + f (y)) = f 2 (0) + ay ∀y ∈ R (1)

Do a 6= 0 nên từ đây f là toàn ánh. Thay x bởi − x vào giả thiết:

f 2 ( x ) = f 2 (− x ) ∀ x ∈ R

Do f là toàn ánh nên tồn tại b sao cho f (b) = 0. Từ đây ta cũng thấy được f (−b) = 0.
Lần lượt thay y bởi b và −b vào phương trình (1), ta được

f 2 (0) + ab = f 2 (0) − ab = 0

Do a 6= 0 nên b = 0. Từ đây dẫn đến khẳng định sau:

f (x) = 0 ⇔ x = 0 (2)

− f 2 (x)
Thay y bởi vào giả thiết, ta được:
a
  
2 f 2 (x) − f 2 (x)
f x − +f = 0 ∀x ∈ R
a a
 
f 2 (x) − f 2 (x) − f 2 (y)
Kết hợp với (2) thì −x2 +f = 0 ∀ x ∈ R. Thay y bởi vào
a a a
giả thiết và thu gọn, ta thu được:

f ( x2 − y2 ) = f 2 ( x ) − f 2 (y) ∀ x, y ∈ R (3).

93
Thay y bởi 0 thì f ( x2 ) = f 2 ( x ) ∀ x ∈ R nên f ( x ) ≥ 0 ∀ x ≥ 0. Hơn nữa, trong (3) khi
thay x bởi 0 thì ta thu được:

f (−y2 ) = − f (y2 ) ∀y ∈ R

Do đó f là hàm lẻ. Bây giờ ta chứng minh rằng f là hàm cộng tính.

• Với x ≥ 0, y ≤ 0, ta có:
√ p
f ( x + y) = f 2 ( x ) − f 2 ( −y) = f ( x ) − f (−y) = f ( x ) + f (y)

• Với x ≤ 0, y ≥ 0, ta thực hiện tương tự như trên.


• Với x ≥ 0, y ≥ 0, ta có:
p √
f ( x + y) − f (y) = f 2 ( x + y) − f 2 ( y) = f ( x + y − y) = f ( x )

Từ đó thì f ( x + y) = f ( x ) + f (y).

• Với x ≤ 0, y ≤ 0, để ý rằng − x ≥ 0, −y ≥ 0 nên:

f (− x ) + f (−y) = f (− x − y)

Bởi f là hàm lẻ, ta có − f ( x ) − f (y) = − f ( x + y) hay f ( x + y) = f ( x ) + f (y).

Tóm lại f là hàm cộng tính trên R. Ta chứng minh rằng f đơn điệu trên R.

• Nếu x ≥ 0 ≥ y thì f ( x ) ≥ 0 ≥ f (y)


√ √
• Nếu x > y ≥ 0, thay x, y vào (3), ta được:

f ( x ) − f (y) = f ( x − y) ≥ 0

Bất đẳng thức cuối do x − y ≥ 0. Do đó f ( x ) ≥ f (y).


• Nếu 0 ≥ x > y, khi đó 0 6 − x < −y nên f (−y) ≥ f (− x ) hay f ( x ) ≥ f (y).

Do f cộng tính và đơn điệu trên R nên f ( x ) = cx ∀ x ∈ R. Thay vào giả thiết thử lại

c( x2 + y + cy) = c2 x2 + ay ∀ x, y ∈ R

Thay x bởi 1 và thay y bởi 0, ta được c = c2 . Loại trường hợp c − 0, ta phải có c = 1.


Thế ngược lại vào giả thiết, ta thu được 2y = ay ∀y ∈ R hay a = 2. Vậy a = 2 là hằng
số duy nhất thỏa mãn yêu cầu bài toán.

2. SỬ DỤNG DÃY SỐ
Ví dụ 4. Tìm tất cả các số nguyên dương k sao cho tồn tại hàm f : N∗ → N∗ thỏa mãn

f ( f (n)) + f (n) = kn ∀n ∈ N∗

94
Phân tích. Ý tưởng về dãy số có thể thấy rõ trong bài toán này. Khi có các đại lượng
f ( f (n)), f (n) thì việc đặt dãy số bởi u1 = n, un+1 = f (un ) là tự nhiên. Sau đó, khi thay un
vào giả thiết, ta sẽ được một quan hệ hồi quy tuyến tính của dãy số, đến đây có thể sử dụng
phương trình đặc trưng để xử lí.

Lời giải. Với mỗi n ∈ N∗ , xét dãy số bởi u0 = 1, u1 = n, ut+1 = f (ut ). Khi đó dễ thấy
rằng:
ut+2 + ut+1 = kut ∀t ∈ N∗
Phương trình đặc trưng của quan hệ trên là λ2 + λ − k = 0. Từ đó ta có:
r !t r !t
1 1 1 1
ut = a k+ − +b − k+ − ∀ t ∈ N∗
4 2 4 2

Tiếp theo, ta sẽ chứng minh b = 0 bằng việc tính giới hạn dãy chẵn, lẻ khi b < 0, b > 0

• Nếu b < 0, khi đó:


 2t 
r
r !2t 1 1
1 1 
  k+ 4 − 2 
u2t = k+ + b + a  r   
4 2   1 1 
k+ +
4 2

Khi đó dễ thấy cho t → ∞ thì lim u2t = −∞, vô lý.


• Nếu b > 0, khi đó:
 2t+1 
r
r !2t+1 1 1
1 1 
  k+ 4 − 2 

u2t+1 = k+ + 
−b + a  r  
4 2  1 1  
k+ +
4 2

Khi đó dễ thấy cho t → ∞ thì lim u2t+1 = −∞, vô lý.



∗ 4k + 1 − 1
Vậy b = 0. Do u1 ∈ N nên là số nguyên dương, nghĩa là:
2

4k + 1 − 1 = 2y (y ∈ N∗ )

Từ đây dễ dàng giải ra được k = y2 + y với y ∈ N∗ . Ngược lại, nếu k = y2 + y với


y ∈ N∗ thì hàm số f (n) = yn thoả mãn bài toán. Vậy tất cả các số nguyên dương cần
tìm có dạng k = y2 + y (y ∈ N∗ ).

Ví dụ 5. Tìm tất cả các số nguyên dương p sao cho tồn tại hàm số f : N∗ → N∗ thỏa mãn
 
f (n)
f = pn2 ∀n ∈ N∗
n

95
Phân tích. Trước hết ta cần dự đoán p có thể nhận giá trị nào. Trước hết, ta thử chọn f là đa
thức và so sánh bậc thì deg f = 2. Nếu f (n) = cn2 thì:
 
f (n)
f = c3 n2 = pn2
n

Vậy nếu p là lập phương đúng của một số nguyên dương thì yêu cầu được thoả mãn. Vấn đề
nằm ở các trường hợp khác, và ta sẽ tiếp tục sử dụng dãy số. Đặt f (n) = ng(n) để đơn giản
về hình thức thì g : N∗ → N∗ và:

g(n) g( g(n)) = pn2 ∀n ∈ N∗

Để chuyển thành phép cộng, ta sẽ logarith hoá cả hai vế của đẳng thức trên, hơn nữa do có số
p nên ta chọn cơ số p để việc tính toán trở nên gọn gàng.

Lời giải. Nếu p = 1 thì hàm số f (n) = n2 thoả mãn bài toán. Ta xét p > 1.
f (n)
Đặt g(n) = . Từ điều kiện bài toán thì g : N∗ → N∗ và:
n

g( g(n)) g(n) = pn2

Với mỗi n ∈ N∗ , xét dãy số bởi u0 = 1, u1 = n, ut+1 = g(ut ) với t ≥ 1. Khi đó:

ut+2 ut+1 = pu2t ∀t ∈ N∗

Đặt log p ut = vt . Do ut ≥ 1 ∀t nên vt ≥ 0 và v0 = 0. Khi đó:

vt+2 + vt+1 = 2vt + 1 ∀t ∈ N∗

t
Lại đặt wt = vt − thì wt+2 + wt+1 = 2wt . Chú ý rằng từ cách đặt thì w0 = 0. Bây
3
giờ sử dụng phương trình đặc trưng, ta thu được

1 − (−2)t
wt = w1
3
 
1 − (−2)t 1 1 1
Từ đó vt = v1 − + t. Đặt c = v1 − . Có các trường hợp sau xảy ra
3 3 3 3

• Nếu c > 0, thay t bởi 2k, ta được

1 − 4k 2
v2k = c + k ∀ k ∈ N∗
3 3
Cho k → ∞, ta thu được lim v2k = −∞, mà vn ≥ 0 nên vô lí.

96
• Nếu c < 0, thay t bởi 2k + 1, ta được

1 + 2.4k 2
v2k+1 = c+ k
3 3
Cho k → +∞, ta thu được lim v2k+1 = −∞, mà vn ≥ 0 nên vô lí.
1 √
Vậy c = 0 hay v1 = . Khi đó u1 = 3 p nên p = u31 , và ta phải có p là số lập phương
3
đúng. Ngược lại, dễ thấy p là số lập phương đúng thỏa mãn yêu cầu đề bài khi chọn

được hàm số f (n) = 3 p.n2 . Bài toán kết thúc.

Nhận xét. Nếu thay p là một số lập phương đúng lớn hơn 1 và yêu cầu tìm tất cả hàm số
thỏa mãn thì sử dụng phương pháp dãy số, ta có thể giải bài toán triệt để và thu được hàm số
duy nhất như trên. Tuy nhiên, ta lại không sử dụng được phương pháp ấy cho trường hợp
p = 1 vì log1 x không có nghĩa. Việc tìm tất cả hàm số thoả mãn yêu cầu trong trường hợp
p = 1 chính là bài toán trong đề thi chọn đội tuyển PTNK năm 2014.

3. SỬ DỤNG Ý TƯỞNG VỀ ĐIỂM BẤT ĐỘNG


Ví dụ 6. Tìm tất cả các số tự nhiên m sao cho tồn tại hàm số f : R → R thỏa mãn

f ( f ( x )) = x2 − m ∀ x ∈ R

Phân tích. Nếu so sánh bài toán này với hai bài toán hàm hợp phía trên, √ ta sẽ thấy ngay sự
khác biệt về độ khó. Trong trường hợp m = 0, ta có hàm số f ( x ) = | x | 2 với cách chọn cũng
khá tinh tế. Việc m ≥ 0 là một điều kiện ta cần quan tâm kỹ. Ta sẽ sử dụng ý tưởng về dãy số
để tính thử f 4 vì đã có f 2 và f 0 , với quy ước f k = f ◦ f ◦ . . . ◦ f (hợp k lần). Ta có:

f 4 ( x ) = ( x 2 − m )2 − m

Điều đặc biệt là ( x2 − m)2 − x − m chia hết cho x2 − x − m. Có thể tạm gọi là đại lượng
x2 − x − m được bảo toàn qua phép lấy hàm hợp. Ta sẽ tập trung tìm hiểu các nghiệm của đa
thức này thay vì các nghiệm của hàm số, khác với kỹ thuật xử lý mọi khi.

Lời giải. Với m = 0, ta chọn f ( x ) = | x | 2 . Chỉ cần xét khả năng m 6= 0.
Dễ thấy phương trình x2 − x − m = 0 luôn có 2 nghiệm x1 , x2 phân biệt và trái dấu,
do đó f ( f ( x )) có 2 điểm bất động. Ta gọi các giá trị đó là x1 , x2 . Thay x bởi f ( f ( x )):

f 4 ( x ) = f ( f ( f ( f ( x )))) = ( x2 − m)2 − m = ( x2 − x − m)( x2 + x + 1 − m)

Do m ≥ 1 nên phương trình x2 + x + 1 − m = 0 có 2 nghiệm phân biệt. Bây giờ ta sẽ


chứng minh rằng vế phải có 4 nghiệm phân biệt. Thật vậy, giả sử rằng hai phương
trình x2 − x − m = 0 và x2 + x + 1 − m = 0 có nghiệm chung là x0 thì dễ dàng có
1 3
x0 = − và m = . Điều này vô lý do m ≥ 1. Vậy vế phải luôn có 4 nghiệm phân biệt.
2 4

97
Nói cách khác, f 4 ( x ) có 4 điểm bất động phân biệt là x1 , x2 , x3 , x4 . Do f ( f ( x )) có 2
điểm bất động là x1 , x2 nên f ( x1 ), f ( x2 ) nhận giá trị trong tập hợp { x1 , x2 }. Do f 4 ( x )
có 4 điểm bất động là x1 , x2 , x3 , x4 nên kết hợp tính chất trên, ta có được f ( x3 ), f ( x4 )
nhận 2 giá trị trong tập hợp { x1 , x2 , x3 , x4 }. Nếu f ( xi ) = x j (i ∈ {3; 4}, j ∈ {1; 2}) thì
f 3 ( xi ) = x j hay f 4 ( xi ) = f ( x j ) = xi . Do đó f ( x3 ), f ( x4 ) nhận 2 giá trị trong tập hợp
{ x3 , x4 }. Xét các khả năng sau:

• Nếu f ( x3 ) = x3 thì f ( f ( x3 )) = x3 hay x3 là điểm bất động của f 2 ( x ), vô lý.


• Nếu f ( x4 ) = x4 , tương tự như trên thì ta cũng có điều vô lý.
• Nếu f ( x3 ) = x4 , f ( x4 ) = x3 , khi đó f ( f ( x3 )) = x3 , vô lý.

Vậy m = 0 là số tự nhiên duy nhất thỏa mãn bài toán.

3
Nhận xét. Với m ∈ R+ , không tồn tại hàm số thoả mãn bài toán khi m > . Tuy nhiên, việc
4
3
xây dựng hàm số thoả mãn khi m ≤ vẫn chưa được biết.
4
Ví dụ 7. (Baltic Way 1992) Tìm tất cả L sao cho tồn tại song ánh f : N∗ → N∗ thỏa mãn:

f (n)
lim = L.
n

Phân tích. Hàm số đầu tiên chúng ta nghĩ đến là f (n) = n, như vậy thì L = 1. Ta biết rằng
nếu f là song ánh thì f −1 xác định và cũng là một song ánh. Sử dụng tính chất đó, ta thấy
1 1
nếu L là số thực thỏa mãn thì cũng thỏa mãn. Trường hợp L = 0, xem = +∞). Thật vậy
L 0

f (n) f (n) n
L = lim = lim −1 = lim −1 (do f (n) → +∞ khi n → +∞).
n f f ((n)) f (n)

f −1 ( n ) 1 1
Từ đây lim = . Ta có thể nhận thấy 1 là nghiệm dương của phương trình L =
n L L
nên sẽ định hướng việc chứng minh đây là hằng số duy nhất. Để có thêm thông tin, ta sẽ lập
f (n)
luận trên các epsilon để xử lí trường hợp L > 1 và xét luôn trường hợp lim = +∞.
n
Lời giải. Ta có hai nhận xét sau:

(i) Nếu f là một song ánh từ N∗ → N∗ thì lim f (n) = +∞. Điều này hiển nhiên do
f là song ánh nên sẽ không bị chặn trên, do đó f có thể nhận giá trị lớn tuỳ ý.
1
(ii) Nếu L 6= 0 thỏa mãn bài toán thì cũng thế. Thật vậy, ta có:
L
f (n) f (n) n
L = lim = lim −1 = lim −1 (do f (n) → +∞ khi n → +∞)
n f (n) f (n)

f −1 ( n ) 1
Từ đó thì lim = .
n L

98
Trở lại bài toán. Nếu L = 1 thì chọn f (n) = n. Xét L 6= 1. Dễ thấy rằng L ≥ 0.
f (n)
• Với L > 1, do lim = L nên với mọi e > 0, ∃ N sao cho với mọi n ≥ N thì
n

f (n)
− L < e ⇒ f (n) > L − e
n n

f (n)
Chọn 0 < e ≤ L − 1 thì có N sao cho > 1 ∀n ≥ N hay:
n

f (n) ≥ n + 1 ∀n ≥ N

Khi đó f ( x ) chỉ nhận giá trị trong tập hợp {1; 2; . . . ; N } với 1 ≤ x ≤ N − 1. Như
vậy tồn tại một số nguyên dương k sao cho f (n) 6= k với mọi n nguyên dương.
Điều này vô lí vì f là song ánh.
1
• Với 0 < L < 1, nếu L thỏa mãn thì > 1 cũng thế, nhưng điều này vô lí
L
n f −1 ( n )
• Với L = 0, ta nhận thấy 0 = lim −1 , nghĩa là lim = +∞. Do đó tồn
f (n) n
tại N sao cho với mọi n ≥ N thì f −1 (n) > n. Đến đây giải tương tự như trường
hợp ban đầu cũng dẫn đến điều vô lý.

Vậy L = 1 là hằng số duy nhất thoả mãn. Bài toán kết thúc.

Nhận xét. Ta cũng có thể chứng minh trực tiếp sự vô lý trong trường hợp 0 ≤ L < 1 bằng
các lập luận trên epsilon. Hơn nữa f (n) = n không phải hàm số duy nhất thoả mãn, chẳng
hạn ta có thể chọn hàm số bởi f (2n − 1) = 2n và f (2n) = 2n − 1.

4. BÀI TOÁN ĐỀ NGHỊ


Bài toán 1. Tìm tất cả số nguyên dương p sao cho tồn tại hàm số f : N∗ → N∗ thỏa mãn

f ( f (n)) = pn ∀n ∈ N∗

Bài toán 2. Tìm tất cả các số nguyên dương a sao cho tồn tại hàm số f : N∗ → N∗ thỏa mãn

f ( f (m) + f (n)) = a(m + n)∀m, n ∈ N∗

Bài toán 3. (Vietnam 2016) Tìm tất cả các số thực α để tồn tại hàm số f : R → R thỏa mãn

i. f (1) = 2016
ii. f ( x + y + f (y)) = f ( x ) + α ∀ x, y ∈ R.

Bài toán 4. Tìm tất cả các số thực k sao cho tồn tại hàm số khác hằng f : R → R thỏa mãn

f ( a)2 + f (b)2 + f (c)2 = k( f ( a) f (b) + f (b) f (c) + f (c) f ( a))

99
với mọi số thực a, b, c ∈ R thoả mãn a + b + c = 0.

Bài toán 5. Tìm tất cả các số nguyên dương k sao cho tồn tại hàm số f : Z → Z thỏa mãn

f ( x + y + f (y)) = f ( x ) + ky ∀ x, y ∈ Z

Bài toán 6. Tìm tất cả các số nguyên c sao cho tồn tại hàm số f : N∗ → N∗ thỏa mãn

f ( f (n − 1)) = f (n + 1) + c f (n) ∀n ∈ N∗ , n ≥ 2

Bài toán 7. Tìm tất cả các số tự nhiên ( a, b) sao cho tồn tại hàm số f : N∗ → N∗ thỏa mãn

f ( f (n)) = an + b ∀n ∈ N∗

Bài toán 8. Tìm tất cả a ∈ N∗ sao cho tồn tại hàm số f : N∗ → N∗ thỏa mãn

i. f (2) = a.
ii. f (mn) = f (m). f (n) với mọi m, n nguyên dương.
iii. f là hàm số đơn điệu thật sự.

Bài toán 9. Tìm tất cả các số nguyên dương p sao cho tồn tại hàm số f : Z → Z thỏa mãn

f ( f (m) + n) + f (m) = f (n) + f (3m) + p ∀m, n ∈ Z

Bài toán 10. Tìm tất cả các số thực a sao cho tồn tại k > 0 và hàm số f : R → R thỏa mãn:
 
f ( x ) + f (y) x+y
≥ f + k| x − y| a ∀ x, y ∈ R
2 2

TÀI LIỆU THAM KHẢO


1. Titu Andreescu, Iurie Boreico, Functional Equations.
2. Trần Nam Dũng, Dương Bửu Lộc, Phương trình hàm trên N.
3. Chuyên đề Toán học số 10 của trường Phổ thông Năng khiếu.
4. Tạp chí Epsilon, Lời giải và bình luận đề thi VMO 2016.
5. Diễn đàn Art of Problem Solving.

100
VỀ PHƯƠNG PHÁP QUY NẠP TRONG GIẢI TOÁN
HÌNH HỌC TỔ HỢP
Võ Kế Huy-Bùi Duy Khang
Lớp chuyên Toán khoá 2017 - 2020

1. DẪN NHẬP
Hình học tổ hợp là một lĩnh vực chuyên xét tính tổ hợp từ các phần tử hình học,
thường tập trung giải quyết các bài toán về sự sắp xếp các phần tử hình học và các
tính chất đặc thù của nó, điển hình như tính chất bao, tô màu, phủ và chia cắt hình,
hơn nữa tính "hình học" chỉ làm nền. Để giải quyết các bài toán có số số hạng cao, các
mệnh đề có tính tổng quát, ta thường sử dụng phương pháp quy nạp. Để áp dụng
hiệu quả phương pháp quy nạp trong giải toán, ta cần phân tích kĩ lưỡng vấn đề, và
chứng minh chặc chẽ các khẳng định được dự đoán nhờ quy nạp, hay nói cách khác
là "nghệ thuật" đề xuất những mệnh đề quy nạp.

2. CÁC BÀI TOÁN ÁP DỤNG


Bài toán 1. Ta sẽ chứng minh phát biểu sau của Euler dựa trên phép quy nạp đầy đủ (tức
xét tất cả các trường hợp có thể) như sau: Đối với đa diện đều tuỳ ý ta có D − C + M = 2,
trong đó D là số đỉnh, C là số cạnh, M là số mặt.

Lời giải. Vì số hình đa diện đều là hữu hạn, ta sẽ xét tất cả các trường hợp có thể như
sau:
Đa diện đều D (số đỉnh) C (số cạnh) M (số mặt)
Tứ diện 4 6 4
Lập phương 8 12 6
Bát diện 6 12 8
Thập nhị diện 20 30 12
Nhị thập diện 12 30 20

101
Với cả 5 trường hợp trên, ta đều có D − C + M = 2. Hoàn tất phép quy nạp.

Phương pháp quy nạp đầy đủ gần với phương pháp phân tích hơn, khi ta cần
phải chia trường hợp tổng quát thành hữu hạn trường hợp riêng và xét chúng một
cách riêng lẽ, vì thế có phạm vi áp dụng rất hạn chế trong toán học. Ta sẽ đến với bài
tập thứ 2, dùng phương pháp quy nạp tiêu chuẩn, để chứng minh một định lý trung
tâm của hình học tổ hợp.

Bài toán 2 (Định lí Helly). Cho n hình lồi (n ≥ 4) thoả mỗi 3 hình có ít nhất 1 điểm chung.
Chứng minh rằng tất cả n hình lồi đó đều có ít nhất 1 điểm chung.

Lời giải. Để chứng minh bằng quy nạp, ta thử xét n = 4. Khi đó, đặt 4 tập lồi là
M1 , M2 , M3 , M4 . Gọi Ai là giao của tất cả các cặp trừ Mi . Ta xét hai trường hợp:

TH1: 1 trong 4 điểm Ai nằm trong tam giác tạo bởi 3 điểm còn lại. Không mất tình
tổng quát giả sử điểm đó là A4 . Nhận xét: vì A1 , A2 , A3 đều thuộc M4 nên các
điếm nằm thuộc A1 A2 A3 cũng thuộc M4 .
TH2: A1 A2 A3 A4 tạo thành 1 tứ giác lồi. Gọi O là giao điểm A1 A2 và A3 A4 . Ta sẽ
chứng minh O thuộc tất cả các điểm trên. Ta có: A1 và A3 thuộc M2 và M4 nên
A1 A3 cũng thuộc M2 và M4 . Tương tự, ta có A2 A4 thuộc M1 và M3 nên giao
của chúng, O, thuộc cả 4 tập lồi đang xét.
Vậy bài toán đúng với n = 4. Ta giả sử đúng n = k. Gọi Mn0 là giao diểm của Mn
và Mn+1 . Hiển nhiên Mn0 cũng là 1 tập lồi. Với mọi i khác j ta có Mi , M j , Mn và
Mn+1 có ít nhất 1 điểm chung (vì ta đã xét trong trường hợp n = 4).

Từ đó, theo giả thiết quy nạp ta có điều phải chứng minh.

Ta sẽ đến với bài toán tiếp theo. Một lần nữa, ý tưởng quy nạp ở bài này khá đơn giản,
nhưng cần phải xét thật kĩ bước quy nạp.

Bài toán 3. Cho n điểm phân biệt trên mặt phẳng sao cho có ít nhất 3 điểm không thẳng
hàng. Chứng minh qua các điểm có thể vẽ được ít nhất n đoạn thẳng. (n >= 3)

Phân tích. Đây là một bài dễ, có thể chứng minh bằng nhiều cách nhưng ở đây ta tập trung
giải bằng phương pháp quy nạp, với ý tưởng quy nạp khá rõ.

102
Lời giải. Với n = 3, ta có 3 điểm không thẳng hàng tạo thành 1 tam giác, mệnh đề
hiển nhiên đúng. Giả sử mệnh đề đúng tới n = k, ta sẽ chứng minh mệnh đề cũng
đúng với n = k + 1.
Trong k + 1 điểm, giả sử tồn tại k điểm thẳng hàng thì ta xét k đường thẳng nối k
điểm đó với điểm còn lại, cộng thêm đường thẳng qua k điểm, ta có thể tạo ra k + 1
đường thẳng.
Giả sử ngược lại, nếu không tồn tại k điểm thẳng hàng thì tồn tại ít nhất 2 điểm A, B
thoả AB không đi qua bất kì điểm nào. Xét tập k điểm không chứa A ta có ít nhất k
đường thẳng, cộng với AB thì ta có k + 1 đường thẳng.
Vậy theo giả thiết quy nạp ta có điều phải chứng minh.

Những bài tập tiếp theo khá thử thách, thử thách ở cả việc đặt ra mệnh đề quy
nạp và chứng minh bước quy nạp, đòi hỏi bạn đọc nắm vững tư duy quy nạp cùng ý
tưởng rõ ràng về phương pháp quy nạp dự định sử dụng.

Bài toán 4 (Liên Xô 1989). Một con ruồi và một con nhện cùng nằm trên một trần nhà hình
vuông có cạnh 1 mét. Trong 1 giây con nhện có thể di chuyển từ vị trí ban đầu đến trung điểm
của đoạn thẳng từ nhện đến 1 trong 4 góc trần nhà. Trong thời gian trên, con ruồi không di
chuyển. Chứng minh rằng trong vòng 8 giây, con nhện có thể tiếp cận con ruồi trong khoảng
cách 1 cm.

Phân tích. Ở bài này ta có thể phát triển ý tưởng theo 2 hướng đi.

• Hướng đi thứ nhất: từ hình ảnh trần nhà hình vuông, cùng với sự độc lập của con nhện
khi di chuyển theo chiều ngang và chiều dọc của trần nhà gợi cho ta phương pháp sử
dụng tọa độ.
• Hướng đi thứ hai: ta có nhận xét sau: con nhện di chuyển đến vị trí một nửa quãng
đường từ nó đến góc trần nhà, sau 8 bước cần nhảy vào trong một hình hình tròn bán
kính 1cm, có tâm là con ruồi. Mục đích của ta trong 8 bước nhảy của nhện là tìm cách
đưa nhện nhảy vào (C ) ((C ) là đường tròn bán kính 1cm, tâm là ruồi). Thế nhưng, để
nhện nhảy vào C, 1 giây trước đó nhện phải nằm trong một hình tròn (C1 ) là phép vị
tự với đỉnh là 1 trong 4 đỉnh hình vuông, hệ số vị tự là 2 của hình tròn (C ). Nhận xét
trên gợi cho ta ý tưởng giải bài toán, đó là sử dụng phép vị tự.

Lời giải. Dưới đây là hai cách giải cho bài này.
Cách 1. Ta sẽ chứng minh một mệnh đề tổng quát hơn như sau. Đặt trần nhà hình
vuông vào hệ tọa độ, với gốc tọa độ là một góc trần nhà, trục Ox và Oy nằm trên cạnh
trần nhà. Ta đặt chiều dài đơn vị là 1m.

103
(0, 1) (1, 1)

y+1
2
( x, y)

y
2

(0, 0) x x+1 (1, 0)


2 2

Gọi ( x; y) là tọa độ vị trí ban đầu của con nhện. Xét tập hợp các điểm:
 
x+i y+i
Ak = k
; k | i, j ∈ Z, 0 ≤ i, j < 2k
2 2

Tập hợp trên cho ta một mạng lưới các điểm hình vuông, mỗi cạnh có chiều dài 2−k .
A0 chỉ gồm 1 điểm ( x; y); A1 gồm 4 điểm:
x y x + 1 y x y + 1 x + 1 y + 1
; , ; , ; , ;
2 2 2 2 2 2 2 2

Khi nhảy từ ( x; y), nhện có thể chọn vị trí theo tung độ và hoành độ riêng biệt. Ta sẽ
chứng minh bằng quy nạp theo k, trong lần nhảy thứ k, nhện có thể đến được bất kì
vị trí nào trong Ak .
Với k = 0, mệnh đề đúng. Giả sử  nhện có thểnhảy đến bất kì vị trí nào trong Ak , sau
x+i y+j
k lượt. Lấy vị trí M bất kì M = ; thuộc Ak+1 . Ta sẽ chỉ ra vị trí N ( x1 ; y1 )
2k +1 2k +1
thuộc Ak trước đó, để từ đó nhện có thể nhảy đến M.
x+i x+i y+j
Lấy x1 = k nếu i < 2k và x1 = k − 1 nếu i ≥ 2k . Tương tự lấy y1 = k nếu
2 2 2
y + j
j < 2k và y1 = k − 1 nếu j ≥ 2k . Điểm N ( x1 ; y1 ) với tọa độ trên sẽ thuộc tập hợp
2
Ak và nhện có thể nhảy từ N đến M.
Ta lại có một nhận xét như sau: với mọi điểm P trên trần nhà, tồn tại một điểm trong

Ak sao cho khoảng cách từ điểm đó đến P không lớn hơn 2.2−k . Với k = 8, ta có
√ −8 1
2.2 < , hoàn tất việc chứng minh.
100

104
A M B

Qi

O N
W

Q i −1

D P C

Cách 2. Đặt trần nhà thành hình vuông ABCD có tâm O, cạnh có chiều dài 100 đơn
vị. Gọi điểm xác định vị trí con ruồi là R, vị trí con nhện lúc đầu là N. Gọi dãy
Q8 , Q7 , Q6 , Q5 , Q4 , Q3 , Q2 , Q1 , Q0 là các điểm được xác định bởi phép vị tự của đỉnh
trước đó, với đỉnh là 1 trong 4 đỉnh ABCD, hệ số vị tự là 2, và Q8 = R. Gọi dãy
N0 , N1 , N2 , N3 , N4 , N5 , N6 , N7 , N8 là vị trí con nhện ở giây thứ 0, 1, 2. . . , 8.
Để N8 thuộc ( Q8 ; 1), ta cần có N7 thuộc ( Q7 ; 2) và tương tự. Một phép quy nạp đơn
giản ta suy ra điều cần chứng minh tương đương với N0 thuộc ( Q0 ; 256). Ở đây, ta
hoàn toàn có thể chỉ ra thuật toán để ( Q0 ; 256) bao tất cả các điểm nằm trong hình
vuông ABCD.
Gọi M, N, P, W lần lượt là trung điểm AB, BC, CD, DA. Gọi S A là hình vuông AMOW,
SB là hình vuông MONB, SC là hình vuông ONCP, SD là hình vuông WOPD. Ta sẽ
tìm đỉnh ABCD thích hợp để các điểm Qi luôn nằm trong hình vuông. Thật vậy, giả sử
Qi thuộc ABCD, ta sẽ chỉ ra tồn tại phép vị tự để Qi−1 cũng thuộc ABCD. Không mất
tính tổng quát, giả sử Qi thuộc S A , H2A ( Qi ; 28−i ) 7→ ( Qi−1 ; 28−i+1 ), H2A ( Qi ) 7→ Qi−1 ,
H2A (S A ) 7→ ABCD. Vì Qi ∈ √S A =⇒ Qi−1 ∈ ABCD. Hoàn tất mệnh đề quy nạp, ta có
Q0 ∈ ABCD, mà 256 > 10 2 là khoảng cách lớn nhất của hai điểm bất kì nằm trong
hình vuông, suy ra ABCD ∈ ( Q0 ; 256).
Ta có N thuộc hình tròn ( Q0 ; 256). Đặt Ki = A nếu Qi ∈ S A . Làm phép đặt tương tự
với SB , SC , SD . Ta xét:

1 1
HKi ( N0 ) 7→ N1 , HKi ( Q0 ; 256) 7→ ( Q1 ; 128)
2 2

Vì N ∈ ( Q0 ; 256) suy ra N1 ∈ ( Q1 ; 128). Làm tương tự, ta có N8 ∈ ( Q0 ; 1). Vị trí ở


giây thứ 8 của nhện cách ruồi ít hơn 1 đơn vị. Vậy tồn tại cách di chuyển sao cho sau
8 bước nhảy khoảng cách giữa nhện và ruồi nhỏ hơn 1 cm. Ta có điều phải chứng
minh.

105
Bài toán 5 (IMO Shortlist 2006). Một tam giác đều có cạnh n được gọi là lỗ nếu trong tam
giác này có n tam giác đơn vị bị cắt ra. Một hình kim cương là một hình kim cương với các
góc 60-120. Chứng minh ta chỉ có thể phủ tam giác lỗ bằng các hình kim cương khi và chỉ khi
mỗi tam giác đều hướng lên có cạnh là k thì không có quá k lỗ, với 1 ≤ k ≤ n.

Phân tích. Đây là một bài toán khá hay sử dụng quy nạp. Tuy vậy, theo em chướng ngại vật
lớn nhất trong bài toán là việc hiểu được mô hình bài toán cùng hướng tiếp cận. Với những
tam giác đơn vị, ta chỉ ra những trường hợp không thoả khi một tam giác đều có cạnh là k có
nhiều hơn k lỗ. Việc đục nhiều lỗ của một tam giác sẽ tạo ra những tam giác không thể tiếp
cận, hay nói cách khác, không thể phủ được, từ đó suy ra mệnh đề quy nạp đúng.

Lời giải. Gọi các tam giác hướng lên là tam giác loại 1, các tam giác hướng xuống là
tam giác loại 2, hình kim cương được tạo bởi 2 tam giác đều ở 2 hàng liền nhau là
hình kim cương loại 1, hình kim cương được tạo bởi 2 tam giác kề nhau là hình kim
cương loại 2.

Tam giác loại 1 Tam giác loại 2 Kim cương loại 1 Kim cương loại 2
Với tam giác có n cạnh, ta có:

n2 + n n2 + n
tam giác loại 1 và tam giác loại 2.
2 2

Hiển nhiên, 1 hình kim cương 60 − 120 được tạo thành bằng việc ghép 1 tam giác mỗi
loại. Vì ta chỉ có thể bỏ đi các tam giác loại 1 nên ta phải bỏ đúng:

n2 + n n2 + n
− = n tam giác loại 1.
2 2

Đây là 1 kết quả khá thú vị, và là phần dễ của bài.


Ta có thể dễ dàng chỉ ra 1 trường hợp thoả yêu cầu để bài, ví dụ bỏ n tam giác loại 1 ở
đáy tầng cuối của tam giác ban đầu. Khi đó ta có thể phủ tam giác bằng những hình
kim cương loại 1 với mô hình khá rõ.
Bây giờ, ta sẽ chứng minh mệnh đề sau đúng:

Với mỗi tam giác đều có cạnh là k thuộc tam giác đều có cạnh là n, nếu không có quá
k lỗ thì các tam giác có k − 1 cạnh bên trong nó cũng không có quá k − 1 lỗ.

(Lưu ý: Lỗ được tạo ra khi ta cắt 1 tam giác đơn vị ra khỏi hình.)
Với k = n, xét 1 tam giác có cạnh là n − 1 và 1 hình thang có 2 đáy lần lượt là n − 1, n
và có số đo các góc là 120 − 60. Không mất tính tổng quát, cho hình thang nghiêng 1
góc 120. Khi đó, nếu ta ghép 2 hình lại với nhau ta sẽ được tam giác ban đầu.

106
Vì tam giác ban đầu, có n cạnh và tam giác có cạnh n − 1 cùng có n lỗ nên mọi điểm
trong hình thang đều được phủ bởi các hình kim cương.
Ta xét vị trí của tam giác đợn vị trên cùng và dưới cùng của hình thang. Khi đó, chỉ có
thể lấp hình thang bằng các hình kim cương loại 1. Tuy vậy, cách lấp này sẽ làm xuất
hiện 1 tam giác loại 1 có 2 cạnh tiếp xúc với 2 hình kim cương, mặt còn lại tiếp xúc
với đáy của hình thang, hay cũng chính là cạnh của tam giác ban đầu. Tam giác này
không thể ghép được với bất kì tam giác loại 2 nào. Suy ra mệnh đề đúng với k = n.
Giả sử mệnh đề đúng tới k = u. Ta cần chứng minh mệnh đề đúng với u − 1. Tới đây,
ta xét 3 trường hợp:

TH1: Tam giác có cạnh là u − 1 có 2 cạnh thuộc 2 cạnh của tam giác có độ dài bằng n.

Xét tam giác có cạnh là u nằm ở vị trí tương tự. Ta cũng chia tam giác cạnh là u
nay thành 1 tam giác đều và 1 hình thang như khi xét k = n. Trong hìn thang
đó, hiển nhiên tồn tại 1 tam giác loại 1 không thể nối với bất kì tam giác loại 2
nào trong hình tam giác đều có cạnh là u. Coi tam giác giác đó là “xấu”.Khi ta
mở rộng mô hình và xét những hình thang tiếp theo với độ dài các đáy tăng 1
thì tam giác xấu này không thể bị loại bỏ mà chỉ đổi chỗ, dũ xét với hình thang
loại 1 hay loại 2. Ta lặp lại bước lấy thêm tam giác với cạnh lớn hơn cho tới khi
ta xét xong tam giác có cạnh là n.
Vậy, trường hợp 1 không cho phép phủ tam giác bằng những hình thang.
TH2: Tam giác có cạnh là u − 1 có 1 cạnh thuộc cạnh của tam giác có độ dài bằng n.

107
Từ đáy tam giác có độ dại là u kẻ d1 song song với đáy hình thang. Khi đó d1
chia tam giác có độ dài n thành 1 tam giác đều và 1 hình thang. Mô hình bài
toán trở về trường hợp 1.
TH3: Tam giác có cạnh là u − 1 có 0 cạnh thuộc cạnh của tam giác có độ dài bằng n.

Kẻ tương tự trường hợp 2, một đường song song với đáy và một đường song
song với 1 trong 2 cạnh của tam giác. Mô hình bài toán tới đây cũng quay về
trường hợp 1.

Vậy ta có nhận xét rằng mệnh đề mà ta đưa ra là đúng với mọi 1 ≤ k ≤ n, từ đó suy
ra được điều phải chứng minh.

Bài toán 6 (IMO 2013). Một tập hợp gồm đúng 4027 điểm trên mặt phẳng được gọi là tập
Colombia nếu không có ba điểm nào trong các điểm đó thẳng hàng, đồng thời có 2013 điểm
được tô màu đỏ và 2014 điểm còn lại được tô màu xanh. Mặt phẳng được phân chia thành các
miền khi ta kẻ một số đường thẳng. Một cách kẻ một số đường thẳng được gọi là cách kẻ tốt
đối với tập Colombia cho trước nếu hai điều kiện sau được thoả mãn:

1. Không đường thẳng nào đi qua dù chỉ một điểm thuộc tập hợp đó;
2. Không miền nào chứa cả điểm màu đỏ và điểm màu xanh.

Tìm số k nhỏ nhất sao cho với tập Colombia tuỳ ý gồm đúng 4027 điểm, tồn tại một cách kẻ k
đường thẳng là cách kẻ tốt.

Phân tích. Dự đoán bước quy nạp: Từ điều kiện không có 3 điểm nào thẳng hàng cho ta ý
tưởng về việc xây dựng và xét cạnh đa giác bao. Gọi S là một tập không có 3 điểm nào thẳng
hàng được tô hai màu xanh và đỏ, k là số đường thẳng nhỏ nhất chia S thoả đề bài, K là tập
hợp các đường thẳng trên. Xét A và B là hai điểm trên cạnh đa giác bao, nếu A và B cùng

108
màu, nếu cả hai không cùng nằm trên phần diện tích cùng màu được chia bởi các đường thẳng
thuộc K, cần thêm 1 đường thẳng song song với AB và chia mặt phẳng thành 2 bờ, 1 bờ chứa
A và B, một bờ chứa các điểm thuộc S, nâng tổng số đường thẳng lên k + 1 là tối thiểu để
chia tập S ∪ { A; B} thoả mãn đề bài. Nếu A và B khác màu và chia bởi 1 đường thẳng thuộc
K, cần nhiều nhất 1 đường thẳng song song với AB và chia mặt phẳng thành 2 bờ, 1 bờ chứa
A và B, một bờ chứa các điểm thuộc S, nâng tổng số đường thẳng lên k + 1 là tối thiểu để
chia tập S ∪ { A; B}. Nếu A và B khác màu và cùng trong 1 vùng, thì A hoặc B khác màu
so với các điểm còn lại, và vì A và B là đỉnh của đa giác bao nên tồn tại đường thẳng chia A
hoặc B ra khỏi các điểm cùng nằm trong phần diện tích trên. Ở bước quy nạp trên không có
B
sự phân biệt về số lượng của các điểm màu xanh và màu đỏ. Gọi n là tổng số điểm tạo thành
tập theo đề. Ta dự đoán |K | tăng 1 đơn vị mỗi khi n tăng 2 đơn vị. Điều ta cần hiện nay là
một mệnh đề quy nạp đủ rộng để có thể thực hiện bước quy nạp.

Dự đoán mệnh đề quy nạp: Số 2013 và 2014 hoàn toàn không đóng một vai trò cụ thể nào,
ta nghĩ ngay đến việc quy nạp theo n điểm màu đỏ và n + 1 điểm màu xanh. Với n = 0, dễ
thấy k = 0. Với n = 1, vì 3 điểm không thẳng hàng tạo thành hình tam giác, ta chọn đường
trung trực ứng với đỉnh có màu đỏ suy ra k = 1. Với n = 2, phân tích một số trường hợp ta
dự đoán k = 2. Với n bất kì, kiểm tra trường hợp lí tưởng đa giác đều 2n + 1 đỉnh, hai đỉnh
kề nhau không đồng thời màu đỏ, tồn tại một đa giác n đỉnh đồng dạng với đa giác n đỉnh đỏ,
các cặp cạnh tương ứng song song với nhau và bao n điểm đỏ, không chứa điểm xanh nào.
Trong trường hợp trên có thể thấ k = n. Từ đây ta dự đoán k = n với mọi n và tìm cách quy
nạp theo n. Ta sẽ cố gắng chứng minh bằng quy nạp mệnh đề sau: Cần ít nhất k = n đường
thẳng để chia n điểm đỏ và n + 1 điểm xanh thành các vùng sao cho trong mỗi vùng không
cùng chứa 2 điểm khác màu. Thế nhưng, áp dụng ý tưởng quy nạp đã phân tích với 2 điểm
cùng màu đỏ không có kết quả, vì mệnh đề quy nạp của ta chưa bao gồm trường hợp trên. Để
ý các trường hợp cơ bản tổng số điểm s = 2 và s = 3. Với s = 2, 2 điểm cùng màu k = 0, hai
điểm khác màu k = 1, suy ra k = 1. Với s = 3, trường hợp 1 điểm khác màu k = 1, trường
hợp 3 điểm cùng màu k = 0, ta kết luận k = 1 với s = 2, 3, trong mọi cách tô màu khác nhau.
Mệnh đề quy nạp không cần phát biểu số lượng hai màu xanh đỏ mà tập trung vào tổng số
điểm thoả đề bài.

Lời giải. Ta phát biểu mệnh đề quy nạp như sau:


Cho n điểm điểm được
h n i tô màu xanh hoặc đỏ, trong đó không có 3 điểm nào thẳng
hàng. Khi đó, k = là số đường thẳng ít nhất có thể để chia mặt phẳng thành các
2
vùng sao cho các điểm khác màu không cùng nằm trong 1 vùng. Với n ≤ 2 khẳng

109
định trên là hiển nhiên. Giả sử n ≥ 3, xét một đường thẳng ` chứa 2 điểm A và B sao
cho tất cả những điểm màu còn lại cùng nằm về một phía của `. Ta có ` là đường
thẳng chứa một cạnh của đa giác bao của các điểm được đánh màu, và A, Bhlà iđỉnh
n
của đa giác bao. Loại A, B khỏi các điểm đang xét, theo giả thiết quy nạp, cần −1
2
đường thẳng chia các điểm còn lại thoả mãn đề bài. Sau đó, ta xét hai điểm A và B,
với các trường hợp như sau:

TH1: Nếu A và B có cùng màu, vẽ đường thẳng song song với ` và chia A, B khỏi các
điểm còn lại. Khi đó mệnh đề quy nạp được thoả mãn.
TH2: Nếu A và B có màu khác nhau, nhưng được chia bởi một đường thẳng, ta lại vẽ
một đường thẳng tương tự như trên, mệnh đề quy nạp được thoả mãn.
TH3: Nếu A và B có màu khác nhau và nằm vào cùng một phần diện tích, theo giả
thiết quy nạp, các điểm trong phần diện tích trên cùng màu với A hoặc B. Không
mất tính tổng quát, giả sử A có màu khác với tất cả các điểm còn lại. Khi đó, tồn
tại một đường thẳng tách A khỏi các điểm còn lại trong phần diện tích trên, vì
A là đỉnh của đa giác bao. Mệnh đề quy nạp được thoả mãn.

Áp dụng với tập Colombia có tổng số điểm là 4027 ta có k = 2013 là số đường thẳng
tối thiểu cần có. k = 2013 là đáp án của bài toán trên.

3. BÀI TẬP
Bài tập 1 (Austrian-Polish MO 2000). Cho 27 điểm phân biệt trong mặt phẳng, không có
3 điểm nào thẳng hàng với bốn điểm tạo thành một hình vuông, 23 điểm còn lại thuộc hình
1
vuông đó. Chứng minh tồn tại 3 điểm X, Y, Z phân biệt thoả [ XYZ ] ≤ .
48

Bài tập 2 (Moscow 1999). Cho đa giác lồi trên mặt phẳng thoả mỗi cạnh được tô màu phía
ngoài đa giác (tức chỉ tô một vài đoạn của cạnh, những đoạn còn lại thì không tô). Trong đa
giác ta vẽ một số đường chéo được tô màu như trên. Chứng minh một trong những đa giác
được tạo từ một phần của đa giác ban đầu cũng được tô màu các cạnh bên ngoài.

Bài tập 3. Cho 1 đa giác n cạnh với 200n đường chéo sao cho không có 3 đường chéo nào
đồng quy. Hỏi các đường chéo chia hình thành bao nhiêu phần?

Bài tập 4. Trong một đa giác lồi n đỉnh (n ≥ 403), có 200n đường chéo được vẽ. Chứng
minh một đường trong số đó giao với ít nhất 10000 đường còn lại.

Bài tập 5. Cho đa giác lồi có n đỉnh, n ≥ 4. Ta tách đa giác thành các tam giác có đỉnh là
đỉnh đa giác một cách bất kì sao cho không tồn tại 2 tam giác có cùng điểm chung bên trong.
Ta tô đen các tam giác có 2 cạnh là cạnh đa giác ban đầu, tô đỏ nếu tam giác chỉ có 1 cạnh là
cạnh đa giác và tô trắng nếu không có cạnh nào của tam giác là cạnh đa giác. Chứng minh số
tam giác đen nhiều hơn số tam giác trắng 2 tam giác.

110
Bài tập 6. Xét một đa giác lồi n đỉnh sao cho không có 3 đường chéo nào của nó đồng quy.
Hỏi các đường chéo trên chia đa giác thành bao nhiêu phần?

Bài tập 7 (Vietnam 2007). Cho đa giác 2007 đỉnh. Tìm k min thoả: nếu xét k đỉnh bất kì của
đa giác, luôn tồn tại 4 đỉnh tạo thành một tứ giác lồi có 3 cạnh là cạnh đa giác ban đầu.

TÀI LIỆU THAM KHẢO


1. Mathematical Induction: A Powerful and Elegant Method of Proof (Titu Andreescu, Vlad Crisan).
2. Tổ hợp và Quy nạp ( N.IA.VILENKIN Dịch giả: GS.TSKH. Hà Huy Khoái)
3. Hình học tổ hợp (Vũ Hữu Bình).
4. IMO shortlist các năm.

111
BÀI TOÁN CHIA KẸO EULER
Trần Nguyễn Nam Hưng
Lớp chuyên Toán khoá 2018-2021

GIỚI THIỆU. Trong nhiều bài toán tổ hợp, thường là các bài toán tổ hợp tính toán,
đếm, ngoài các phương pháp thông thường như áp dụng quy tắc cộng, quy tắc nhân,
nguyên lý bao hàm loại trừ,. . . , việc áp dụng “ Bài toán chia kẹo của Euler” cũng hết
sức cần thiết. Bài toán được đề xuất bởi nhà toán học người Thụy Sĩ Leonhard Euler
(1707 – 1783) – một trong những nhà toán học lừng lẫy nhất mọi thời đại và đã trở
thành một trong những công cụ không thể thiếu trong việc giải quyết các bài toán tổ
hợp ngày nay. Vậy trên thực tế bài toán này có nội dung như thế nào, và nó lợi hại
như thế nào trong các bài toán như vậy?

1. SƠ LƯỢC VỀ NỘI DUNG CƠ BẢN VÀ MỘT VÀI MỞ RỘNG


TIÊU BIỂU CỦA BÀI TOÁN CHIA KẸO EULER

1.1. Khái quát về bài toán chia kẹo của Euler


Bài toán chia kẹo của Euler về cơ bản có nội dung như sau

Bài toán 1. Có bao nhiêu cách chia n viên kẹo cho k đứa trẻ (n ≥ k) sao cho đứa trẻ nào cũng
có kẹo ?
1
Chứng minh. Đáp số cho bài toán này là Cnk− −1 .
Thật vậy, gọi k đứa trẻ là A1 , A2 , . . . , Ak . Ta xếp n viên kẹo trên một hàng ngang. Xét
một cách đặt k − 1 vách ngăn giữa các viên kẹo bất kì thỏa

• Những viên kẹo nằm bên trái vách ngăn thứ nhất là của A1 , nhưng viên kẹo
nằm bên phải vách ngăn cuối cùng là của Ak .
• Những viên kẹo nằm giữa vách ngăn thứ i và vách ngăn thứ i + 1 là của Ai+1

Một cách đặt như vậy tương ứng với duy nhất một cách chia kẹo cho k đứa trẻ cũng
như một cách chọn ra k − 1 số nguyên dương từ n − 1 số nguyên dương đầu tiên. Do
đó một cách chia kẹo cho k đứa trẻ cũng tương ứng với đúng một cách chọn ra k − 1
số nguyên dương từ n − 1 số nguyên dương đầu tiên, và như vậy số cách chia kẹo
1
cho k đứa trẻ này bằng số cách chọn nói trên và bằng Cnk−
−1 .

113
1.2. Một vài mở rộng và ứng dụng tiêu biểu của bài toán chia kẹo của Euler
Đầu tiên, ta hãy xét ví dụ sau.

Bài toán 2. Có bao nhiêu bộ các số tự nhiên có thứ tự ( x1 , x2 , . . . , xk ) sao cho

x1 + x2 + . . . + xk = n,

với k là số nguyên dương và n là số tự nhiên cho trước ?

Nếu để ý, có thể thấy bài toán chia kẹo của Euler nói trên chính là bài toán tính số
nghiệm nguyên dương của phương trình

x1 + x2 + . . . + xk = n, (với n ≥ k).

Với bài toán 1, ta không những có thể tính số nghiệm nguyên dương của phương
trình trên mà còn có thể tính số nghiệm tự nhiên của phương trình

x1 + x2 + . . . + xk = n, (với n tự nhiên bất kỳ).

Chứng minh. Thật vậy, nếu xi là số tự nhiên thì xi + 1 là số nguyên dương và ngược
lại. Do đó nếu xi là số tự nhiên với mọi 1 ≤ i ≤ k và

x1 + x2 + . . . + xk = n, (1)

thì xi + 1 nguyên dương với mọi 1 ≤ i ≤ k và

( x1 + 1) + ( x2 + 1) + . . . + ( xk + 1) = n + k, (2)

và ngược lại. Do đó một bộ nghiệm tự nhiên của (1) tương ứng với một và chỉ một bộ
nghiệm nguyên dương của (2), và như vậy (1) và (2) có số nghiệm như nhau. Theo bài
1
toán 1, số nghiệm đó là Cnk−
+k−1 . Bài toán 2 được giải quyết hoàn toàn.

Như vậy, từ bài toán 2, chúng ta có thể dễ dàng tính được số nghiệm tự nhiên của
phương trình

x1 + x2 + . . . + xk = n, (với k nguyên dương và n tự nhiên bất kỳ).

Điều này khá thường gặp trong các bài toán tổ hợp đếm. Từ đây, có thể dễ dàng giải
quyết bài toán sau.

Bài toán 3. Có bao nhiêu đa thức đơn khởi, bậc 2018 và có các hệ số đều là số tự nhiên sao
cho P(1) = 1285 ? (Đa thức đơn khởi, hay còn được gọi là đa thức monic, là đa thức có hệ số
bậc cao nhất bằng 1, trong trường hợp này đa thức P( x ) có hệ số bậc 2018 bằng 1)

114
Chứng minh. Giả sử P( x ) = x2018 + a2017 x2017 + . . . + a1 x + a0 , với a2017 , a2016 , . . . , a0
là các số tự nhiên. Thay x = 1 vào đa thức, ta có

P(1) = 1 + a2017 + a2016 + . . . + a0 = 1285,

suy ra
a0 + a1 + . . . + a2017 = 1284.
Như vậy, số đa thức P( x ) thỏa yêu cầu đề bài bằng số nghiệm tự nhiên của phương
2018−1 2017
trình trên, và bằng C1284 +2018−1 , hay C3301 . Bài toán được giải quyết hoàn toàn.

Tiếp theo, ta hãy xét bài toán sau

Bài toán 4. Để bảo vệ máy tính của mình khỏi sự tấn công của hacker, một lập trình viên
muốn thiết lập một mật khẩu cho máy tính của mình. Mật khẩu này bao gồm tất cả các chữ
cái trong bảng chữ cái tiếng Anh, mỗi chữ cái xuất hiện đúng 1 lần, hơn nữa 2 nguyên âm
không đứng cạnh nhau. Hỏi lập trình viên đó có bao nhiêu cách cài đặt mật khẩu ?

Việc cài đặt mật khẩu bao gồm 2 công việc : Chọn các vị trí đặt nguyên âm và sắp
xếp các nguyên âm đó, tương tự đối với các phụ âm. Ta đều biết trong bảng chữ cái
tiếng Anh có 5 nguyên âm (A, E, I, O, U) và 21 phụ âm. Một mật khẩu thỏa yêu cầu
đề bài khi và chỉ khi giữa 2 nguyên âm bất kì có ít nhất một phụ âm. Như vậy, có thể
thấy, bài toán này, tính riêng công việc chọn vị trí các nguyên âm (phụ âm) đòi hỏi
việc giữa 2 nguyên âm liên tiếp phải có ít nhất một phụ âm. Để giải quyết vấn đề này,
ta sẽ áp dụng “Bài toán chia kẹo của Euler” cho số phụ âm đứng giữa 2 nguyên âm
liên tiếp, rồi từ đó tiếp tục tính toán bằng các công cụ thông thường.

Chứng minh. Giả sử các ký tự trong mật khẩu của máy tính là A1 , A2 , . . . , A26 (theo
thứ tự từ trái sang phải). Để cài đặt mật khẩu thỏa yêu cầu đề bài, ta phải thực hiện 2
công việc:

• Công việc 1: Chọn các vị trí đặt nguyên âm (phụ âm).


• Công việc 2 : Sắp xếp các nguyên âm (phụ âm) ứng vào các vị trí đó.

Để thực hiện công việc 1, giả sử 5 nguyên âm được đặt tại Ai , A j , Ak , Al , Am (i < j <
k < l < m). Ta chèn thêm 2 nguyên âm A0 và A27 vào 2 đầu mật khẩu. Ta xét 4 trường
hợp sau.

• i = 1, m = 26.
Khi này giữa A0 và Ai , giữa Am và A27 không có phụ âm nào. Như vậy, 21 phụ
âm tập trung vào 4 vị trí :giữa Ai và A j , giữa A j và Ak , giữa Ak và Al , giữa Al
và Am . Theo đề, mỗi vị trí như vậy phải có ít nhất 1 phụ âm và có tổng cộng 21
phụ âm trong 4 vị trí đó.

115
Gọi x1 , x2 , x3 , x4 là số phụ âm trong mỗi vị trí đó thì xi là số nguyên dương với
1 ≤ i ≤ 4 và
x1 + x2 + x3 + x4 = 21.
3 .
Theo bài toán chia kẹo của Euler, số bộ ( x1 , x2 , x3 , x4 ) thỏa mãn là C20
• i = 1, m 6= 26 (tương tự với trường hợp i 6= 1, m = 26).
Khi này giữa A0 và Ai không có phụ âm nhưng giữa Am và A27 có ít nhất 1 phụ
âm. Tương tự, 21 phụ âm sẽ tập trung vào 5 vị trí, mỗi vị trí ứng với 2 nguyên
âm "kẹp” nó, và mỗi vị trí có ít nhất 1 phụ âm. Gọi x1 , x2 , x3 , x4 , x5 là số phụ âm
4 .
trong mỗi vị trí đó, tương tự thì ta có số bộ ( x1 , x2 , x3 , x4 , x5 ) thỏa mãn là C20
• i 6= 1, m 6= 26
Tương tự, do 21 phụ âm tập trung tại 6 vị trí, mỗi vị trí ứng với 2 nguyên âm
5 .
“kẹp” nó, và mỗi vị trí có ít nhất 1 phụ âm nên số cách chọn là C20

3 + 2 C4 + C5 .
Như vậy, tổng số cách chọn vị trí các phụ âm là C20 20 20
Tiếp theo, với công việc 2, khi đã thực hiện công việc 1, ta chỉ cần có được một hoán
vị của các phụ âm và một hoán vị của các nguyên âm tương ứng. Có 21! hoán vị có
thể cho các phụ âm và 5! hoán vị cho các nguyên âm, nên số cách thực hiện công việc
này là 21! × 5!.
Do hai công việc trên được thực hiện liên tục nên số cách để cài mật khẩu là

3 4 5
21! × 5! × (C20 + 2 C20 + C20 ).

Bài toán được giải quyết.

Có thể thấy, với “Bài toán chia kẹo của Euler”, chúng ta có thể dễ dàng tính được
số các chỉnh hợp, tổ hợp thỏa mãn các điều kiện cho sẵn. Trong các ví dụ trên, các
phần tử trong các chỉnh hợp và tổ hợp nói trên đôi một khác nhau. Vậy khi có 2 trong
số các phần tử đó giống nhau thì liệu rằng “ Bài toán chia kẹo của Euler” có còn là
một công cụ hữu dụng ? Hãy cùng đến với ví dụ đặc biệt sau.

Bài toán 5. Trong một cái hộp có 7 loại kẹo, mỗi loại kẹo có nhiều hơn 40 viên kẹo. Hỏi có bao
nhiêu cách lấy ra 40 viên kẹo từ hộp ? (hai cách lấy được gọi là khác nhau nếu tồn tại một loại
kẹo nào đó mà số kẹo thuộc loại đó được lấy ra trong hai cách là khác nhau).

Chứng minh. Gọi x1 , x2 , . . . , x7 là số kẹo mỗi loại được lấy ra từ hộp trong mỗi cách
lấy. Theo đề bài, mỗi bộ 7 số tự nhiên ( x1 , x2 , . . . , x7 ) thỏa mãn điều kiện

x1 + x2 + . . . + x7 = 40

tương ứng với một cách lấy phù hợp và ngược lại. Hơn nữa, do xi ≤ 40 mà trong có
nhiều hơn 40 viên kẹo mỗi loại nên không có cách lấy nào trong số các cách lấy nói
trên “phạm quy” ( tức là không đủ kẹo để lấy). Do đó, các cách lấy này đều thỏa mãn

116
7−1 6
và số cách lấy, theo bài toán 1, bằng C40 +7−1 , hay C46 . Bài toán được giải quyết hoàn
toàn.

Có thể thấy, bài toán chia kẹo của Euler những tưởng chỉ đơn thuần là việc chia
kẹo cho trẻ con hóa ra lại có nhiều ứng dụng trong các bài toán đến như vậy. Từ việc
thiết lập mật mã, thiết lập đa thức cho đến việc tính số nghiệm tự nhiên của một
phương trình dạng tổng, thậm chí là việc thành lập các tổ hợp lặp. Trong phần tiếp
theo, chúng ta sẽ cùng đến với những “biến thể” của “Bài toán chia kẹo của Euler”, từ
đó giúp các bạn hiểu rõ các vận dụng phương pháp này vào việc giải toán.

2. MỘT SỐ BIẾN THỂ CỦA BÀI TOÁN CHIA KẸO EULER

2.1. Thêm, bớt để tạo ra các đại lượng mới


Trong phần này, chúng ta sẽ cùng đến với một ví dụ khá thú vị sau.

Bài toán 6. Một trường học có cách phát thưởng cho học sinh khá, giỏi vào cuối năm học khá
đặc biệt. Theo đó, trường sẽ phát cho mỗi lớp 200 quyển vở và yêu cầu lớp phải phát thưởng
cho 10 học sinh có thứ hạng cao nhất lớp theo quy tắc : Bạn học sinh đứng hạng i trong lớp
phải nhận được ít nhất 15˘i quyển vở. Lớp 9A của trường này có đúng 1 bạn đứng hạng i, với
1 ≤ i ≤ 10. Hỏi cô giáo chủ nhiệm của lớp có bao nhiêu cách chia 200 quyển vở nói trên cho
10 bạn này ?

Chứng minh. Từ đề bài, ta suy ra bạn đứng hạng 1 phải được ít nhất 14 quyển vở,
bạn đứng hạng 2 phải được ít nhất 13 quyển vở,...,bạn đứng hạng 10 phải được ít nhất
5 quyển vở. Gọi xi là số quyển vở mà bạn đứng hạng i nhận được, với q ≤ i ≤ 10. Khi
đó ta có
x1 + x2 + . . . + x10 = 200,
suy ra

( x1 − (15 − 1)) + ( x2 − (15 − 2)) + . . . + ( x10 − (15 − 10)) = 200 − (5 + 6 + . . . + 14) = 105.

Hơn nữa, các số xi − (15 − i ) trong mỗi cách chia của cô giáo đều phải là số tự nhiên
nên số cách chia chính là số nghiệm tự nhiên của phương trình trên. Do đó, số cách
10−1 9
chia mà cô giáo có thể thực hiện chính là C105 +10−1 , hay C114 .

Với bài toán này, nếu chỉ giải theo phương pháp thông thường, rõ ràng việc vận
dụng “Bài toán chia kẹo của Euler” vào bài toán là điều gần như không thể. Với việc
thêm, bớt tinh tế như bài toán trên (hay nói một cách khác là “cho trước” các em học
sinh lượng vở mà nhà trường yêu cầu cô giáo phải thưởng cho các em rồi sau đó chia
vở cho các em một cách tùy ý), có thể thấy bài toán trên đã được đưa về “Bài toán chia
kẹo của Euler” dạng bình thường. Phương pháp này cũng được sử dụng rất nhiều
trong các bài toán, mà điển hình là bài 5 trong đề thi VMO 2012 (xem trong phần bài
tập tự luyện).

117
2.2. Sử dụng bài toán chia kẹo của Euler nhiều lần trong cùng một bài toán
Trong các ví dụ trên, “Bài toán chia kẹo của Euler” chỉ được sử dụng một lần.
Trong phần này, chúng ta sẽ được “chiêm ngưỡng” một hướng áp dụng mới của “Bài
toán chia kẹo của Euler” vào giải toán, đó là dùng chúng nhiều lần trong cùng một
bài.

Bài toán 7. Nhằm thỏa mãn niềm mong ước được gặp thần tượng của nhiều fan K-pop (âm
nhạc pop Hàn Quốc) tại Việt Nam, một công ty tổ chức sự kiện đã quyết định tổ chức đêm
nhạc K-Pop Connection. Đêm nhạc có sự tham gia của 3 nhóm nhạc đình đám hiện nay là
Momoland, Blackpink và BTS. Trong đêm nhạc này, Momoland sẽ trình diễn tổng cộng 6 tiết
mục, Blackpink sẽ trình diễn tổng cộng 8 tiết mục và BTS sẽ trình diễn tổng cộng 12 tiết mục.
Một lượt diễn của một nhóm nhạc được coi là một chuỗi các tiết mục liên tiếp (có thể có đúng
1 tiết mục) của 1 nhóm nhạc nào đó mà tiết mục ngay trước tiết mục đầu tiên của chuỗi đó và
tiết mục ngay sau tiết mục cuối cùng của chuỗi đó không phải do nhóm nhạc đó biểu diễn.
Theo đó, trong đêm nhạc này, mỗi nhóm nhạc sẽ có 2 lượt diễn. Theo thứ tự, Blackpink sẽ thực
hiện lượt diễn của mình trước, sau đó đến Momoland và cuối cùng đến BTS trong cả 2 lượt.
Hỏi có bao nhiêu cách sắp xếp các buổi biểu diễn của các nhóm nhạc nói trên sao cho trong cả
hai lượt diễn, số tiết mục Blackpink biểu diễn không ít hơn số tiết mục Momoland biểu diễn
và số tiết mục mà BTS biểu diễn không ít hơn số tiết mục Blackpink biểu diễn?
(Hai cách sắp xếp được coi là khác nhau nếu như tồn tại một tiết mục thứ j của buổi diễn mà
trong cách sắp xếp này với cách sắp xếp kia được biểu diễn bởi 2 nhóm nhạc khác nhau)

Chứng minh. Gọi A1 , A2 là số tiết mục Momoland được biểu diễn trong 2 lượt diễn;
B1 , B2 là số tiết mục Blackpink được biểu diễn trong 2 lượt diễn; C1 , C2 là số tiết mục
BTS được biểu diễn trong 2 lượt diễn. Vì thứ tự thực hiện lượt diễn của mình của các
nhóm nhạc đều đã được định sẵn nên 2 cách sắp xếp được coi là khác nhau khi và chỉ
khi tồn tại 1 trong 6 số A1 , A2 , B1 , B2 , C1 , C2 khác nhau trong cả 2 cách.
Theo đề bài, ta có

A1 + A2 = 6, B1 + B2 = 8, C1 + C2 = 12,


C1 ≥ B1 ≥ A1 , C2 ≥ B2 ≥ A2 .
Số cách sắp xếp chính là số bộ có thứ tự ( A1 , A2 , B1 , B2 , C1 , C2 ) thỏa các yêu cầu trên.
Để thu được bộ sắp này, ta sẽ lần lượt chọn A1 , A2 , sau đó chọn B1 , B2 và cuối cùng
chọn C1 , C2 .

• Chọn A1 , A2 . Theo đề bài thì A1 , A2 nguyên dương và

A1 + A2 = 6

nên theo số cách chọn là C62−


−1 1
1 hay C5 .

118
• Chọn B1 , B2 . Theo đề bài ta có B1 ≥ A1 và B2 ≥ A2 nên B1 − A1 , B2 − A2 là các
số tự nhiên và

( B1 − A1 ) + ( B2 − A2 ) = ( B1 + B2 ) − ( A1 + A2 ) = 8 − 6 = 2.

Vì đã chọn A1 , A2 nên số cách chọn B1 , B2 cũng chính là số cách chọn B1 −


A1 , B2 − A2 thỏa yêu cầu trên, do đó số cách chọn là C22+ −1 1
2−1 hay C3 .
• Chọn C1 , C2 . Tương tư thì C1 − B1 , C2 − B2 là các số tự nhiên và

(C1 − B1 ) + (C2 − B2 ) = (C1 + C2 ) − ( B1 + B2 ) = 12 − 8 = 4,

nên số cách chọn là C42+


−1 1
2−1 hay C5 .

Vì 3 công việc này được thực hiện một cách liên tiếp nên số cách thực hiện việc sắp
xếp là C51 × C31 × C51 = 75.

2.3. Áp dụng “Bài toán chia kẹo của Euler” để tính số nghiệm nguyên không âm của
bất phương trình dạng x1 + x2 + . . . + xn ≤ k (với n là số nguyên dương và k là số
tự nhiên)
Trong các bài toán trước đó, chúng ta đã được biết đến cách tính số nghiệm nguyên
dương, nghiệm nguyên không âm của phương trình có dạng x1 + x2 + . . . + xn = k
thông qua “Bài toán chia kẹo của Euler”. Vậy còn với bất phương trình dạng

x1 + x2 + . . . + xn ≤ k,

để tính được số nghiệm tự nhiên (hoặc là số nghiệm nguyên dương) của chúng, ta
cần phải làm như thế nào ? Hãy cùng đến với ví dụ sau.

Bài toán 8. Bốn anh em An, Bình, Cường, Danh rất thích ăn kẹo. Nhân dịp Tết, mẹ của các
cậu được thưởng rất nhiều quà, trong đó có một phần quà là một bịch kẹo bao gồm 125 viên
kẹo. Vì quá thích ăn kẹo nên bốn anh em đã xin mẹ thêm một vài bịch kẹo khác giống như vậy
nữa, nhưng mẹ nhất quyết không cho vì sợ ăn nhiều kẹo sẽ bị sâu răng, do đó mẹ chỉ cho các
cậu ăn số kẹo trong bịch kia thôi. Thế là họ ngay lập tức chia nhau số kẹo nói trên. Hỏi họ có
tổng cộng bao nhiêu cách chia kẹo ? (cho rằng 125 viên kẹo là giống hệt nhau, và anh em họ có
quyền không ăn một số viên kẹo trong số 125 viên kẹo ban đầu, đồng thời cũng có thể xảy ra
trường hợp có một người không được chia viên kẹo nào).

Chứng minh. Gọi x1 , x2 , x3 , x4 là số viên kẹo mà mỗi người nhận được. Theo đề bài,
ta có x1 , x2 , x3 , x4 là các số tự nhiên và

x1 + x2 + x3 + x4 ≤ 125.

119
Đặt x5 là số kẹo còn lại trong bịch sau khi họ đã chia kẹo xong, ta có x5 là số tự nhiên

x1 + x2 + x3 + x4 + x5 = 125.
Do đó, số cách chia kẹo chính là số nghiệm tự nhiên của phương trình trên (vì mỗi
bộ nghiệm ứng với một cách chia và ngược lại). Theo bài toán 1, số nghiệm đó là
5−1 4
C125 +5−1 , hay C129 . Bài toán được giải quyết hoàn toàn.

Có thể thấy, bài toán này đưa về việc tìm số nghiệm tự nhiên của bất phương
trình
x1 + x2 + . . . + xn ≤ k,
trong trường hợp cụ thể là n = 4 và k = 125. Trong bài toán này, với việc thêm biến
xn+1 có giá trị bằng k − ( x1 + x2 + . . . + xn ), ta đã đưa được bất phương trình đã cho
về phương trình nghiệm tự nhiên

x1 + x2 + . . . + xn + xn+1 = k,

và biến bài toán tìm số nghiệm tự nhiên của bất phương trình nói trên thành “Bài
toán chia kẹo của Euler” dạng bình thường. Việc đặt x5 là số kẹo còn lại trong bịch
sau khi họ đã chia kẹo xong trong bài toán trên chính là một cách giải thích “dân dã”
cho kết quả của bài toán tìm số nghiệm tự nhiên của bất phương trình dạng này.
Qua các ví dụ trên đây, ta có thể nhận ra vô vàn những ứng dụng thú vị của “Bài
toán chia kẹo của Euler” trong việc giải toán. Để củng cố thêm kiến thức về công cụ
hữu dụng này, xin mời các bạn hãy cùng đến với phần bài tập tự luyện dưới đây.

3. MỘT SỐ BÀI TẬP TỰ LUYỆN


Bài tập 1. Lớp 10 chuyên Toán của một trường THPT được nhà trường thưởng 100 quyển
vở do có thành tích tốt trong các kỳ thi học sinh giỏi. Nhằm khích lệ tinh thần học tập của
lớp, cô giáo đã quyết định thưởng các quyển vở này cho các bạn có thành tích trong các kỳ thi
học sinh giỏi. Theo đó, lớp có 13 bạn có thành tích trong các kỳ thi học sinh giỏi, và bạn được
thưởng ít tập nhất được thưởng n quyển tập (với n là số nguyên dương). Hỏi cô giáo có bao
nhiêu cách thưởng tập cho các bạn học sinh sao cho

a) n có giá trị nhỏ nhất.


b) n có giá trị lớn nhất.
c) n = k với k là số nguyên dương cho trước bất kỳ.
d) Giải lại câu a) và b), khi thay 100 bởi x, thay 13 bởi y (với x ≥ y).

Bài tập 2 (P140, Tạp chí Pi, số 1 năm 2018). Có 6 đội bóng đá tham gia một giải thi đấu
bóng đá học sinh của một trường Trung học phổ thông chuyên. Biết rằng giải đấu kéo dài 45
ngày và được tổ chức theo thể thức thi đấu vòng tròn (nghĩa là, hai đội bất kì đều thi đấu với
nhau đúng một trận). Hỏi Ban tổ chức giải có tất cả bao nhiêu phương án sắp xếp lịch thi đấu
sao cho các điều kiện sau được đồng thời thỏa mãn

120
i. Ba trận đấu đầu tiên có sự góp mặt của cả 6 đội và diễn ra trong ba ngày liên tiếp, mỗi
ngày một trận,
ii. Kể từ trận đấu thứ ba, giữa hai trận đấu liên tiếp có ít nhất 2 ngày không có trận đấu
nào.
(Hai phương án sắp xếp được coi là khác nhau nếu tồn tại trận đấu thứ k, với k ≥ 1 sao cho ở
trận đấu đó, hai đội gặp nhau trong phương án này khác hai đội gặp nhau trong phương án
kia hoặc tồn tại k, với k ≥ 3 mà thời gian giữa trận đó và trận thứ k + 1 ở phương án này
khác phương án kia).

Bài tập 3 (Chọn Đội tuyển Quảng Ngãi 2017 – 2018). Trên một đường thẳng có 20 điểm
P1 , P2 , . . . , P20 được sắp xếp theo thứ tự đó, mỗi điểm sẽ được tô bởi một trong hai màu xanh
hoặc đỏ. Hỏi có bao nhiêu cách tô màu để sao cho nếu số các điểm liền kề được tô màu giống
nhau thì luôn là một số lẻ ?

Bài tập 4. Hỏi trong tam giác ABC có tổng cộng bao nhiêu điểm P sao cho từ điểm P ta có
thể kẻ 27 tia gốc P, đôi một phân biệt chia tam giác ABC thành 27 tam giác nhỏ có diện tích
bằng nhau ?

Bài tập 5 (Vietnam 2012). Cho một nhóm gồm 5 cô gái, kí hiệu là G1, G2, G3, G4, G5 và
12 chàng trai. Có 17 chiếc ghế được xếp thành một hàng ngang. Người ta xếp nhóm người đã
cho ngồi vào các chiếc ghế đó sao cho các điều kiện sau được đồng thời thỏa mãn
i. Mỗi người ngồi đúng 1 ghế.
ii. Các cô gái xếp theo đúng thứ tự G1, G2, G3, G4, G5 từ trái qua phải.
iii. Giữa G1, G2 có ít nhất 3 chàng trai.
iv. Giữa G4, G5 có ít nhất 1 chàng trai và nhiều nhất là 4 chàng trai.
Hỏi có tất cả bao nhiêu cách xếp như vậy ?
(Hai cách xếp được coi là khác nhau nếu tồn tại một chiếc ghế mà người ngồi ở chiếc ghế đó
trong hai cách xếp là khác nhau).

Bài tập 6 (Trường Đông Toán học 2013). Cho n ≥ 2 là một số nguyên dương. Xét tập hợp
các đường đi ngắn nhất trên lưới nguyên từ điểm A(0; 0) đến điểm B(n; n). Một đường đi
như thế sẽ tương ứng với dãy gồm n lệnh T (lên trên) và n lệnh P (sang phải). Trong dãy đó,
một cặp lệnh ( T, P) kề nhau được gọi là một bước chuyển (lưu ý, cặp ( P, T ) không được gọi
là bước chuyển). Ví dụ dãy PTTPTPPT có 2 bước chuyển. Tìm số các đường đi ngắn nhất từ
A đến B có đúng
a) 1 bước chuyển.
b) 2 bước chuyển.

Bài tập 7 (Chọn đội tuyển Sóc Trăng 2017 – 2018). Cho tập hợp A = {1, 2, . . . , 2049}
gồm 2049 số nguyên dương đầu tiên. Hỏi có thể chọn được tất cả bao nhiêu tập con B =
{ a1 , a2 , . . . , a9 } là tập con gồm 9 phần tử của A thỏa điều kiện | ai − a j | ≥ 5, với mọi
1≤i< j≤9?

121
Bài tập 8. Một tập hợp các số nguyên được gọi là tập nghỉ nếu như giữa phần tử nhỏ thứ i
và phần tử nhỏ thứ i + 1 của tập đó có ít nhất i2 số nguyên khác. Tập nghỉ bao gồm các số
nguyên dương thì được gọi là tập nghiêm. Hỏi có bao nhiêu tập nghiêm bao gồm 26 số nguyên
dương và các phần tử trong tập hợp này đều không lớn hơn 1984 ?

Bài tập 9. Một số nguyên dương biểu diễn được dưới dạng a0 a1 . . . an với a0 6= 0 trong hệ
thập phân được gọi là lên dốc nếu như a0 ≤ a1 ≤ . . . ≤ an , và được gọi là xuống dốc nếu
a0 ≥ a1 ≥ . . . ≥ a n .

a) Hỏi có bao nhiêu số tự nhiên có 5 chữ số thỏa mãn điều kiện số đó hoặc là số lên dốc
hoặc là số xuống dốc ?
b) Với mỗi số tự nhiên n, đặt f (n) là số số tự nhiên lên dốc có n chữ số và g(n) là số số tự
nhiên xuống dốc có n chữ số. Hãy so sánh f(n) và g(n).

Bài tập 10. Một dãy hữu hạn các số nguyên dương u1 , u2 , . . . , un được gọi là dãy đòn bẩy
nếu như dãy đó thỏa mãn đồng thời các điều kiện sau

i. n chia hết cho 3.


ii. u1 = 1, un = 100.
iii. u3k−2 < u3k−1 < u3k , với mọi 1 ≤ k ≤ n3 .

Tìm số nguyên dương n lớn nhất sao cho tồn tại một dãy đòn bẩy bao gồm n số nguyên dương
mà n n n
3 −1 3 −1 3 −1

∑ u3k+1 = 266, ∑ u3k+2 = 355, ∑ u3k+3 = 414.


i =0 i =0 i =0

Tính xem với n lớn nhất đó, có bao nhiêu dãy đòn bẩy bao gồm n số hạng.

Bài tập 11 (Vietnam 2014). Cho đa giác đều có 103 cạnh. Tô màu đỏ 79 đỉnh của đa giác và
tô màu xanh các đỉnh còn lại. Gọi A là số cặp đỉnh đỏ kề nhau và B là số cặp đỉnh xanh kề
nhau.

a) Tìm tất cả các giá trị có thể nhận được của cặp ( A, B).
b) Xác định số cách tô màu các đỉnh của đa giác để B = 14. Biết rằng hai cách tô màu
được xem là như nhau nếu chúng có thể nhận được nhau từ một phép quay quanh tâm
của đường tròn ngoại tiếp đa giác.

Bài tập 12. Tại một văn phòng phẩm nọ, người ta bán rất nhiều loại mặt hàng, trong đó có 7
loại mặt hàng quan trọng là bút bi, bút chì, tẩy (gôm), thước, vở 100 trang, vở 200 trang và
compass. Bạn An đã đến văn phòng phẩm này 3 lần để mua 7 loại mặt hàng nói trên, mỗi loại
1 cái. Khá là thú vị là kể từ lần thứ hai An đến trở đi, cậu ta đều phát hiện ra rằng giá của các
mặt hàng đều tăng so với lần trước cậu ta đến, hơn nữa cậu cũng phát hiện ra rằng giá của 7
loại mặt hàng nói trên trong cả 3 lần đến đều được tính chẵn theo đơn vị ngàn đồng. Bình biết
các thông tin trên và cũng biết rằng trong cả 3 lần nói trên, An lần lượt phải trả 31, 37 và 46
ngàn đồng cho các loại mặt hàng (do tiền An dùng để mua hàng thực tế là tiền An vay mượn

122
từ Bình). Hỏi xác suất để Bình đoán đúng được giá của cả 7 mặt hàng trong cả 3 lần An đến
văn phòng phẩm là bao nhiêu?

Bài tập 13. Trong mặt phẳng tọa độ, một con nhện đang ở gốc tọa độ và cho điểm
A(2015; 2017). Trong mỗi bước di chuyển, con nhện chỉ có thể di chuyển sang phải hoặc di
chuyển lên trên, hơn nữa nếu trong bước này nó đã di chuyển sang phải thì trong bước sau nó
phải di chuyển lên trên và ngược lại. Ngoài ra, con nhện cũng chỉ có thể di chuyển đến các
điểm nguyên trên mặt phẳng tọa độ. Hỏi con nhện có bao nhiêu cách di chuyển để sau đúng
45 bước di chuyển lên trên và 44 bước di chuyển sang phải thì nó đến được điểm C thỏa mãn
−→
điều kiện CA có tọa độ (m, n), với m, n là các số nguyên dương ?

TÀI LIỆU THAM KHẢO


1. Art of Problem Solving: http://artofproblemsolving.com

123
LỜI GIẢI ĐỀ THI CHỌN ĐỘI DỰ TUYỂN 2017
Ban biên tập

1. ĐỀ BÀI
Bài 1. Cho x, y, z là các số thực dương thoả mãn x + y + z = 1. Chứng minh rằng:

x4 y4 z4 1
+ + ≥ .
x 3 + y2 + z2 y3 + z2 + x 2 z3 + x 2 + y2 7

Bài 2. Tìm tất cả các hàm số f : N ∗ → N ∗ thoả mãn đồng thời các điều kiện:

i. f (mn) = f (m) f (n) ∀m, n ∈ N∗ .


ii. f (m) + f (n) chia hết cho m + n ∀m, n ∈ N∗ .
iii. f (2017) = 20173 .

Bài 3. Cho đường tròn (O) và dây cung AB cố định. C là một điểm thay đổi trên cung
lớn AB sao cho tam giác ABC nhọn. Gọi I, Ia , Ib lần lượt là tâm đường tròn nội tiếp,
tâm đường tròn bàng tiếp góc ∠ BAC và ∠ ABC của tam giác ABC.

1. Gọi M đối xứng với I qua O. Chứng minh rằng tam giác MIa Ib cân.
2. Gọi H, K lần lượt là hình chiếu của Ia , Ib trên OI. Đường thẳng qua H vuông góc
với BIa và đường thẳng qua K vuông góc với AIb cắt nhau tại P. Chứng minh
rằng P thuộc một đường cố định khi C thay đổi.

Bài 4. Cho S là tập hợp khác rỗng và A1 , A2 , ..., Am (m ≥ 2) là m tập con của S. Gọi T
là tập hợp gồm tất cả các tập hợp Ai 4 A j (1 ≤ i, j ≤ m). Chứng minh rằng | T | ≥ m.

2. LỜI GIẢI
Bài toán 1. Cho x, y, z là các số thực dương thoả mãn x + y + z = 1. Chứng minh rằng:

x4 y4 z4 1
+ + ≥
x 3 + y2 + z2 y3 + z2 + x 2 z3 + x 2 + y2 7

Lời giải. Trong các lời giải, ta quy ước ∑ f ( a, b, c) = f ( a, b, c) + f (b, c, a) + f (c, a, b).

125
Cách 1. Theo bất đẳng thức Cauchy-Schwarz, ta có:

x4 ( x 2 + y2 + z2 )2
∑ x 3 + y2 + z2
≥ 3
x + y3 + z3 + 2( x 2 + y2 + z2 )

( x 2 + y2 + z2 )2 1
Do đó ta chỉ cần chứng minh rằng 3 3 3 2 2 2
≥ hay:
x + y + z + 2( x + y + z ) 7

7( x2 + y2 + z2 )2 ≥ ( x3 + y3 + z3 )( x + y + z) + 2( x2 + y2 + z2 )( x + y + z)2

Khai triển và thu gọn, điều trên tương đương với:

∑(2(x2 + y2 + z2 ) − xy)(x − y)2 ≥ 0


Nhưng bất đẳng thức này luôn đúng hay bài toán ban đầu được chứng minh hoàn
toàn.
Cách 2. Tương tự lời giải thứ nhất, ta sẽ chứng minh rằng:

7( x 2 + y2 + z2 )2 ≥ ( x 3 + y3 + z3 ) + 2( x 2 + y2 + z2 )

Từ giả thiết, ta dễ dàng suy ra được các đẳng thức sau:

x2 + y2 + z2 = 1 − 2( xy + yz + zx )

x3 + y3 + z3 = 1 − 3( xy + yz + zx ) + 3xyz
Do đó bất đẳng thức trên sau khi thu gọn có thể viết lại được thành

7(1 − 2( xy + yz + zx ))2 ≥ 3xyz + 3 − 7( xy + yz + zx )

Do ( xy + yz + zx )2 ≥ 3xyz( x + y + z) = 3xyz nên ta chỉ cần chứng minh rằng

7(1 − 2( xy + yz + zx ))2 ≥ ( xy + yz + zx )2 + 3 − 7( xy + yz + zx )

Nhưng điều này tương đương với (3( xy + yz + zx ) − 1)(9( xy + yz + zx ) − 4) ≥ 0.


( x + y + z )2 1
Điều này đúng do xy + yz + zx ≤ = . Bài toán ban đầu được chứng
3 3
minh.
Cách 3. Nhận thấy rằng vai trò của x, y, z là bình đẳng nên không mất tính tổng quát,
giả sử rằng x ≥ y ≥ z. Khi đó do x, y, z ≤ 1, ta dễ dàng có các bất đẳng thức sau:
1 1 1
x4 ≥ y4 ≥ z4 và ≥ ≥
x 3 + y2 + z2 y3 + z2 + x 2 z3 + x 2 + y2

126
Từ đó theo bất đẳng thức Chebyshev và bất đẳng thức Cauchy-Schwarz, ta có:

x4 1 4 4 4 1 3( x 4 + y4 + z4 )
∑ x 3 + y2 + z2
≥ (x + y + z ) ∑ 3
3 x + y2 + z2

∑ x3 + 2 ∑ x2

3( x 4 + y4 + z4 ) 1
Do đó bây giờ ta chỉ cần chứng minh 3 2
≥ hay là:
∑x +2∑x 7

21( x4 + y4 + z4 ) ≥ x3 + y3 + z3 + 2( x2 + y2 + z2 )

Theo bất đẳng thức Cauchy-Schwarz và AM-GM, ta dễ dàng có được những điều sau:

18( x4 + y4 + z4 ) ≥ 6( x2 + y2 + z2 )2 ≥ 2( x2 + y2 + z2 )

4 3 1
3( x 4 + y4 + z4 ) ≥ ( x + y3 + z3 ) − ≥ x 3 + y3 + z3
3 27
Cộng hai bất đẳng thức trên lại theo vế, bài toán được chứng minh hoàn toàn.

Nhận xét. Lời giải thứ nhất ở trên là của Nguyễn Nguyễn, học sinh lớp 10 Toán trường
PTNK đề xuất, với ý tưởng chính là sử dụng bất đẳng thức Cauchy-Schwarz dạng
phân thức, sau đó thuần nhất hoá và đưa về dạng tổng các bình phương. Cũng ý
tưởng sử dụng bất đẳng thức Cauchy-Schwarz trong lời giải thứ hai, thầy Nguyễn
Việt Hùng, GV trường THPT chuyên KHTN, đã có một cách xử lý khá tự nhiên là cố
gắng đưa về bất đẳng thức 1 biến theo ab + bc + ca. Vũ Thế Việt, học sinh lớp 10 Toán
trường PTNK, tác giả của lời giải thứ ba đánh giá vế trái của bài toán bằng bất đẳng
thức Chebyshev, và phần sau của bài toán khá đơn giản như đã trình bày.

Bài toán 2. Tìm tất cả các hàm số f : N ∗ → N ∗ thoả mãn đồng thời các điều kiện:

i. f (mn) = f (m) f (n) ∀m, n ∈ N∗ .


ii. f (m) + f (n) chia hết cho m + n ∀m, n ∈ N∗ .
iii. f (2017) = 20173 .

Lời giải. Chúng ta có hai lời giải cho bài toán này.
Cách 1 (Lâm Hữu Phúc). Ta sẽ giải bài toán khi thay 2017 bởi p ∈ N ∗ bất kỳ.
Từ i., bằng phép quy nạp đơn giản, ta có f ( pn ) = p3n (n ∈ N ∗ ).
.
Từ ii., thay n bởi m, ta có f (m) .. m (m ∈ N ∗ ).
Đặt f (m) = m · g(m) (m ∈ N ∗ ). Với phép đặt đó thì ta dễ dàng có được các điều sau:

• g(mn) = g(m).g(n) ∀m, n ∈ N ∗


• m · g(m) + n · g(n) chia hết cho m + n ∀m, n ∈ N ∗
• g( pn ) = p2n (n ∈ N ∗ )
.
Lại đặt h(m) = g(m) − m2 và thay n bởi pn tại b), ta có m · h(m) .. m + pn .
Cố định số nguyên dương m và cho n đủ lớn. ta phải có h(m) = 0 ∀m ∈ N ∗ .

127
Từ đó ta có được g(m) = m2 hay f (m) = m3 ∀m ∈ N ∗ .
Thử lại, tất cả các hàm số thoả mãn là f (m) = m3 ∀m ∈ N ∗ .
Cách 2 (Nguyễn Nguyễn). Ta chứng minh được f ( pn ) = p3n (n ∈ N ∗ ).
Từ i., thay n bởi 1, ta tính được f (1) = 1.
.
Từ ii., ta suy ra f (n) + 20173m .. n + 2017m .
. .
Lại để ý rằng n3 + 20173m .. n + 2017m nên f (n) − n3 .. n + 2017m .
Cố định số nguyên dương n và cho m đủ lớn thì f (n) − n3 = 0 hay f (n) = n3 .
Thử lại, tất cả các hàm số thoả mãn là f (m) = m3 ∀m ∈ N ∗

Nhận xét. Lời giải thứ hai ngắn gọn hơn và phát triển bài toán được xa hơn khi ta
hoàn toàn thay được điều kiện iii bởi số mũ k lẻ bất kỳ.

Bài toán 3. Cho đường tròn (O) và dây cung AB cố định. C là một điểm thay đổi trên cung
lớn AB sao cho tam giác ABC nhọn. Gọi I, Ia , Ib lần lượt là tâm đường tròn nội tiếp, tâm
đường tròn bàng tiếp góc ∠ BAC và ∠ ABC của tam giác ABC.

1. Gọi M đối xứng với I qua O. Chứng minh rằng tam giác MIa Ib cân.
2. Gọi H, K lần lượt là hình chiếu của Ia , Ib trên OI. Đường thẳng qua H vuông góc với
BIa và đường thẳng qua K vuông góc với AIb cắt nhau tại P. Chứng minh rằng P thuộc
một đường cố định khi C thay đổi.

Lời giải. Lời giải sau đây là của bạn Vũ Thế Việt.

1. Gọi Ic là tâm đường tròn bàng tiếp góc ∠ BCA thì (O) chính là đường tròn Euler
của 4 Ia Ib Ic . Mặt khác 4 Ia Ib Ic có O là tâm đường tròn Euler và I là trực tâm nên
đối xứng M của I qua O chính là tâm ngoại tiếp 4 Ia Ib Ic hay 4 MIb Ic cân tại M.
2. Không mất tính tổng quát, giả sử vị trí tương đối của các điểm như hình vẽ trên.
Gọi D, E, F lần lượt là trung điểm BC, CA, AB thì các tứ giác ODBP, ODCQ
nội tiếp nên ∠ DPQ = ∠OBC = ∠OCB = ∠ DQP hay tam giác DPQ cân tại
D, dẫn đến ∠ PDQ = 180◦ − 2∠ DPQ = 180◦ − 2∠OBC = ∠ BOC = 2∠ BAC.
Mặt khác ∠ PRQ = 180◦ − ∠ BAC và D, R khác phía với PQ nên D là tâm ngoại
tiếp tam giác PQR. Do DF || AC nên PR ⊥ AC hay P, R đối xứng qua DF,
tương tự thì Q, R đối xứng qua DE. Từ đó ∠ PFR = 2∠ PFD = 2∠ POD =
360◦ − 2∠ DOQ = 360◦ − 2∠ DEQ = ∠ REQ hay 4 PFR ∼ 4 REQ (c.g.c). Vậy
∠ FRE = ∠ PRQ = ∠180◦ − ∠ BAC = 180◦ − ∠ FDE nghĩa là R thuộc đường
tròn Euler của tam giác ABC.

Bài toán kết thúc.

Nhận xét. Qua lời giải trên, ta nhận thấy vai trò của điểm I không quan trọng nên
đường thẳng OI có thể thay thế bằng đường thẳng bất kỳ đi qua tâm ngoại tiếp. Bài
toán trên không mới mà chỉ là một kết quả kinh điển về cực trực giao: cực trực giao

128
của đường thẳng đi qua tâm ngoại tiếp thì nằm trên đường tròn Euler của tam giác
đó. Tuy nhiên phép chứng minh trên lại rất mới khi chỉ sử dụng các kiến thức đơn
giản về tam giác đồng dạng và biến đổi góc. Về khái niệm cực trực giao, có thể tham
khảo thêm trong [2].
Về kết quả được nêu ra ở phần sau bài toán, xin được giới thiệu một chứng minh
khác tham khảo từ thầy Trần Quang Hùng, trường THPT chuyên KHTN tại [3]: Gọi l
là đường thẳng qua trực tâm H của tam giác ABC và vuông góc với d. Gọi S là điểm
anti-Steiner của l. K đối xứng H qua BC thì K thuộc (O). Do S là điểm anti-Steiner
của l nên T đối xứng S qua BC nằm trên l. Từ đó gọi ST cắt (O) tại X khác S thì theo
tính đối xứng ta có ∠ AXS = ∠ AKS = ∠KHT do đó AX k HT ⊥ d nên AX đi qua D
hay D là trung điểm AX. Từ đó đường thẳng qua D vuông góc BC đi qua trung điểm
G của SH. Tương tự các đường thẳng còn lại qua G và dễ thấy G nằm trên đường
tròn Euler của tam giác ABC 

Bài toán 4. Cho S là tập hợp khác rỗng và A1 , A2 , ..., Am (m ≥ 2) là m tập con của S. Gọi
T là tập hợp gồm tất cả các tập hợp Ai 4 A j (1 ≤ i, j ≤ m). Chứng minh rằng | T | ≥ m.

Lời giải. Lời giải sau được đề nghị bởi Nguyễn Nguyễn và đã được chỉnh sửa lại cho
rõ ràng và gọn gàng hơn.
Trước hết ta chứng minh bổ đề sau: "Nếu M 4 N = M 4 P thì N = P".
Thật vậy, giả sử rằng N 6= P. Không mất tính tổng quát, gọi x là phần tử sao cho
x ∈ N và x 6∈ P. Có hai khả năng sau xảy ra:

• Nếu x ∈ M, khi đó ta có x 6∈ M\ N và x 6∈ N \ M, do đó x 6∈ M 4 N, nhưng lại


có x ∈ M4 P nên M4 N 6= M4 P, trái với giả thiết.
• Nếu x 6∈ M, khi đó tương tự ta có x ∈ M4 N và x 6∈ M 4 P, dẫn đến M 4 N 6=
M4 P, trái với giả thiết.

Do đó N = P. Bổ đề được chứng minh. Quay trở lại bài toán ban đầu.
Ta sẽ thực hiện chứng minh bằng quy nạp theo m. Với m = 2, dễ thấy rằng T =
{∅, A1 4 A2 } hay | T | = 2. Bây giờ giả sử rằng bài toán đã đúng đến m = k (k ∈
N ∗ , k ≥ 2), ta sẽ chứng minh rằng bài toán cũng đúng với m = k + 1. Nếu như với
m = k mà ta có | T | = k + 1 thì bài toán hiển nhiên đúng, nên ta chỉ xét trường hợp
| T | = k.
Xét dãy tập hợp Bi = Ak+1 4 Ai (1 ≤ i ≤ k + 1). Ta bổ sung vào T các tập hợp Bi . Rõ
ràng | T | là đại lượng không giảm. Giả sử rằng vẫn xảy ra | T | = k, do trong dãy tập
hợp vừa thêm có k + 1 tập nên phải tồn tại chỉ số p, q (1 ≤ p < q ≤ k + 1) sao cho
B p = Bq . Từ đó theo bổ đề dẫn đến A p = Aq , trái với giả thiết các tập hợp Ai là đôi
một phân biệt. Do đó điều đã giả sử là sai, hay là | T | ≥ k + 1. Tóm lại ta luôn chứng
minh được | T | ≥ k + 1, quy nạp thành công.
Vậy bài toán được giải quyết hoàn toàn.

129
3. LỜI KẾT
Sau kỳ thi, đội dự tuyển khối 10 gồm 17 học sinh đã được thành lập, và 3 học sinh
sau đây đã được chọn để tham dự kỳ thi Olympic truyền thống 30/4 lần thứ XXIII
môn Toán 10:

1. Nguyễn Nguyễn, lớp 10 Toán.


2. Vũ Thế Việt, lớp 10 Toán.
3. Nguyễn Tiến Hoàng, lớp 10 Toán.

Xin cảm ơn thầy Nguyễn Tăng Vũ đã đọc lại toàn bộ và chỉnh sửa bài viết.

TÀI LIỆU THAM KHẢO


1. Đề thi chọn đội dự tuyển PTNK năm 2017: http://diendantoanhoc.net/topic/169274-\T5\djề-thi-chọn-
\T5\djội-dự-tuyển-lớp-10-ptnk-\T5\djhqgtphcm/
2. Orthopole - Wolfram MathWorld: http://mathworld.wolfram.com/Orthopole.html.
3. VMF Marathon Hình học Olympic - Trang 12: http://diendantoanhoc.net/topic/159439-vmfs-marathon-
hình-học-olympic/page-12

130
LỜI GIẢI ĐỀ THI CHỌN ĐỘI TUYỂN TOÁN
TRƯỜNG PHỔ THÔNG NĂNG KHIẾU
NĂM 2016, 2017, 2018
Nguyễn Ngọc Duy, Nguyễn Tăng Vũ,
Vương Trung Dũng, Lê Phúc Lữ

1. ĐỀ BÀI

1.1. Đề thi năm 2016


Bài 1. Tìm a để dãy số (un ) hội tụ biết u1 = a và

 2un − 1
 khi un > 0
u n +1 = −1 khi − 1 ≤ un ≤ 0 ∀n ∈ N

 u2 + 4u + 2 khi u < −1
n n n

Bài 2. Tìm số nguyên dương k nhỏ nhất sao cho bất đẳng thức

x k y k z k ( x 3 + y3 + z3 ) ≤ 3

với x, y, z là các số dương thoả mãn x + y + z = 3.

Bài 3. Cho hàm số f : N∗ → N∗ thoả mãn điều kiện: f tăng thực sự và f (2n) = 2 f (n) với
mọi số nguyên dương n.

a) Giả sử f (1) = 3 và p là số nguyên tố lớn hơn 3. Chứng minh rằng tồn tại n sao cho
f (n) chia hết cho p.
b) Cho q là số nguyên tố lẻ. Hãy xây dựng một hàm f thoả mãn các điều kiện của bài toán
mà f (n) không chia hết cho q với mọi n.

Bài 4. Cho tam giác ABC có góc ∠ BAC tù, AH ⊥ BC (H thuộc BC). Điểm M thay đổi trên
cạnh AB. Dựng điểm N sao cho ∆BMN ∼ ∆HCA (H và N khác phía đối với AB).

a) Gọi CM cắt đường tròn ngoại tiếp tam giác BMN tại điểm K (K 6= M). Chứng minh
rằng đường thẳng NK luôn đi qua một điểm cố định.

131
b) Gọi NH cắt AC tại điểm P. Dựng Q sao cho ∆HPQ ∼ ∆HN M (Q và M khác phía
đối với PN). Chứng minh rằng Q thuộc một đường thẳng cố định.

Bài 5. Với mỗi số nguyên dương n, ta biết rằng tồn tại duy nhất số tự nhiên a thoả mãn
a2 ≤ n < ( a + 1)2 . Đặt ∆n = n − a2 .

a) Tìm giá trị nhỏ nhất của ∆n khi n thay đổi và thoả mãn n = 15m2 với m ∈ N∗ .
b) Cho p, q là các số nguyên dương và d = 5(4p + 3)q2 . Chứng minh rằng ∆d ≥ 5.

Bài 6. Với các số nguyên a, b, c, d thoả mãn 1 ≤ a < b < c < d; kí hiệu

T ( a, b, c, d) = {{ x, y, z, t}|1 ≤ x < y < z < t; x ≤ a, y ≤ b, z ≤ c, t ≤ d}

a) Tính số phần tử của T (1, 4, 6, 7).


b) Cho a = 1, b ≥ 4. Gọi d1 là số phần tử của T ( a, b, c, d) chứa 1 và không chứa 2; d2 là
số phần tử chứa 1, 2 và không chứa 3; d3 là số phần tử chứa 1, 2, 3 và không chứa 4.
Chứng minh rằng d1 ≥ 2d2 − d3 . Đẳng thức xảy ra khi nào?

Bài 7. Trong một hệ thống máy tính, một máy tính bất kì có kết nối trực tiếp với ít nhất 30%
máy tính khác của hệ thống. Hệ thống này có một chương trình cảnh báo và ngăn chặn khá
tốt, do đó khi một máy tính bị virus, nó chỉ có đủ thời gian lây cho các máy tính được kết nối
trực tiếp với nó. Chứng minh rằng dù vậy, kẻ tấn công vẫn có thể chọn hay máy tính của hệ
thống mà nếu thả virus vào hai máy đó, ít nhất 50% máy tính của hệ thống bị nhiễm virus.

Bài 8. Cho tam giác ABC nhọn. Đường tròn ( I ) có tâm I thuộc cạnh BC và tiếp xúc với các
cạnh AB, AC lần lượt tại E, F. Lấy M, N bên trong tứ giác BCEF sao cho EFN M nội tiếp
( I ) và các đường thẳng MN, EF, BC đồng quy. MF cắt NE tại P, AP cắt BC tại D.

a) Chứng minh rằng A, D, E, F cùng thuộc một đường tròn.


b) Lấy trên các đường thẳng BN, CM cắt điểm H, K sao cho ∠ ACH = ∠ ABK = 90o .
Gọi T là trung điểm HK. Chứng minh rằng TB = TC.

1.2. Đề thi năm 2017


Bài 1. Cho dãy số (un ) thoả mãn un > 0, un > un+1 ∀n ∈ N∗ và dãy (sn ) hội tụ với
n
sn = ∑ ui .
i =1

a) Chứng minh rằng lim(nun ) = 0.


1 1
b) Đặt bn = − , ∀n ∈ N∗ . Chứng minh rằng dãy số (bn ) không bị chặn.
u n +1 u n

Bài 2. Gọi S là tập con của {1, 2, . . . , 2017} sao cho S không chứa hai phần tử mà phần tử
này chia hết cho phần tử kia và cũng không chứa hai phần tử nguyên tố cùng nhau. Hỏi S
chứa nhiều nhất bao nhiêu phần tử?

132
Bài 3. Cho n là số tự nhiên lớn hơn 2 và tập hợp X = {1, 2, 3, . . . , n}. Với mỗi song ánh
f : X → X, gọi A f là tập hợp tất cả các bộ số (i; j) sao cho i < j và f (i ) > f ( j).

a) Có bao nhiêu song ánh f thoả mãn | A f | = 1?


b) Giả sử f là một song ánh mà | A f | = k > 0. Chứng minh rằng tồn tại một song ánh
n n
g : X → X sao cho | A g | = k − 1 và ∑ | f (i ) − i | > ∑ | g (i ) − i |.
i =1 i =1

Ở đây | X | chỉ số phần tử của tập hợp X.

Bài 4. Cho tam giác ABC nhọn nội tiếp (O) và điểm D di động trên cung BC chứa điểm A
(D 6= A). Trên AB, AC lần lượt lấy các điểm M, N sao cho MD = MB, NC = ND.

a) Chứng minh đường cao DH trong tam giác DMN luôn đi qua một điểm cố định.
b) DM, DN theo thứ tự cắt (O) tại E, F ( E, F khác D). Chứng minh các đường tròn
ngoại tiếp các tam giác EMB, FNC cắt nhau tại điểm K thuộc đường thẳng BC và
đường cao KI của tam giác KMN luôn qua một điểm cố định.

Bài 5. Với mỗi số nguyên n ∈ / {−1, 0, 1}, ký hiệu p(n) là ước nguyên tố lớn nhất của n. Gọi
F là tập hợp tất cả các đa thức f ( x ) có hệ số nguyên và thoả mãn

f (n + p(n)) = n + p( f (n))

với mọi số nguyên n > 2017 và f (n) ∈


/ {−1, 0, 1}.

a) Tìm tất cả các đa thức bậc nhất thuộc F.


b) F có bao nhiêu phần tử ?

Bài 6. Với mỗi số tự nhiên n, ký hiệu T (1 + n, 3 + n, 4 + n) là tập hợp tất cả các bộ ( a, b, c)


với a, b, c là các số tự nhiên thoả mãn:

1 ≤ a ≤ 1 + n, a + 1 ≤ b ≤ 3 + n, b + 1 ≤ c ≤ 4 + n

Gọi an là số phần tử của T (1 + n, 3 + n, 4 + n).

a) Tính a4 .
b) Tìm tất cả n sao cho an chia hết cho 3.

Bài 7. An và Bình luân phiên đánh dấu các ô vuông của hình vuông 101 × 101 ô. An là
người bắt đầu. Một ô sẽ không thể được đánh dấu nếu trên cùng hàng với nó hoặc cùng cột
với nó đã có ít nhất 2 ô được đánh dấu. Ai không đi được nữa sẽ thua. Hãy xác định ai là
người có chiến thuật thắng.

Bài 8. Cho đường tròn (O) nội tiếp tứ giác ABCD và lần lượt tiếp xúc với các cạnh
AB, BC, CD, DA tại E, F, G, H. Gọi I, J là trung điểm của AC, BD và gọi IB, ID, J A, JC
theo thứ tự cắt EF, GH, HE, FG tại M, N, P, Q.

133
a) Chứng minh rằng I J, MN, PQ đồng quy. Gọi điểm đồng quy là S.
b) Các tia đối của các tia J A, IB, JC, ID lần lượt cắt (O) tại các điểm A0 , B0 , C 0 , D 0 . Giả
sử A0 C 0 , B0 D 0 lần lượt cắt PQ, MN tại U, V. Gọi K là hình chiếu của S trên UV.
Chứng minh rằng ∠ AKB = ∠CKD.

1.3. Đề thi năm 2018


Bài 1. Cho số nguyên dương a > 1. Tìm giá trị lớn nhất của số thực d sao cho tồn tại một
cấp số cộng có công sai d và số hạng đầu là a và có đúng hai số trong các số a2 , a3 , a4 , a5 là
các số hạng của cấp số cộng đó.

Bài 2. Cho n số thực x1 , x2 , ..., xn . Với mỗi i ∈ {1, 2, 3, . . . , n}, gọi ai là số các chỉ số j sao

cho xi − x j ≤ 1 và bi là số các chỉ số j sao cho xi − x j ≤ 2 (i, j có thể bằng nhau).

a) Chứng minh rằng tồn tại i mà bi ≤ 3ai .



b) Gọi A là số cặp số (i, j) có thứ tự mà xi − x j ≤ 1 và B là số cặp số (i, j) có thứ tự mà

xi − x j ≤ 2 (các số i, j có thể bằng nhau). Chứng minh rằng B ≤ 3A.

Bài 3. Cho p là số tự nhiên. Xét phương trình x3 + x + p = y2 trên tập hợp các số nguyên
dương.

a) Tìm số nguyên tố p nhỏ nhất dạng 4k + 1 sao cho phương trình có nghiệm.
b) Chứng minh rằng nếu p là số chính phương thì phương trình luôn có nghiệm.

Bài 4. Cho tam giác ABC nhọn nội tiếp trong đường tròn (O) có B, C cố định và A di động
trên (O). Gọi D là trung điểm BC. Trên AB lấy điểm M, P và trên AC lấy các điểm N, Q
sao cho DA = DP = DQ, DM ⊥ AC, DN ⊥ AB.

a) Chứng minh rằng các điểm M, N, P, Q cùng thuộc một đường tròn, đặt là (C ) và
đường tròn này luôn đi qua một điểm cố định.
b) Chứng minh rằng tâm của (C ) luôn di chuyển trên một đường tròn cố định.

Bài 5. Cho số thực a 6= 0 và dãy số (un ) thoả mãn:


(
u1 = 0,
u n +1 ( u n + a ) = a + 1 ∀ n ∈ Z+

Tìm giới hạn của dãy số (un ).

Bài 6. Tìm tất cả các hàm số f : R+ → R+ thoả mãn điều kiện


 
f x f (y2 ) − y f ( x2 ) = (y − x ) f ( xy) ∀ y > x > 0.

134
Bài 7. Cho n = 2018.2019. Gọi A là tập hợp tất cả các bộ số ( a1 ; a2 ; . . . ; an−1 ; an ) có tính
n
thứ tự sao cho ai ∈ {0; 1} ∀i = 1, 2, ..., n và ∑ ai = 20182. Có bao nhiêu bộ số như thế để
i =1
thoả mãn
k k
k k
∑ ai ≥ 2 và ∑ an−k+1 ≥ 2 với k = 1, 2, ..., n?
i =1 i =1

Bài 8. Cho đường tròn (C ) có tâm I nội tiếp tam giác ABC và tiếp xúc với các cạnh AB, AC
tại E, F. Gọi AM, AN là phân giác trong và ngoài của tam giác với M, N ∈ BC. Gọi d M , d N
lần lượt là tiếp tuyến khác BC của (C ) và đi qua M, N.

a) Chứng minh rằng d M , d N và EF đồng quy. Gọi điểm đồng quy đó là D.


b) Trên AB, AC lần lượt lấy P, Q thoả mãn DP k AC, DQ k AB. Gọi R, S lần lượt là
trung điểm DE, DF. Chứng minh rằng I thuộc đường thẳng qua trực tâm của các tam
giác DPS, DQR.

2. LỜI GIẢI

2.1. Đề thi năm 2016


Bài 1. Tìm a để dãy số (un ) hội tụ biết u1 = a và

 2un − 1
 khi un > 0
u n +1 = −1 khi − 1 ≤ un ≤ 0 ∀n ∈ N

 u2 + 4u + 2 khi u < −1
n n n

Lời giải. Ta xét các trường hợp sau:

a) Nếu a > 1 thì u2 = 2u1 − 1 > 1, từ đó un > 1 với mọi n. Hơn nữa tìm được công
thức tổng quát un = 2n−1 ( a − 1) + 1. Dãy số (un ) không hội tụ.
b) Nếu a = 1 thì u2 = 1. Dễ dàng có được un = 1, ∀n. Vậy (un ) hội tụ.
c) Nếu 0 < a < 1, ta chứng minh rằng trong dãy có ít nhất một số hạng âm.
Thật vậy, giả sử rằng un > 0 ∀n ∈ N∗ . Dễ dàng có được 0 < un < 1 và:

u n = 2n −1 ( a − 1 ) + 1

Cho n → ∞ thì ta có un → −∞, vô lý. Do đó tồn tại n sao cho un < 0. Gọi
k là chỉ số nhỏ nhất để thoả mãn uk < 0 thì k ≥ 2 và 0 < uk−1 < 1, khi đó
−1 < uk = 2uk−1 − 1 < 0 và uk+1 = −1 hay un = −1 ∀n ≥ k. Vậy (un ) hội tụ.
d) Nếu −1 ≤ a ≤ 0 thì u2 = −1, từ đó un = −1 ∀n > 1. Dãy số (un ) hội tụ.
e) Nếu a < −1, ta có u2 = a2 + 4a + 2. Do các trường hợp đã xét, nếu u2 > 1 hay

nói cách khác là a < −2 − 3 thì dãy số không hội tụ. Ta chứng minh rằng (un )

hội tụ khi −2 − 3 ≤ a < −1.

135
• Nếu −2 < a < −1, ta có u2 − u1 = ( a + 2)( a + 1) < 0, suy ra u2 < u1 < −1
và u2 + 2 = ( a + 2)2 > 0 nên u2 > −2. Do đó −2 < u2 < −1. Bằng quy
nạp, ta chứng minh được (un ) giảm và −2 < un < −1 nên (un ) hội tụ.

• Nếu −2 − 3 ≤ a ≤ −2 thì u2 + 2 = ( a + 2)2 , suy ra −1 ≤ u2 ≤ 1. Theo
các trường hợp dã xét thì (un ) hội tụ.

Vậy dãy số (un ) hội tụ khi và chỉ khi −2 − 3 ≤ a ≤ 1.

Bài 2. Tìm số nguyên dương k nhỏ nhất sao cho bất đẳng thức

x k y k z k ( x 3 + y3 + z3 ) ≤ 3

với x, y, z là các số dương thoả mãn x + y + z = 3.


 
1 1
Lời giải. Với k = 1, ta chọn bộ số , , 2 thì bất đẳng thức sai.
  2 2
4 4 7
Với k = 2 thì bộ số , , cũng làm bất đẳng thức sai.
5 5 5
Ta chứng minh rằng với k = 3 thì bất đẳng thức đúng hay

x 3 y3 z3 ( x 3 + y3 + z3 ) ≤ 3 (1)

Không mất tính tổng quát, giả sử z nhỏ nhất. Khi đó z ≤ 1.


Ta có x3 + y3 = ( x + y)3 − 3xy( x + y) = (3 − z)3 − 3xy( x + y) và

3 1
(1) ⇔ (3 − z )3 + z3 ≤ + 3xy( x + y) ⇔ 3z2 − 9z + 9 ≤ + x2 y + xy2
x 3 y3 z3 x 3 y3 z3

Lại có s
1 2 2 3 x 3 y3 3
3 3 3
+ x y + xy ≥ 3 3 3 3
=
x y z x y z z
Do đó ta chỉ cần chứng minh rằng:

3
3z2 − 9z + 9 ≤ ⇔ 3(z − 1)3 ≤ 0 (đúng)
z

Vậy (1) đúng hay k = 3 là số nguyên dương nhỏ nhất thoả mãn đề bài.

Bài 3. Cho hàm số f : N∗ → N∗ thoả mãn điều kiện: f tăng thực sự và f (2n) = 2 f (n) với
mọi số nguyên dương n.

a) Giả sử f (1) = 3 và p là số nguyên tố lớn hơn 3. Chứng minh rằng tồn tại n sao cho
f (n) chia hết cho p.
b) Cho q là số nguyên tố lẻ. Hãy xây dựng một hàm f thoả mãn các điều kiện của bài toán
mà f (n) không chia hết cho q với mọi n.

136
Lời giải.

a) Đặt A = { f (n + 1) − f (n)|n ∈ N∗ }. Vì f là tăng thực sự nên A là tập con của


tập hợp các số nguyên dương. Khi đó tồn tại phần tử nhỏ nhất của A, ta đặt là
k. Khi đó tồn tại số nguyên dương n thoả mãn k = f (n + 1) − f (n). Khi đó:

f (2n + 2) − f (2n) = 2 f (n + 1) − 2 f (n) = 2k

Mặt khác ta có: 



 f (2n) < f (2n + 1) < f (2n + 2)

f (2n + 1) − f (2n) ≥ k


 f (2n + 2) − f (2n + 1) ≥ k

Do đó ta có f (2n + 1) − f (2n) = k hay f (2n + 1) = f (2n) + k.


Bằng quy nạp, ta chứng minh được:

f (2i n) = 2i f (n), f (2i n + 1) = 2i f (n) + k, ..., f (2i n + i ) = 2i f (n) + ik.

Do f (1) = 3, f (2) = 6 nên k ≤ 3 hay (k, p) = 1. Khi đó các số

f (2 p n), f (2 p n + 1), f (2 p n + 2), ... f (2 p n + p − 1)

sẽ tạo thành một hệ thặng dư đầy đủ modulo p nên sẽ có một số chia hết cho p.
b) Ta xây dựng hàm số f như sau:

 a
 f (1) = 2 > q

f (2n) = 2 f (n) ∀n ∈ N∗


 f (2n + 1) = f (2n) + q ∀n ∈ N∗

Ta chứng minh f là hàm số thoả mãn đề bài.

i. f là hàm tăng thực sự. Ta chứng minh f (n + 1) − f (n) ≥ q bằng quy nạp.
Với n = 1, ta có f (2) − f (1) = 2.2a − 2a = 2a > q. Giả sử điều này đúng
đến k. Ta xét f (k + 1).
• Nếu k chẵn, ta có f (k + 1) = f (k ) + q, thoả mãn yêu cầu.
• Nếu k lẻ, ta có:
     
k+1 k−1
f ( k + 1) = 2 f ≥2 f + q = f (k − 1) + 2q.
2 2

Vậy ta có điều cần chứng minh.


ii. Ta chứng minh không tồn tại n thoả mãn f (n) chia hết cho q bằng quy nạp.
• Ta có f (1) = 2a không chia hết cho q.

137
• Giả sử điều này đúng đến k.
– Nếu k chẵn thì f (k + 1) = f (k ) + q không chia hết cho q.
 
k+1
– Nếu k lẻ thì f (k + 1) = 2 f không chia hết cho q.
2
Vậy f (n) không chia hết cho q với mọi số nguyên dương n.

Tóm lại hàm số f được xây dựng như trên thoả mãn đề bài. Bài toán kết thúc.

Bài toán kết thúc.

Bài 4. Cho tam giác ABC có góc ∠ BAC tù, AH ⊥ BC (H thuộc BC). Điểm M thay đổi trên
cạnh AB. Dựng điểm N sao cho ∆BMN ∼ ∆HCA (H và N khác phía đối với AB).

a) Gọi CM cắt đường tròn ngoại tiếp tam giác BMN tại điểm K (K 6= M). Chứng minh
rằng đường thẳng NK luôn đi qua một điểm cố định.
b) Gọi NH cắt AC tại điểm P. Dựng Q sao cho ∆HPQ ∼ ∆HN M (Q và M khác phía
đối với PN). Chứng minh rằng Q thuộc một đường thẳng cố định.

A
K
M

N
C
F

B H

Lời giải.

138
a) Gọi X là giao điểm của đường thẳng vuông góc với BC tại B và đường thẳng
AC. K 0 là giao điểm của NX và CM. Ta có ∆BMN ∼ ∆BCX (cùng hướng). Do
đó có một phép vị tự quay tâm B, biến M 7→ N, C 7→ X. Khi đó K 0 là giao
điểm của CM và BX thì K 0 thuộc đường tròn ngoại tiếp tam giác BMN. Từ đây
K 0 ≡ K. Vậy NK luôn đi qua điểm X cố định.
b) Xét phép vị tự tâm H : N 7→ P, M 7→ Q, B 7→ F. Ta có được ∆BMN ∼ ∆FQP.
Từ đó ∠ FQP = ∠ BMN = ∠ ACB = ∠ FCP hay tứ giác CFPQ nội tiếp. Do đó
∠QCP = ∠QFP = ∠ MBN = 90◦ . Vậy Q thuộc đường thẳng qua C vuông góc
với AC cố định.

Bài toán kết thúc.

Bài 5. Với mỗi số nguyên dương n, ta biết rằng tồn tại duy nhất số tự nhiên a thoả mãn
a2 ≤ n < ( a + 1)2 . Đặt ∆n = n − a2 .

a) Tìm giá trị nhỏ nhất của ∆n khi n thay đổi và thoả mãn n = 15m2 với m ∈ N∗ .
b) Cho p, q là các số nguyên dương và d = 5(4p + 3)q2 . Chứng minh rằng ∆d ≥ 5.

Lời giải.

a) Ta cần tìm số ∆n nhỏ nhất để phương trình 15m2 − a2 = ∆n có nghiệm nguyên


dương. Ta thấy 15 − 32 = 6. Do đó ta sẽ chứng minh phương trình không có
.
nghiệm với ∆n < 6. Ta có a2 + ∆n .. 3 nên

. .
∆n .. 3 hoặc ∆n + 1 .. 3 (2)

.
Mặt khác a2 + ∆n .. 5 nên

∆n ≡ 0, 1, 4 (mod 15) (3)

Từ (2) và (3), ta có nếu ∆n < 6 thoả mãn đề bài thì ∆n = 5.


Ta có 15m2 − a2 = 5 nên a là bội của 5. Đặt a = 5s, ta có 3m2 − 5s2 = 1.
Mà 3m2 − 1 ≡ −1, 2, 1 (mod 5) nên phương trình vô nghiệm. Vậy ∆n nhỏ nhất
là 6.
b) Xét phương trình 5(4p + 3)q2 − a2 = ∆d . Ta có a2 ≡ 0, 1, 4 (mod 5) nên k ≡
0, 1, 4 (mod 5). Ta có 5(4p + 3) ≡ 3 (mod 4) nên không phải số chính phương.
Nếu ∆d = 1 ta có a2 + 1 = 5(4p + 3)q2 . Ta có 5(4p + 3) là số có dạng 4k + 3
nên tồn tại ước nguyên tố r có dạng 4k + 3, suy ra a2 + 1 ≡ 0 (mod r ) và 1 ≡ 0
(mod r ), vô lý.
Tương tự cho trường hợp ∆d = 4 thì phương trình 5(4p + 3)q2 − a2 = 4 cũng
vô nghiệm. Vậy ∆d ≥ 5.

Bài toán kết thúc.

139
Bài 6. Với các số nguyên a, b, c, d thoả mãn 1 ≤ a < b < c < d; kí hiệu

T ( a, b, c, d) = {{ x, y, z, t}|1 ≤ x < y < z < t; x ≤ a, y ≤ b, z ≤ c, t ≤ d}

a) Tính số phần tử của T (1, 4, 6, 7).


b) Cho a = 1, b ≥ 4. Gọi d1 là số phần tử của T ( a, b, c, d) chứa 1 và không chứa 2; d2 là
số phần tử chứa 1, 2 và không chứa 3; d3 là số phần tử chứa 1, 2, 3 và không chứa 4.
Chứng minh rằng d1 ≥ 2d2 − d3 . Đẳng thức xảy ra khi nào?

Lời giải.

a) Do x ≤ 1 nên x = 1. Do đó 2 ≤ y ≤ 4 và y ∈ {2, 3, 4}. Có các khả năng:

i. Nếu y = 2 thì 3 ≤ z ≤ 6. Mỗi giá trị của z có 7 − z giá trị của t nên có 10 bộ
số.
ii. Nếu y = 3 thì 4 ≤ z ≤ 6. Tương tự, ta có 6 bộ số.
iii. Nếu y = 4 thì 5 ≤ z ≤ 6. Tương tự, ta có 3 bộ số.

Vậy có 19 bộ số thoả mãn yêu cầu bài toán.


b) Đặt các tập hợp sau:


 T1 = T (1, b, c, d) = {(1, y, z, t)|3 ≤ y ≤ b, y ≤ z ≤ c, z ≤ t ≤ d}

T2 = T (1, 2, c, d) = {(1, 2, z, t)|4 ≤ z ≤ c, z ≤ t ≤ d}


 T = T (1, 2, 3, d) = {(1, 2, 3, t)|5 ≤ t ≤ d}
3

Ta có d3 = | T3 | = d − 4 và

c
( c + 4) ( c − 3)
d2 = ∑ ( d − z ) = ( c − 3) d + 2
z =4

Tiếp theo ta tính d1 = | T1 |. Vì b ≥ 4 nên y ≥ 3. Có các khả năng sau:

• Nếu y = 3 thì T (1, 3, z, t) = d2 .


c
(c + 5)(c − 4)
• Nếu y = 4 thì T (1, 4, z, t) = ∑ ( d − z ) = ( c − 4) d − 2
.
z =5

(c + 5)(c − 4)
Suy ra d1 ≥ d2 + (c − 4)d − . Do đó:
2

(c + 5)(c − 4) (c + 4)(c − 3)
d1 + d3 − 2d2 ≥ (c − 4)d − + d − 4 − ( c − 3) d + =0
2 2

Vậy d1 + d3 ≥ 2d2 . Đẳng thức xảy ra khi b = 4. Bài toán kết thúc.

Bài toán kết thúc.

140
Bài 7. Trong một hệ thống máy tính, một máy tính bất kì có kết nối trực tiếp với ít nhất 30%
máy tính khác của hệ thống. Hệ thống này có một chương trình cảnh báo và ngăn chặn khá
tốt, do đó khi một máy tính bị virus, nó chỉ có đủ thời gian lây cho các máy tính được kết nối
trực tiếp với nó. Chứng minh rằng dù vậy, kẻ tấn công vẫn có thể chọn hay máy tính của hệ
thống mà nếu thả virus vào hai máy đó, ít nhất 50% máy tính của hệ thống bị nhiễm virus.

Lời giải. (Trần Nam Dũng) Trước hết ta chứng minh bổ đề sau:

Bổ đề. Xét một tập con S bất kỳ của tập các máy tính X. Khi đó tồn tại 1 máy tính của hệ
thống kết nối trực tiếp với ít nhất 30% máy tính của S.

Chứng minh. Xét các cặp (s, x ) với s thuộc S và x thuộc X và s, x kết nối trực tiếp
3
nhau. Khi đó, tính theo s thì số cặp như vậy sẽ không ít hơn |S|.| X |. Do đó nếu tính
10
3
theo x thì sẽ phải tồn tại x kết nối trực tiếp với ít nhất |S|.
10
Quay trở lại bài toán. Xét hệ thống có n máy tính và máy tính A bất kỳ. Gọi S là tập
hợp các máy tính không kết nối trực tiếp với A. Nếu S rỗng thì bài toán là hiển nhiên.
Nếu S không rỗng thì theo bổ đề, tồn tại máy tính B kết nối trực tiếp với ít nhất 30%
máy tính trong S. Ta chứng minh hai máy tính A và B thoả mãn yêu cầu bài toán.

Thật vậy, giả sử A kết nối trực tiếp với k máy khác. Khi đó, theo cách chọn A hợp với
B sẽ kết nối trực tiếp với ít nhất:

k + 0, 3(n − k ) = 0, 7k + 0, 3n ≥ 0, 7.0, 3n + 0, 3n = 0, 51n

Do đó ta có điều phải chứng minh.

Bài 8. Cho tam giác ABC nhọn; đường tròn ( I ) có tâm I thuộc cạnh BC và tiếp xúc với các
cạnh AB, AC lần lượt tại E, F. Lấy M, N bên trong tứ giác BCEF sao cho EFN M nội tiếp
( I ) và các đường thẳng MN, EF, BC đồng quy. MF cắt NE tại P, AP cắt BC tại D.

a) Chứng minh A, D, E, F cùng thuộc một đường tròn.


b) Lấy trên các đường thẳng BN, CM cắt điểm H, K sao cho ∠ ACH = ∠ ABK = 90o .
Gọi T là trung điểm HK. Chứng minh TB = TC.

Lời giải.

141
A

F
E H
T
K N
M
Q
B D I C

Bổ đề 1 Cho đường tròn (O; R) và điểm Q nằm ngoài đường tròn. Cát tuyến qua Q cắt
(O) tại M, N. Tiếp tuyến tại M, N cắt nhau tại A, vẽ AD ⊥OQ. Khi đó OD.OQ = R2 .

Bổ đề 2 (Định lý Brocard) Cho tứ giác ABCD nội tiếp đường tròn. Gọi P là giao điểm
của AD và BC, Q là giao điểm của AB, CD. I là giao điểm của AC và BD. Khi đó
PI ⊥OQ và nếu D là giao điểm của PI và OQ thì OD.OQ = R2 .
P

A
I
K

Q
O
H

Bổ đề 3 (Định lý Desargue) Cho hai tam giác ABC và A0 B0 C 0 . Gọi M là giao điểm
của ABvA0 B0 , N là giao điểm của AC và A0 C 0 ; P là giao điểm của BC và B0 C. Khi đó
M, N, P thẳng hàng khi và chỉ khi AA0 , BB0 và CC 0 đồng quy.

142
Bổ đề 4. Cho tam giác ABC,hai tia Ax, Ay đối xứng nhau qua phân giác góc A. Gọi
H, K là hình chiếu của B trên Ax, Ay; P, Q là hình chiếu của C trên Ax, Ay. Khi đó 4
điểm H, K, P, Q cùng thuộc một đường tròn và tâm là trung điểm BC.
P
A

C
B
M

Trở lại bài toán.

a) Theo Bổ đề 1, gọi D 0 là hình chiếu của A trên BC. Ta có ID 0 .IQ = R2 .


Gọi X là giao điểm của ME và NF. XP cắt BC tại D 00 . Suy ra XP⊥ BC và
ID 00 .IQ = R2 . Suy ra D 00 ≡ D 0 . Từ đó ta có X, A, P, D thẳng hàng và EFID nội
tiếp.

b) Gọi S là giao điểm của CM và CN. Áp dụng định lý Desargue cho hai tam giác
PEF và SBC ta có A, P, S thẳng hàng. Suy ra S ∈ AD.
Ta chứng minh được ∠ BAK = ∠CAH. Suy ra AK, AH đối xứng nhau qua phân
giác góc ∠ BAC.
Áp dụng Bổ đề 4, ta có trung điểm HK cách đều BC.

Bài toán kết thúc.


2.2. Đề thi năm 2017
n
Bài 1. Cho dãy số (un ) thoả un > 0, un > un+1 , ∀n ∈ N∗ và dãy (sn ) hội tụ với sn = ∑ ui
i =1
.

a) Chứng minh lim(nun ) = 0.


1 1
b) Đặt bn = − , ∀n ∈ N∗ . Chứng minh dãy (bn ) không bị chặn.
u n +1 u n

143
Lời giải.

a) Lấy ε > 0 bất kỳ. Dãy (sn ) hội tụ nên ta có


ε
Với > 0 luôn tồn tại N > 0 sao cho
2
ε
∀n > m > N ⇒ |sn − sm | < (tính chất của dãy Cauchy) (4)
2
hay khi đó
ε
| u m +1 + u m +2 + · · · + u n | < .
2
Cố định m = N + 1.

Do dãy giảm nên suy ra (n − m) un < ε, hay


ε
nun < + mun . (5)
2

Lại do dãy (sn ) hội tụ nên tiếp tục áp dụng (4) cho n = m + 1, ta chứng minh
được (un ) → 0.
Khi đó, với
ε ε
> 0, ∃ N 0 : ∀n > N 0 ⇒ |un | <
2m 2m
ε
mun < (6)
2
0
Từ (5) và (6) suy ra ∀n > max { N, N } thì nun < ε. Vậy ta có đpcm.
b) (Kỹ thuật tách tương tự định lý Cesaro). Giả sử (bn ) bị chặn bởi c. Ta có
   
1 1 1 1 1 1
− +···+ − +
1 u u n u n −1 u1 u0 u0
− ( c + 1) = n − ( c + 1) = − ( c + 1)
nun n n
 
n −1
 ∑ bi  1
 i =0 
= − ( c + 1)  +
 n  nu0

cn 1 1
≤ − ( c + 1) + = − 1, với mọi n.
n nu0 nu0

1 1
Suy ra ≤ (c + 1), khi n đủ lớn thoả mãn < 1.
nun nu0
Điều này mâu thuẫn câu 1. Do đó điều giả sử là sai và ta có đpcm.

Bài toán kết thúc.

Nhận xét. Trong bài toán này, câu a chính là một bước quan trọng để xử lý câu b. Tuy
nhiên, ta có thể chứng minh trực tiếp kết quả ở b như sau.

144
Giả sử ngược lại rằng dãy (bn ) bị chặn, tức là tồn tại M > 0 để bn < M với mọi n.
Khi đó ta có
n −1  
1 1 1 1 1
= +∑ − ≤ + (n − 1) M.
un u 1 k =1 u k +1 u k u1
Suy ra
u1 1
un ≥ = .
1 + ( n − 1) u1 M 1
+ ( n − 1) M
u1
1
Ta thấy rằng với n đủ lớn, giả sử rằng n ≥ N0 thì (n − 1) M > nên khi đó
u1
1
un > . Khi đó, tổng
2( n − 1) M
n
1 1
s n = a1 + a2 + . . . + a n > ∑
2M k= N k − 1
.
0

n1 n 1
Chú ý rằng tổng ∑ → +∞ nên tổng ∑ cũng thế. Điều này cho thấy
k =1 k k= N0 k − 1
sn → +∞, mâu thuẫn.
Vậy dãy bn không bị chặn.

Bài 2. Gọi S là tập con của {1, 2, . . . , 2017} sao cho S không chứa hai phần tử mà phần tử
này chia hết cho phần tử kia và cũng không chứa hai phần tử nguyên tố cùng nhau. Hỏi S
chứa nhiều nhất bao nhiêu phần tử?

Lời giải. Xét tập hợp các số S = {1010, 1012, 1014, . . . , 2016} thì rõ ràng tất cả đều
chẵn nên không có hai số nào nguyên tố cùng nhau. Hơn thế nữa, số lớn nhất nhỏ
hơn 2 lần số nhỏ nhất nên cũng không có hai số nào mà số này chia hết cho số kia.
Khi đó, S thoả mãn đề bài và có 504 phần tử.
Ta sẽ chứng minh rằng đây chính là giá trị lớn nhất cần tìm. Giả sử ngược lại rằng tồn
tại một tập hợp S0 thoả mãn tính chất đề bài và có ít nhất 505 phần tử.
Với mỗi số chẵn i từ 1010 → 2016 với dạng i = 2α · m và (m, 2) = 1, xét tập hợp Ai có

dạng Ai = 2k · m|0 ≤ k ≤ α . Rõ ràng 2017 nguyên tố nên 2017 không thuộc S (vì
nếu không thì tất cả các số còn lại đều nguyên tố cùng nhau với nó, mâu thuẫn với
tính chất của S).
Ta sẽ chỉ ra rằng mọi số nguyên dương a không vượt quá 1008 đều thuộc về ít nhất
một tập hợp Ai nào đó nêu trên.
Thật vậy, xét các số 2a, 22 a, 23 a, . . . và giả sử rằng không có số k nào để 1010 ≤ k ≤
2016. Khi đó, phải có số m để 2m a < 1010 < 2016 < 2m+1 a , suy ra
( (
2m a ≤ 1008 2m a ≤ 1008

2m+1 a ≥ 2018 2m a ≥ 1009.

145
Điều vô lý này cho thấy nhận xét được chứng minh.
Tiếp theo, ta xét 504 tập A1009 ∪ {1011}, A1012 ∪ {1011}, . . . , A2016 ∪ {2015} thì các tập
hợp này chứa tất cả các số từ 1 đến 2016.
Vì S0 có ít nhất 505 phần tử nên phải có ít nhất hai phần tử a < b thuộc cùng một tập
nào đó trong 504 tập hợp ở trên. Ta có hai trường hợp:

• Nếu b = i + 1 và a ∈ Ai thì rõ ràng (i, i + 1) = 1 nên b nguyên tố cùng nhau với


tất cả các số thuộc Ai , dẫn đến ( a, b) = 1, không thoả.
• Nếu a, b ∈ Ai thì theo cách xây dựng tập Ai , ta sẽ có a|b, cũng không thoả.

Vậy không tồn tại tập S0 nên giá trị lớn nhất cần tìm là 504.

Nhận xét. Ta biết các kết quả quen thuộc sau:

a) Nếu chọn n + 1 số nguyên dương bất kỳ từ 2n số nguyên dương đầu tiên thì
luôn có số này chia hết cho số kia (chứng minh bằng cách xét các lũy thừa của
2). Giá trị n + 1 cũng là tốt nhất vì nếu chỉ chọn n số, ta có thể chọn các số từ
n + 1, n + 2, . . . , 2n và không có hai số nào chia hết cho nhau.
b) Nếu chọn n + 1 số nguyên dương bất kỳ từ 2n số nguyên dương đầu tiên thì
luôn có hai số nguyên tố cùng nhau (chứng minh bằng cách chia n nhóm các
cặp số lẻ và chẵn liên tiếp). Giá trị n + 1 cũng là tốt nhất vì nếu chỉ chọn n, ta có
thể chọn tất cả các số chẵn thì không có hai số nào nguyên tố cùng nhau.

Bài toán và lời giải ở trên có thể nói là một kết hợp rất thú vị, độc đáo của hai bài toán
rất đơn giản và quen thuộc vừa nêu.

Bài 3. Cho n là số tự nhiên lớn hơn 2 và X = {1, 2, 3, . . . , n}. Với mỗi song ánh f : X → X,
gọi A f là tập hợp tất cả các bộ số (i; j) sao cho i < j và f (i ) > f ( j).

a) Có bao nhiêu song ánh f thoả | A f | = 1?


b) Giả sử f là một song ánh mà A f = k > 0. Chứng minh rằng tồn tại một song ánh
n n
g : X → X sao cho | A g | = k − 1 và ∑ | f (i ) − i | > ∑ | g (i ) − i |.
i =1 i =1

(| X | chỉ số phần tử của tập hợp X).

Lời giải.

a) Với mỗi song ánh f , đặt ai = f (i ) thì rõ ràng ( a1 , a2 , . . . , an ) chính là hoán vị của
(1, 2, . . . , n).

Gọi Sn là số song ánh f có A f = 1 ứng với tập hợp n số nguyên dương đầu
tiên.

Dễ thấy với n = 3, ta có hai hoán vị thoả mãn là 1, 3, 2 và 2, 1, 3 nên S3 = 2.

Xét quan hệ giữa Sn+1 , Sn thông qua hoán vị của dãy (1, 2, . . . , n, n + 1).

146
• Nếu an+1 = n + 1 thì loại n + 1 ra, ta có một hoán vị thoả mãn đề bài của n
số nguyên dương đầu tiên, có tất cả Sn hoán vị như thế.
• Nếu an+1 6= n + 1 thì rõ ràng an = n + 1 vì nếu ai = n + 1 với i < n thì

A f ≥ 2, không thoả. Khi đó, phải có an+1 = n, an = n + 1 và ai = i với
1 ≤ i ≤ n − 1. Nghĩa là có thêm đúng 1 hoán vị nữa.

Từ đó suy ra Sn+1 = Sn + 1 và có Sn = n − 1 với mọi n. Số song ánh cần tìm là


n − 1.
b) Trong song ánh f đang xét, gọi j là số lớn nhất sao cho a j = t < j. Rõ ràng j luôn

tồn tại vì A f = k > 0. Khi đó, ta cũng có ai = i với mọi j + 1 ≤ i ≤ n.
Xét số m sao cho am = t + 1 ≤ j thì rõ ràng 1 ≤ m ≤ j. Khi đổi chỗ hai số ( a j , am )

này sẽ làm giảm giá trị A f đi đúng một đơn vị. Khi đó, giả sử ta có song ánh

g mới tương ứng với việc đổi chỗ này mà A g = k − 1 và đặt bi = g(i ). Ta sẽ
n n
chứng minh rằng T = ∑ |bi − i | − ∑ | ai − i | ≤ 0.
i =1 i =1
Thật vậy,
T = b j − m + | bm − j | − a j − j − | a m − m |
= |t + 1 − j| + |t − m| − |t − j| − |t + 1 − m|
Vì t + 1 ≤ j nên ta có

T = j − ( t + 1) − ( j − t ) + | t − m | − | t + 1 − m | = | t − m | − | t + 1 − m | − 1

Vì t − m và t + 1 − m là hai số nguyên liên tiếp nên chênh lệch của chúng đúng
bằng 1, suy ra T ≤ 1 − 1 = 0.
Do đó, hoán vị g như trên thoả mãn điều kiện đề bài. Ta có đpcm.

Bài toán kết thúc.

Nhận xét. Ta biết rằng số lượng A f được nêu trong bài chính là số nghịch thế. Nếu
gọi Sk là "độ lệch" nhỏ nhất của song ánh có k nghịch thế thì theo kết quả ở câu b, ta
có dãy Sk là một dãy không giảm.
Từ đó suy ra rằng độ lệch ở trên lớn nhất khi số nghịch thế là lớn nhất, điều này đạt
được khi hai nửa của các số 1, 2, . . . , n được đổi chỗ cho nhau.

Ở câu a, nếu yêu cầu tính số song ánh f để A f = 2 thì bài toán sẽ thú vị hơn nhiều.
Khi đó, vẫn bằng cách truy hồi tương tự, ta sẽ có được công thức là Sn = Sn−1 + n − 2
(n − 1)(n − 2)
và S3 = 1 hay Sn = .
2
Bài 4. Cho tam giác ABC nhọn nội tiếp đường tròn (O) và điểm D di động trên cung BC chứa
điểm A (D 6= A). Trên AB, AC lần lượt lấy các điểm M, N sao cho MD = MB, NC = ND.

a) Chứng minh đường cao DH trong tam giác DMN luôn đi qua một điểm cố định.

147
b) DM, DN theo thứ tự cắt (O) tại E, F ( E, F khác D). Chứng minh các đường tròn
ngoại tiếp các tam giác EMB, FNC cắt nhau tại điểm K thuộc đường thẳng BC và
đường cao KI của tam giác KMN luôn qua một điểm cố định.

Lời giải.

a) Cách 1. Giả sử DH cắt (O) tại X. Ta có

∠ AMD = 2∠ ABD = 2∠ ACD = ∠ AND,

suy ra tứ giác AMND nội tiếp. Suy ra ∠ DMN = ∠ DAC = ∠ DBC. Ta có

∠ BDX = ∠ HDM − ∠ MDB = 90◦ − ∠ HMD − ∠ MDB


= 90◦ − ∠ MBD − ∠ DBC = 90◦ − ∠ ABC.

Suy ra AX ⊥ BC nên X cố định. Ta có đpcm.

Cách 2. Gọi Q là giao điểm của ( M; MB) và ( N; NC ). Ta có

∠ MQB = ∠ MBQ, ∠ NQC = ∠ NCQ, ∠ MQN = ∠ MDN = ∠ MAN.

Suy ra ∠ MQB + ∠ NQC + ∠ MQN = 180◦ nên B, Q, C thẳng hàng.

148
Giả sử DQ cắt (O) tại X. Khi đó

∠ BMQ
XB sin ∠ BDX sin cos ∠ ABC
= = 2 =
XC sin ∠CDX CNQ cos ∠ ACB
sin
2

không đổi, nên X cố định.


b) Ta có ∠ MEB = 180◦ − ∠ BCD = 180◦ − ∠ BOM, suy ra OMEB nội tiếp.

Tương tự CNOF nội tiếp. Khi đó (OMB) và (OCN ) cắt nhau tại K thuộc BC.

Ta có

∠OMK = ∠OBC, ∠∠ MKN = ∠ MKO + ∠ NKO = ∠OBA + ∠OCA = ∠ BAC.

Suy ra ∠OMK + ∠ MKN = ∠ BAC + ∠OBC = 90◦ , suy ra MO⊥KN. Chứng


minh tương tự thì ON ⊥KM. Do đó O là trực tâm tam giác KMN.

Vậy KO⊥ MN, từ đó KI qua O cố định.

Bài toán kết thúc.

Bài 5. Với mỗi số nguyên n ∈ / {−1, 0, 1}, ký hiệu p(n) là ước nguyên tố lớn nhất của n. Gọi
F là tập hợp tất cả các đa thức f ( x ) có hệ số nguyên và thoả mãn

f (n + p(n)) = n + p( f (n))

với mọi số nguyên n > 2017 và f (n) ∈


/ {−1, 0, 1}.

a) Tìm tất cả các đa thức bậc nhất thuộc F.


b) F có bao nhiêu phần tử?

Lời giải. Trước hết, ta thấy rằng các đa thức hằng f (n) ≡ 0, f (n) ≡ 1, f (n) ≡ −1
thoả mãn bài toán vì không có ràng buộc ở đề bài cho các giá trị này.
Ta thấy rằng có hữu hạn giá trị n > 2017 để f (n) ∈ {0, 1, −1} nên có vô hạn giá trị
n > 2017 để f (n) ∈ / {0, 1, −1}, vì thế nên trong lập luận bên dưới, ta chỉ xét các giá
trị n để f (n) ∈
/ {0, 1, −1}.
Ta có nhận xét rằng với mọi n nguyên và |n| ≥ 2 thì 2 ≤ p(n) ≤ |n| và đẳng thức ở
bất đẳng thức thứ hai xảy ra khi và chỉ khi |n| nguyên tố.
Trong đẳng thức đã cho, thay n = q > 2017 là số nguyên tố, ta có

f (q + p(q)) = q + p( f (q)) hay f (2q) = q + p( f (q)). (∗)

149
Suy ra f (2q) > q với mọi q > 2017 là số nguyên tố. Do đó, f (n) không thể là đa thức
hằng (khác 0, 1, −1) và trong trường hợp deg f > 0, bậc cao nhất của f cũng không
thể âm (nếu không thì tồn tại q đủ lớn để f (2q) < 0, mâu thuẫn.
Cũng từ đẳng thức trên, ta có f (2q) ≤ q + f (q) với mọi q nguyên tố và q > 2017.

• Nếu deg f = 1, đặt f (n) = an + b với a 6= 0. Bất đẳng thức ở trên đưa về

a(2q) + b ≤ q + aq + b ⇔ aq ≤ q ⇔ a ≤ 1.

Theo nhận xét ở trên thì a > 0 nên a = 1. Thay vào (∗), ta có

2q + b = q + p(q + b) ⇔ q + b = p(q + b).

Từ đây suy ra q + b là số nguyên tố. Chú ý rằng ta chọn được q đủ lớn để


q + b > 2017 nên áp dụng lập luận trên khi thay q bởi q + b, ta có q + 2b cũng
phải là số nguyên tố.

Tương tự như thế, suy ra q + kb là số nguyên tố với k nguyên dương bất kỳ.
Chọn k = q, ta có q + qb = q(b + 1) phải là số nguyên tố. Điều này chỉ xảy ra
khi b = 0.

Do đó, f (n) = n. Thử lại ta thấy thoả mãn.


• Nếu deg f = k ≥ 2, theo chứng minh ở trên thì f (2q) ≤ q + f (q) và hệ số bậc
cao nhất a > 0. Chia hai vế cho (2q)k rồi tính lim q → +∞, ta thấy vế trái tiến
tới a trong khi vế phải tiến tới 0.

Suy ra a ≤ 0, điều mâu thuẫn này cho thấy không tồn tại đa thức bậc cao hơn 1
thoả mãn đề bài.

Vậy có tất cả 4 đa thức thoả mãn đề bài là f (n) ≡ 0, 1, −1 và f (n) = n thoả mãn nên
| F | = 4.

Nhận xét. Một bài toán tương tự với bài toán trên nhưng xét ước nguyên tố nhỏ nhất:
/ {0, 1, −1}. Xác định tất cả các đa
Gọi d(n) là ước nguyên tố nhỏ nhất của số nguyên n ∈
thức P( x ) hệ số nguyên sao cho

P(n + d(n)) = n + d( P(n)).

Việc thêm yêu cầu với mọi số nguyên n > 2017 vừa là một ràng buộc, vừa là một
gợi ý khi đòi hỏi học sinh chỉ xét các số nguyên đủ lớn. Một bài toán đa thức số
học có ý tưởng tương tự khi xét đa thức bậc nhất và chứng minh đa thức bậc từ 2
trở lên không thoả có xuất hiện trong đề VN TST 2014 như sau: Cho dãy số nguyên
u1 = 1, un+1 = P(un ), n ≥ 1 với P( x ) là một đa thức có hệ số nguyên. Biết rằng mọi số
nguyên dương là ước của một số hạng nào đó của dãy. Chứng minh rằng P(n) = n + 1.

150
Bài 6. Với mỗi số tự nhiên n, ký hiệu T (1 + n, 3 + n, 4 + n) là tập hợp tất cả các bộ ( a, b, c)
với a, b, c là các số tự nhiên thoả 1 ≤ a ≤ 1 + n, a + 1 ≤ b ≤ 3 + n, b + 1 ≤ c ≤ 4 + n. Gọi
an là số phần tử của T (1 + n, 3 + n, 4 + n).

a) Tính a4 .
b) Tìm tất cả n sao cho an chia hết cho 3.

Lời giải.

a) Xét một bộ ( a, b, c) gồm các số nguyên dương thoả mãn 1 ≤ a < b < c ≤ n + 4.
Rõ ràng có tất cả Cn3 +4 bộ như thế.

Ta có hai trường hợp:

• Nếu a = n + 2 thì rõ ràng b = n + 3, c = n + 4 và chỉ có duy nhất một bộ


( a, b, c) như thế. Bộ này không thoả mãn điều kiện của bài toán.
• Nếu a ≤ n + 1, dĩ nhiên ta cũng có b ≤ n + 3, c ≤ n + 4 và đây là tất cả các
bộ thoả mãn đề bài.

Do đó, số các bộ thoả mãn đề bài là an = Cn3 +4 − 1.

Suy ra a4 = C83 − 1 = 55.


b) Ta có

(n + 4)(n + 3)(n + 2) (n + 1)(n2 + 8n + 18) n(n + 1)(n + 8)


an = −1 = = + 3( n + 1).
6 6 6

Vì 3|3(n + 1) nên để an chia hết cho 3 thì n(n + 1)(n + 8) phải chia hết cho 9. Ta
thấy rằng không có hai số nào trong ba số n, n + 1, n + 8 có thể cùng chia hết
cho 3 nên điều kiện trên chỉ xảy ra khi 9|n hoặc 9|n + 1 hoặc 9|n + 8.

Vậy tất cả các số n cần tìm để 3| an là n chia 9 dư 0, 1, 8.

Bài toán kết thúc.

Nhận xét. Nếu nhận ra được sự tương ứng giữa T (1 + n, 3 + n, 4 + n) trong đề bài
với tổ hợp các số nguyên dương phân biệt không vượt quá n + 4 thì bài toán có thể
giải quyết đơn giản như trên. Nếu không, ta có thể dùng nguyên lý bù trừ, đưa về
công thức truy hồi dạng

(n + 2)(n + 3)
a0 = 3, an = an−1 + , n ≥ 0.
2

Bài toán tương tự đã từng xuất hiện trong đề chọn đội tuyển của PTNK năm trước:
Với các số nguyên a, b, c, d thoả 1 ≤ a < b < c < d, ký hiệu

T ( a, b, c, d) = {( x, y, z, t) ∈ N | 1 ≤ x < y < z < t, x ≤ a, y ≤ b, z ≤ c, t ≤ d} .

151
a) Tính số phần tử của T (1, 4, 6, 7).
b) Cho a = 1 và b ≥ 4. Gọi d1 là số phần tử của T ( a, b, c, d) chứa 1 nhưng không chứa
2, d2 là số phần tử chứa 1, 2 nhưng không chứa 3, d3 là số phần tử chứa 1, 2, 3 nhưng
không chứa 4. Chứng minh rằng d1 ≥ 2d2 − d3 . Dấu đẳng thức xảy ra khi nào?

Bài 7. An và Bình luân phiên đánh dấu các ô vuông của hình vuông 101 × 101 ô. An là
người bắt đầu. Một ô sẽ không thể được đánh dấu nếu trên cùng hàng với nó hoặc cùng cột
với nó đã có ít nhất 2 ô được đánh dấu. Ai không đi được nữa sẽ thua. Hãy xác định ai là
người có chiến thuật thắng.

Lời giải.
Cách 1. Để đơn giản, ta gọi hai người chơi là A và B thay vì An và Bình. Ta sẽ chứng
minh rằng B có chiến thuật để thắng. Điều này cũng đúng khi thay 101 bằng một số
nguyên dương n ≥ 2 bất kỳ.
Rõ ràng theo luật chơi thì có không quá 2n ô được đánh dấu. Vì thế nên chiến thuật ở
đây là B sẽ tìm cách đánh được ô cuối cùng.
Chiến thuật là: người đi trước đánh ô nào thì người sau sẽ đánh một ô bất kỳ cùng
dòng với nó sao cho số cột được đánh là nhiều nhất có thể.
Đặc điểm của chiến thuật này là:
• Sau mỗi lượt của A, B thì có thêm một hàng có hai ô được đánh; không có hàng
nào chứa một ô.
• Sau mỗi lượt của A thì trên hàng mà A vừa đánh, còn đúng n − 1 ô chưa được
đánh và B sẽ đánh tuỳ ý vào ô thuộc cột chưa có ô nào được đánh; nếu như tất
cả các cột đều có ô được đánh thì B sẽ chọn cột tuỳ ý mà chỉ có 1 ô được đánh
trên đó.
Bằng cách đó, trong n − 1 lượt đầu tiên, một khi A còn đi được thì B vẫn đi được vì
vẫn luôn còn hàng trống và các cột vẫn chưa đầy 2 ô. Do đó, sau n − 1 lượt của A, B,
bảng còn lại sẽ có đặc điểm là:
• Chỉ còn một hàng duy nhất mà chưa có ô nào được đánh, n − 1 hàng kia đều có
ô đã được đánh.
• Tất cả các cột đều có ô được đánh (có cột có 1 ô, có cột có 2 ô).
Tổng số ô đã đánh là 2n − 2 nên gọi a, b lần lượt là số cột có 1 ô, 2 ô được đánh thì

a + b = n và a + 2b = 2n − 2.

Suy ra a = 2, b = n − 2, nghĩa là có đúng hai cột mà trên đó có 1 ô được đánh. Hai ô


nằm ở vị trí giao giữa hai cột đó và dòng chưa được đánh là hai vị trí cuối cùng có
thể đánh. A, B thay phiên đánh vào hai ô đó và B là người đánh cuối cùng nên chiến
thắng.
Cách 2. Ta có nhận xét rằng:

152
a) Nếu ở lượt đầu tiên, An đánh vào ô (1, 51) giữa hàng 1 thì Bình sẽ đánh vào
ô (101, 51) giữa hàng cuối. Khi đó, hai bạn sẽ không thể điền vào ô trung tâm
nữa. Tiếp theo, mỗi khi An đánh dấu ô nào thì Bình sẽ đánh dấu vào ô đối xứng
với ô đó qua tâm. Mỗi khi An điền được thì Bình cũng điền được, vậy Bình sẽ
thắng.
b) Nếu ở lượt đầu tiên, An đánh vào ô ( a, b) thì ta có thể đưa về nhận xét 1 bằng
cách xây dựng một bảng tương ứng cùng kích thước nhưng hàng 1 và hàng a,
cột 51 và cột b đổi chỗ cho nhau. Khi đó, An đánh vào ô nào thì Bình sẽ đánh
vào ô ở chỉ số hàng/cột tương ứng ở bảng đối chiếu; Bình sẽ đánh vào ô đối
xứng như ở chiến lược 1 rồi đánh vào ô có cùng chỉ số hàng/cột ở bảng gốc. Dễ
thấy rằng đánh dấu được của mỗi hàng và cột ở hai bảng là giống nhau.

Hình minh họa cho trường hợp 5 × 5 khi An đánh vào ô (5, 5) trong nước đi đầu tiên.
Từ hai nhận xét trên, ta thấy Bình là người có chiến lược thắng trò chơi.

Nhận xét. Rõ ràng theo cách thứ nhất thì bài toán không chỉ đúng với số 101 hay các
số lẻ mà còn đúng với bảng vuông có kích thước bất kỳ không nhỏ hơn 2. Để cảm
nhận và đoán được ai là người có chiến lược thắng cuộc, ta có thể thử với trường hợp
n = 2, n = 3. Khi n = 2, ta thấy tất cả ô vuông đều có thể được đánh dấu nên Bình
hiển nhiên thắng. Khi n = 3, ta thấy ô ở trung tâm có vai trò rất quan trọng cho chiến
lược chơi. Chú ý rằng vẫn còn nhiều chiến lược khác thú vị hơn cho Bình để thắng trò
chơi này.

Bài 8. Đường tròn (O) nội tiếp tứ giác ABCD và tiếp xúc với các cạnh AB, BC, CD, DA
lần lượt tại E, F, G, H. Gọi I, J là trung điểm của AC, BD và IB, ID, J A, JC theo thứ tự cắt
EF, GH, HE, FG tại M, N, P, Q.

a) Chứng minh rằng I J, MN, PQ đồng quy (tại điểm S).


b) Các tia đối của các tia J A, IB, JC, ID lần lượt cắt (O) tại các điểm A0 , B0 , C 0 , D 0 . Giả
sử A0 C 0 , B0 D 0 lần lượt cắt PQ, MN tại U, V. Gọi K là hình chiếu của S trên UV.
Chứng minh rằng ∠ AKB = ∠CKD.

Lời giải.

a) Trước hết, sử dụng định lý Menelaus trong các tam giác ABC và ADC, ta thấy
rằng các đường thẳng GH và EF cùng cắt AC tại một điểm, đặt là U. Tương tự,
các đường thẳng EH, FG cùng cắt BD tại một điểm, đặt là V.

153
Tiếp theo, áp dụng định lý Desargue cho hai tam giác AHP và CGQ ta có
GH; AC; PQ đồng quy tại U. Tương tự thì HE, MN, BD đồng quy tại V.

Áp dụng định lý Menelaus cho tam giác AIU thì

DN IU AH DN AU DH
· · = 1 nên = · .
I N AU DH IN IU AH

Gọi S0 = MN ∩ I J thì

IS0 V J ND IS0 N I VD H A U I VD
· · = 1 nên = · = · · (7)
JS VD N I
0 JS 0 ND V J HD U A V J

AP VD AH
Tương tự, ta cũng có = · . Gọi S00 là giao điểm của PQ và I J thì
PJ V J HD

IS00 U I AP U I VD AH
= · = · · (8)
JS 00 US PJ U A V J HD

Từ (7) và (8) suy ra S0 ≡ S00 .

154
Vậy PQ, MN, I J đồng quy tại một điểm đặt là S. Ta có đpcm.
b) Bước 1. Chứng minh S trùng O. Theo câu a thì

IS U I VD AH
= . . (9)
JS U A V J HD

Theo định lý đường thẳng Newton, ta có I, O, J thẳng hàng và

IO CF + AE
= .
OJ BE + DH

AE BF UC AE · UC
Ta có · · = 1, suy ra CF = . Suy ra
BE CF U A UA

UC AE(UC + U A) AE · U I
AE + CF = AE(1 + )= =2 .
UA UA UA
2DH · V J
Tương tự thì BF + DH = . Khi đó
VD
OI 2 · AE · U I 2 · DH · V J U I VD AH
= : = . . (10)
OJ UA VD U A V J DH

Bước 2. A0 C 0 , PQ, AC đồng quy tại U và B0 D 0 ; MN; BD đồng quy tại V.

Thật vậy, xét trong đường tròn (O), vì GH là đường đối cực của D và đi qua U
nên đường đối cực của U đi qua D. Tương tự thì đường đối cực của U đi qua
B nên BD là đường đối cực của U. Suy ra J ( RU, C 0 A0 ) = −1, suy ra U, C 0 , A0
thẳng hàng.

Do đó A0 C 0 , PQ, AC đồng quy. Tương tự với B0 D 0 ; MN; BD.


Ta cũng chú ý rằng HF, GE, AC, BD đồng quy tại R (theo định lý Brianchon suy
biến từ lục giác ngoại tiếp thành tứ giác ngoại tiếp).
Bước 3. Theo định lý Brocard thì OR⊥UV tại K.

Ta có V (UR, VF ) = −1, suy ra B(UR, EF ) = −1, suy ra (U, R, A, C ) = −1. Do


đó K (UR, AC ) = −1.

Mà KU ⊥KR nên KR là phân giác góc ∠ AKC. Chứng minh tương tự, ta có KR
là phân giác ∠ BKD.

Vậy ∠ AKB = ∠CKD.

Bài toán được giải quyết hoàn toàn.

Nhận xét. Câu a của bài toán có thể chứng minh không dùng tính toán mà bằng cực -
đối cực như sau: Qua A dựng đường thẳng song song với BD thì A( DB, Jd) = −1

155
(vì J là trung điểm BD) nên A( HE, Pd) = −1. Gọi X là giao điểm của HE và d thì
( HE, PX ) = −1 nên X thuộc đường đối cực của P đối với (O).
Vì P thuộc đối cực của A nên A thuộc đối cực của P, đó chính là d. Suy ra OP ⊥ d hay
OP ⊥ BD, vì thế nên U, P, O thẳng hàng. Tương tự thì U, Q, O cũng thẳng hàng nên
PQ đi qua O. Chứng minh tương tự ta cũng có MN đi qua O nên MN, PQ, I J đồng
quy tại O.
Một điều thú vị của câu a là nếu các điểm I, J thay đổi tuỳ ý trên các đoạn AC, BD
và các điểm M, N, P, Q được định nghĩa tương tự thì bài toán vẫn đúng. Tuy nhiên,
điểm đồng quy đó không hẵn là O. Câu b là kết hợp của mô hình tứ giác ngoại tiếp
cùng tứ giác toàn phần nội tiếp EFGHUV. Tuy nhiên, nó đòi hỏi phải xử lý khéo léo
các chùm điều hòa nên thực sự là một thử thách đáng kể. Đây có thể nói là bài toán
khó nhất trong kỳ thi chọn đội tuyển này.

2.3. Đề thi năm 2018


Bài 1. Cho số nguyên dương a > 1. Tìm giá trị lớn nhất của số thực d sao cho tồn tại một
cấp số cộng có công sai d và số hạng đầu là a và có đúng hai số trong các số a2 , a3 , a4 , a5 là
các số hạng của cấp số cộng đó.

Lời giải. Do cần tìm giá trị lớn nhất của d nên ta chỉ cần quan tâm trường hợp d > 0.
Rõ ràng b thuộc cấp số cộng công sai d và số hạng đầu là u1 = a khi và chỉ khi
b−a
∈ Z+ .
d
Điều này có nghĩa là cần tìm d sao cho trong các số a2 − a, a3 − a, a4 − a, a5 − a chỉ có
đúng hai số chia cho d thu được số nguyên.
Chú ý rằng a2 − a là ước của các số còn lại nên số này không được thuộc cấp số cộng.
Ngoài ra, a3 − a | a5 − a nên hai số xuất hiện không thể là a3 − a, a4 − a. Ta có hai
trường hợp:
• Nếu a3 − a, a5 − a thuộc cấp số cộng. Khi đó d ≤ a3 a và với d = a3 − a thì

a4 − a a4 − a a2 + a + 1 1
= 3 = = a+ / Z.

d a −a a+1 a+1

Do đó, trong trường hợp này, dmax = a3 − a.


• Nếu a4 − a, a5 − a thuộc cấp số cộng. Chú ý rằng a5 − a = a( a4 − a) + ( a2 − a)
nên
a2 − a a5 − a a ( a4 − a )
= + ∈ Z,
d d d
kéo theo a2 − a cũng thuộc cấp số cộng, mâu thuẫn với điều kiện đề bài.
Vậy giá trị lớn nhất cần tìm của d là a3 − a.
Nhận xét. Ở bài này, nhận xét đầu tiên mang tính quyết định cho bài toán. Nếu lan
man vào việc xét trường hợp ngay từ đầu thì rất khó đi đến kết quả. Bài toán chỉ dùng
các kết quả nhẹ nhàng về chia hết nhưng cách đặt câu hỏi rất hay.

156
Một số bài toán tương tự:

a) (Trường xuân 2013) Một cấp số cộng gồm các số nguyên dương được gọi là
“chuẩn” nếu nó có ít nhất ba số hạng và tích các số hạng của nó là ước số của
một số có dạng n2 + 1.

a) Chứng minh rằng không tồn tại cấp số cộng chuẩn có công sai là d =
10, d = 11.
b) Chứng minh rằng tồn tại cấp số cộng có d = 12. Hỏi cấp số đó có nhiều
nhất mấy số hạng?
√ √ √
b) Chứng minh rằng 2, 5, 7 không thể là các số hạng của cùng một cấp số
cộng.
c) (KHTN 2011) Chứng minh rằng không tồn tại các số nguyên dương thoả b2 −
a2 = c2 − b2 = d2 − c2 .

Bài 2. Cho n số thực x1 , x2 , ..., xn . Với mỗi i ∈ {1, 2, 3, . . . , n}, gọi ai là số các chỉ số j sao

cho xi − x j ≤ 1 và bi là số các chỉ số j sao cho xi − x j ≤ 2 (các số i, j có thể bằng nhau).

a) Chứng minh rằng tồn tại i mà bi ≤ 3ai .



b) Gọi A là số cặp (i, j) có thứ tự mà xi − x j ≤ 1 và B là số cặp (i, j) có thứ tự mà

xi − x j ≤ 2 (các số i, j có thể bằng nhau). Chứng minh rằng B ≤ 3A.

Lời giải.

a) Không mất tính tổng quát, giả sử các số đã cho được sắp xếp tăng dần. Xét
k = max { a1 , a2 , . . . , an } và giả sử ai = k, khi đó tồn tại k số trong dãy là

xu ≤ xu+1 ≤ . . . ≤ xi ≤ . . . ≤ xv với | xu − xi | , | xv − xi | ≤ 1.

Ngoài ra vì tính lớn nhất của k nên | xu−1 − xi | > 1, | xv+1 − xi | > 1.

Trong [ xu , xv ], có đúng k số j để x j − xi ≤ 1 < 2; còn trước xu , xét hai số xr , xs
trong đó sao cho xr ≤ xs và | xr − xi | ≤ 2, | xs − xi | ≤ 2 thì

| xr − x s | = x s − xr = ( xi − xr ) − ( xi − x s ) < 2 − 1 = 1

nên sẽ có không quá k số j để x j − xi ≤ 2 vì nếu ngược lại, sẽ có nhiều hơn k
số liên tiếp trong dãy cách nhau không quá 1 đơn vị, mâu thuẫn với tính lớn
nhất của k. Tương tự với các số sau xv , vì thế nên bi ≤ 3k ⇒ bi ≤ 3ai .
b) Ta sẽ chứng minh bằng quy nạp theo n.
Với n = 1 rõ ràng A = B = 1 nên khẳng định hiển nhiên đúng.
Giả sử kết quả đúng với n ≥ 1, ta sẽ chứng minh nó cũng đúng với n + 1.
Xét dãy số thực T = ( x1 , x2 , . . . , xn+1 ) bất kỳ và giả sử x1 ≤ x2 ≤ . . . ≤ xn+1 , ký
hiệu A T , BT là số cặp có thứ tự các chỉ số (i, j) tương ứng với định nghĩa của đề

157
bài. Giả sử k ≥ 1 là số lượng lớn nhất các số của T được chứa trong một đoạn
độ dài bằng 2 nào đó.
Gọi xi là số cuối cùng của dãy mà trong đoạn [ xi − 1, xi + 1] có chứa đúng k số
(kể cả xi ). Gọi T 0 là dãy mới sau khi bỏ xi đi. Khi đó, số lượng các số thuộc T 0 có
trong [ xi − 1, xi + 1] là k − 1, ngoài ra xi đã bị bỏ đi thuộc về đúng 2k − 1 cặp
của A T (gồm k − 1 số ∗ thuộc đoạn trên tạo thành các cặp có dạng ( xi , ∗), (∗, xi )
cùng với ( xi , xi )). Do đó
A T = A T 0 + 2k − 1.
Ta viết [ xi − 2; xi + 2] = [ xi − 2; xi − 1] ∪ [ xi − 1; xi + 1] ∪ [ xi + 1; xi + 2], trừ
đoạn ở giữa thì hai đoạn đầu và cuối chứa tối đa k phần tử của T. Hơn nữa,
do định nghĩa số xi nên trong đoạn [ xi + 1; xi + 2] có tối đa k − 1 phần tử của
T. Suy ra có tối đa 2(k − 1) + k = 3k − 2 phần tử của T (không tính xi ) thuộc
[ xi − 2; xi + 2], suy ra

BT ≤ 2(3k − 2) + 1 + BT 0 = 3(2k − 1) + BT 0 .

Áp dụng giả thiết quy nạp, ta có BT 0 < 3A T 0 nên từ các điều trên, suy ra

BT ≤ 3(2k − 1) + BT 0 < 3(2k − 1) + 3A T 0 = 3( A T 0 + 2k − 1) = 3A T .

Theo nguyên lý quy nạp, ta có đpcm.


Bài toán kết thúc.

Nhận xét. Ta thử xét một số ước lượng giữa A, B khi các số x1 , x2 , . . . , xn đặc biệt.
• Nếu n số bằng nhau thì ai = bi = n nên A = B.
• Nếu n số chẵn liên tiếp thì ai = 1, ∀i = 1, 2, . . . , n; còn b1 = bn = 2, bi = 3, ∀i =
B 2
2, . . . , n − 1 nên A = n, còn B = 3n − 2 và rõ ràng = 3 − → 3 chứng tỏ 3 là
A n
hằng số tốt nhất.
Bằng phương pháp tương tự trên, ta cũng có thể chứng minh được rằng với 0 <

a < b thì đặt số cặp có thứ tự (i, j) thoả xi − x j ≤ a và số cặp có thứ tự (i, j) thoả

xi − x j ≤ b lần lượt là A, B thì sẽ có A < (2k − 1) B với k = db/ae .

Bài 3. Cho p là số tự nhiên. Xét phương trình x3 + x + p = y2 trên tập số nguyên dương.
a) Tìm số nguyên tố p nhỏ nhất dạng 4k + 1 sao cho phương trình có nghiệm.
b) Chứng minh rằng nếu p là số chính phương thì phương trình luôn có nghiệm.

Lời giải.
a) Các số nguyên tố có dạng 4k + 1 là 5, 13, 17, ...
Trước hết, ta thấy với p = 13 thì x3 + x + 13 = y2 có nghiệm là ( x; y) = (4; 9).
Ta cần chứng minh x3 + x + 13 = y2 không có nghiệm nguyên. Xét (mod 4).

158
• Khi x chia 4 dư 0, 1, 2, 3, vế trái chia 4 dư 1, 3, 3, 3.
• Khi y chia 4 dư 0, 1, 2, 3, vế phải chia 4 dư 0, 1, 0, 1.

Do đó, y phải lẻ và x phải chia hết cho 4.


Ta viết biểu thức đã cho thành x3 + x + 30 = y2 + 25 hay ( x + 3)( x2 − 3x + 10) =
y2 + 52 .
Do x + 3 chia 4 dư 3 nên nó có ước nguyên tố dạng q = 4k + 3. Ta biết rằng với
a, b ∈ Z thì a2 + b2 chia hết cho số nguyên tố q = 4k + 3 khi và chỉ khi q | a, q | b,
nên từ trên ta suy được q | 5, vô lý. Vậy nên phương trình trên vô nghiệm và
p = 13 là số nguyên tố nhỏ nhất cần tìm.
b) Trước hết xin phát biểu không chứng minh bổ đề sau:

Bổ đề. Với các số nguyên dương a, b, c, d thoả mãn ab = cd thì tồn tại các số nguyên
x, y, z, t sao cho a = xy, b = zt, c = xz, d = yt.

Quay lại bài toán. Do p là số chính phương nên đặt p = a2 , a ∈ Z. Ta viết lại
phương trình thành

x3 + x + a2 = y2 ⇔ x ( x2 + 1) = (y − a)(y + a).

Áp dụng bổ đề vào bài toán, ta thấy tồn tại các số m, n, p, q nguyên dương để
x = mn; x2 + 1 = pq; y + a = mp; y − a = nq. Suy ra

(mn)2 + 1 = pq và mp − nq = 2a.

Ta biết rằng dãy số sai phân tuyến tính có dạng un+2 = αun+1 + βun , khi
đặt vn = u2n+1 − un un+2 thì vn là cấp số nhân công bội − β (thật vậy, vn =
un+1 (αun + βun−1 ) − un (αun+1 + βun ) = − β(u2n − un+1 un−1 ) = − βvn−1 ).
Ta xét dãy tuyến tính u0 = 0, u1 = 1, un+2 = αun+1 + un thì rõ ràng với mọi n
thì
u2n − un+1 un−1 = (−1)n−1 (u21 − u2 u0 ) = (−1)n−1 .
Khi đó, với mọi n chẵn thì u2n − un+1 un−1 = −1 nên có thể chọn mn = u2k của
dãy. Ta có

u2 = α, u3 = α2 + 1, u4 = α(α2 + 2), u5 = α4 + 3α2 + 1.

Chọn p = u3 , q = u5 , mn = u4 thì rõ ràng (mn)2 + 1 = pq.


Ta cần có

mu3 − nu5 = 2a ⇔ m(α2 + 1) − n(α4 + 3α2 + 1) = 2a.

159
Chọn tiếp α = 4a2 và viết m = 2a(α2 + 2), n = 2a(α4 + 3α2 + 1) thì đẳng thức
trên đúng vì
(α2 + 1)(α2 + 2) − (α4 + 3α2 + 1) = 1.
Từ đó ta chỉ được nghiệm cụ thể của phương trình là
  √
( x; y) = 4a2 (16a4 + 2); 2a(16a4 + 2)(16a4 + 1) − a với a = p ∈ Z+ .

Bài toán kết thúc.

Nhận xét. Ở câu a, việc ứng dụng bổ đề trên là khá tự nhiên. Bổ đề là hệ quả quen
thuộc của định lý Fermat nhỏ. Một kết quả tương tự đối với số nguyên tố có dạng
3k + 2 là a2 + ab + b2 chia hết cho p khi và chỉ khi a, b cùng chia hết cho p. Một số bài
toán tương tự:

a) (Euler) Chứng minh rằng phương trình sau không có nghiệm nguyên: y2 =
x3 + 7.
b) (USA TST 2008) Chứng minh rằng phương trình x2 = n7 + 7 không có nghiệm
nguyên.
c) A = 3 p + p − 4 với p là số nguyên tố thì A không phải là số chính phương, cũng
không là tích của hai số tự nhiên liên tiếp.

Ở câu b, việc dùng bổ đề giúp ta chuyển từ bài toán “tồn tại hai số” sang “tồn tại bốn
số”, khả năng chọn được sẽ cao hơn. Một số bài toán tương tự:

a) (Hàn Quốc TST, 2012) Cho số nguyên dương n. Chứng minh rằng tồn tại vô
hạn bộ ba các số nguyên dương ( x, y, z) sao cho nx2 + y3 = z3 và x, y, z đôi một
nguyên tố cùng nhau.
b) (Ấn Độ) Tìm tất cả các nghiệm nguyên của phương trình x3 + ( x + 4)2 = y2 .
c) Chứng minh rằng với mọi số nguyên dương m thì luôn tồn tại x, y ∈ Z+ sao cho
x3 + 4x + m2 = y2 .

Bài 4. Cho tam giác ABC nhọn nội tiếp trong đường tròn có B, C cố định và A di động trên
(O). Gọi D là trung điểm BC. Trên AB lấy điểm M, P và trên AC lấy các điểm N, Q sao cho
DA = DP = DQ, DM⊥ AC, DN ⊥ AB.

a) Chứng minh rằng các điểm M, N, P, Q cùng thuộc một đường tròn, đặt là (C ) và
đường tròn này luôn đi qua một điểm cố định.
b) Chứng minh rằng tâm của (C ) luôn di chuyển trên một đường tròn cố định.

Lời giải.

a) Dễ thấy tam giác AMQ cân tại M nên

180◦ − 2∠ A 180◦ − ∠ PDQ


∠ DMQ = ∠ DMA = 90◦ − ∠ A = = = ∠ DPQ,
2 2

160
suy ra tứ giác MPDQ nội tiếp. Chứng minh tương tự, ta có tứ giác QNDP nội
tiếp nên suy ra M, N, P, Q cùng nằm trên một đường tròn đi qua điểm D cố
định.
b) Gọi I là tâm MNPQ và Z, T lần lượt là giao điểm của DN, DM với AB, AC. Giả
sử OD cắt MN ở J ta sẽ chứng minh rằng J là trung điểm MN. Thật vậy, gọi
R là điểm đối xứng của N qua D thì MN ⊥ DR và BRCN là hình bình hành,
kéo theo PR k AC mà DM ⊥ AC nên BR ⊥ DM. Suy ra B là trực tâm tam giác
DMR. Do đó BD ⊥ MR ⇒ MR k DJ, kéo theo J là trung điểm MN. Lại có

1 1 1
DJ = RM = · ( BD · tan ∠ MDR) = BD · tan A = BC · tan A
2 2 2

nên suy ra J là điểm cố định.


Gọi X, Y lần lượt là trung điểm AD, DJ thì rõ ràng Y cũng là điểm cố định. Ta
sẽ chứng minh rằng DXI J là hình bình hành.
Dễ thấy J là trung điểm MN cũng là tâm đường tròn ngoại tiếp tứ giác MZTN;
còn X là tâm ( AZT ) nên JX ⊥ ZT mà ∠ ATZ = ∠ AN M = ∠ APQ nên ZT k
PQ, dẫn đến JX ⊥ PQ.
Lại có D là tâm ( APQ) và I là tâm ( DPQ) nên DI ⊥ PQ, kéo theo JX k ID.
Ngoài ra, vì D là trực tâm tam giác AMN nên AD ⊥ MN, mà I J ⊥ MN suy
ra I J k AD. Từ đó ta có tứ giác DX J I là hình bình hành, dẫn đến X, I đối xứng
nhau qua Y. Mà điểm X chạy trên đường tròn (ω ) là ảnh của (O) qua phép vị
1
tự tâm D tỉ số cố định nên I cũng chạy trên đường tròn cố định là ảnh của
2
(ω ) qua phép đối xứng tâm I.
Bài toán kết thúc.

Nhận xét. Bài toán trên còn nhiều cách xử lý khác, nhưng ở đây giới một cách thuần
túy nhất. Chẳng hạn, để có J là trung điểm MN ta có thể dùng định lý con bướm cho
đường tròn ( MNZT ). Ta cũng nhận thấy AJ đẳng giác với AD trong góc ∠ BAC nên
AJ đi qua giao điểm T của hai tiếp tuyến kẻ từ B, C của (O). Khi đó AMTN nội tiếp
đường tròn đường kính AT với DMTN là hình bình hành,. . . Nói chung các mô hình
liên kết nhau rất thú vị.
Thực ra ở điểm này, ta có thể thấy bài toán có liên hệ với bài 4 của đề Sharygin khối 9
năm 2018: Cho tam giác ABC nội tiếp đường tròn (O) có BC cố định và A di động.
Gọi H là trực tâm tam giác và D ∈ AB, E ∈ AC sao cho H là trung điểm DE Chứng
minh rằng tâm của ( ADE) thuộc một đường tròn cố định khi A di động.
Bài toán 4 là phát biểu ngược lại của bài toán Sharygin trên. Cuối cùng, chúng tôi
xin giới thiệu lời giải ngắn gọn của BTC Sharygin để bạn đọc tham khảo: Giả sử P, Q
là trung điểm AD, AE và trung trực AD, AE cắt BC ở R, S. Gọi O0 là tâm ( ADE) thì
1
∠ RO0 S = 180◦ − ∠ A cố định. Vì ∠ BHP = 90◦ ⇒ BP = BH · , mà H di chuyển
cos A
trên đường tròn (ω ) cố định nên P cũng di chuyển trên đường tròn cố định (ω )0 ,

161
1
là ảnh của (ω ) qua phép vị tự tâm B tỷ số . Mà ∠ BPR = 90◦ nên suy ra BR là
cos A
đường kính của (ω )0 , kéo theo R cố định. Tương tự thì S cũng cố định nên O0 thuộc
cung chứa góc 180◦ − ∠ A dựng trên RS.

Bài 5. Cho số thực a 6= 0 và dãy (un ) thoả mãn u1 = 0, un+1 (un + a) = a + 1 với mọi
n ∈ Z+ . Tìm giới hạn của dãy (un ).

xn a+1 x
Lời giải. Đặt un = với n ≥ 1. Theo giả thiết thì un+1 = ⇒ n +1 =
yn un + a y n +1
( a + 1) y n
.
xn + ayn
Xét quan hệ truy hồi của hai dãy là xn+1 = ( a + 1)yn và yn+1 = xn + ayn thì

yn+2 = xn+1 + ayn+1 = ayn+1 + ( a + 1)yn .

a+1 ( a + 1)n − (−1)n


Vì u1 = 0, u2 = nên chọn y1 = 1, y2 = a; từ đó, ta có yn = với
a a+2
n ≥ 1.
Công thức trên chỉ xác định với a 6= −2 nên xét trường hợp a = −2, ta có dãy

 u1 = 0,
1 .
 u n +1 = ,n ≥ 1
2 − un

1
Bằng quy nạp, ta chứng minh được rằng un ∈ [0; 1) nên un+1 − un = − un =
2 − un
( u n − 1)2
> 0 nghĩa là dãy (un ) tăng, và dãy bị chặn trên bởi 1 nên có giới hạn, đặt là
2 − un
1
L ∈ (0, 1) thì giải ra ta có L = , tức là L = 1.
2−L
xn ( a + 1 ) y n −1
Tiếp theo, xét a 6= −2 ta có un = = =
yn yn
( a + 1)n + ( a + 1)(−1)n
với mọi n.
( a + 1)n − (−1)n
bn − b
Đặt −( a + 1) = b ∈ {−1; 1}, ta viết lại un = n với mọi n ≥ 1.
b −1
Nếu b > 1 hoặc b < −1, tương ứng là a < −2 hoặc a > 0, thì lim un = 1.
Nếu −1 < b < 1, tương ứng là −2 < a < 0, thì lim un = b = −( a + 1).
Vậy ta có kết luận sau:

• Nếu a ∈ (−2; 0) thì lim un = −( a + 1).


• Nếu a ∈
/ (−2; 0) thì lim un = −1.

Bài toán kết thúc.

Nhận xét. Cách giải trên hiệu quả và triệt để vì khi đã tìm được công thức tổng quát
của dãy thì mọi thứ sẽ rõ ràng hơn nhiều.

162
Dạng toán này không mới, tuy nhiên điểm khó ở đây là đề bài xét a 6= 0 chứ không
phải Và nếu như đã có a > 0 thì un > 0 nên bài toán sẽ trở thành dạng un+1 = f (un )
a+1
kinh điển, ở đây vì chưa biết a > 0 hay a < 0 nên nếu ngay từ đầu, ta xét f ( x ) =
x+a
thì chưa xác định được miền cụ thể của x; và cần phải làm rõ tại sao dãy số un luôn
xác định, tức là un 6= − a, ∀n. Trong lời giải trên, ta cũng thấy rằng số hạng đầu u1 = 0
cũng quan trọng và ảnh hưởng nhiều đến việc biện luận sau đó.
Một số bài toán tương tự:

a) Chứng minh rằng để dãy số xác định bởi


(
u1 = b,
,n ≥ 1
un+1 = u2n − (2a − 1)un + a2

hội tụ thì điều kiện của a, b là a − 1 ≤ b ≤ a.


b) (VMO 2000) Cho dãy số ( xn ) xác định bởi
(
x0 ∈ (0;p
c ),
√ , n ≥ 0.
x n +1 = c − c + x n

Tìm c để dãy số xác định với mọi n và tồn tại giới hạn hữu hạn lim xn .

Bài 6. Tìm tất cả các hàm số f : R→ R+ thoả mãn điều kiện


 
f x f (y2 ) − y f ( x2 ) = (y − x ) f ( xy) ∀ y > x > 0.

Lời giải. Để dễ theo dõi, ta chia lời giải thành các bước sau:
Bước 1. Hàm số f ( x ) đồng biến trên R+ .
Theo giả thiết thì với mọi y > x > 0 ta đều có

f ( y2 ) y
x f ( y2 ) − y f ( x 2 ) > 0 ⇒ 2
> > 1.
f (x ) x

Do đó, y2 > x2 ⇔ y > x ⇔ f (y2 ) > f ( x2 ) nên hàm f đã cho đồng biến trên R+ .
Bước 2. f (1) = 1.
Trong đề bài, thay y = x + 1, ta có
 
f x f (( x + 1)2 ) − ( x + 1) f ( x2 ) = f ( x ( x + 1))

hay
x f (( x + 1)2 ) − ( x + 1) f ( x2 ) = x ( x + 1)
f (( x + 1)2 ) f ( x2 )
⇔ = + 1, ∀ x > 0
x+1 x

163
Suy ra
f ( n2 ) f ((n − 1)2 ) f (1)
= +1 = ··· = + ( n − 1) = a + n − 1
n n−1 1
hay
f (n2 ) = n( a + n − 1) với mọi số nguyên dương n.
Trong đề bài, thay y = m2 , x = n2 với m, n ∈ Z+ sao cho m2 − n2 = p2 là số chính
phương (rõ ràng có vô hạn bộ như thế là các bộ ba Pythagoras), ta có
 
f n2 f ( m4 ) − m2 f ( n4 ) = ( m2 − n2 ) f ( m2 n2 )

⇔ f (m2 n2 p2 ) = p2 mn( a + mn − 1)
⇔ mnp( a + mnp − 1) = p2 mn( a + mn − 1)
⇔ ( p − 1)( a − 1) = 0
Đẳng thức cuối phải luôn đúng với mọi p được chọn như trên nên a = 1. Suy ra
f (1) = 1.
Bước 3. f ( x ) = x ∀ x ∈ Q.
Tiếp theo, cũng từ đẳng thức

f (( x + 1)2 ) f ( x2 )
= + 1, ∀ x > 0,
x+1 x
ta suy ra
f (( x + n)2 ) f ( x2 )
= + n, ∀ x > 0, n ∈ Z+
x+n x
hay
x f (( x + n)2 ) − ( x + n) f ( x2 ) = nx ( x + n).
Trong đề bài, thay y = x + n, ta có
 
f x f (( x + n)2 ) − ( x + n) f ( x2 ) = n f ( x ( x + n))

⇔ f (nx ( x + n)) = n f ( x ( x + n))


Với mọi n ∈ Z+ , y > 0, ta luôn chọn được x > 0 để x ( x + n) = y nên ta có

f (ny) = n f (y), ∀ n ∈ Z+ , y ∈ R+ .
   
1 1 1 1
Với mọi n ∈ Z+ cho y = , suy ra f (1) = n f ⇒ f = . Suy ra
n n n n
n  
1 n
f = nf = , ∀m, n ∈ Z+
m m m

164
hay f ( x ) = x, ∀ x ∈ Q+ .
Bước 4. f ( x ) = x ∀ x ∈ R+ .
Với mọi số thực x0 > 0, giả sử f ( x0 ) > x0 thì chọn hai dãy số hữu tỷ ( an , bn ) sao
cho an < x0 < bn và lim an = lim bn = x0 . Rõ rang f ( an ) < f ( x0 ) < f (bn ) ⇒ an <
f ( x0 ) < bn , cho n → ∞, ta có f ( x0 ) = x0 . Do đó, với mọi số thực x > 0 thì f ( x ) = x.
Thử lại ta thấy thoả vì
 
f x f (y ) − y f ( x ) = f ( xy2 − yx2 ) = xy2 − x2 y = (y − x ) xy = (y − x ) f ( xy).
2 2

Vậy tất cả các hàm số cần tìm là f ( x ) = x, ∀ x > 0.

Nhận xét. Bài toán dùng các kỹ thuật quen thuộc nhưng đòi hỏi nhiều bước nên thí
sinh cần chắc các phương trình hàm dạng này thì mới xử lý triệt để được. Hàm xác
định trên R+ và cho điều kiện các biến phân biệt cũng gây không ít khó khăn chỉ nắm
các phép thế trong phương trình hàm.
Hai mấu chốt quan trọng để giải quyết bài toán là:
• Hàm số f ( x ) đơn điệu thì f ( x ) = f (y) ⇒ x = y.
• Hàm số đơn điệu và cộng tính trên Q thì f ( x ) = ax ∀ x ∈ R.
Dưới đây là một số bài tương tự:
a) Tìm tất cả các hàm số f : R+ → R+ sao cho

f ( x + f (y)) = f ( x ) − x + f ( x + y) ∀ x, y > 0.

b) Tìm tất cả các hàm số f : R+ → R+ thoả

f ( f ( x ) + 2y) = f (2x + y) + 2y ∀ x, y > 0.

c) Cho hai hàm số f , g : R+ → R+ thoả mãn và với mọi x, y > 0 thì

f ( g( x ) + y) = f ( x ) + g(y)
.
g ( f ( x ) + y) = g( x ) + f (y)

Chứng minh rằng f ( x ) = g( x ), ∀ x > 0.

Bài 7. Cho n = 2018.2019. Gọi A là tập hợp tất cả các bộ số ( a1 ; a2 ; . . . ; an−1 ; an ) có tính
n
thứ tự sao cho ai ∈ {0; 1} ∀i = 1, 2, ..., n và ∑ ai = 20182. Có bao nhiêu bộ số như thế để
i =1
thoả mãn
k k
k k
∑ ai ≥ 2
và ∑ an−k+1 ≥ 2 với k = 1, 2, ..., n?
i =1 i =1

Lời giải. Ta giải bài toán tổng quát khi thay 2018 bởi m ∈ Z+ . Bài toán đã cho tương
đương với bài toán sau:

165
Trong hệ trục tọa độ Oxy xét lưới điểm nguyên trong hình chữ nhật có đỉnh
dưới bên trái là O(0, 0) và đỉnh trên bên phải là A(m2 ; m). Đặt B(m; m) và
C (m2 − m; 0), hỏi có bao nhiêu đường đi từ O → A sao cho mỗi bước, ta đi
sang phải hoặc lên trên 1 đơn vị, gọi là đường đi đơn, và không vượt lên trên
OB cũng như không xuống dưới AC?

Ở đây, các số 0; 1 tương ứng với các bước đi lên trên, các bước đi sang phải; còn điều
k
kiện tổng k số đầu và tổng k số cuối không nhỏ hơn tương ứng với số lượng bước
2
đi lên không vượt quá số lượng bước đi sang phải.
Để thuận tiện, ta gọi đường đi cắt d nếu nó có các phần nằm về cả hai phía của d.
Trước hết, ta sẽ chứng minh bổ đề sau:
m +1
Bổ đề. Số đường đi đơn từ O → A(m; n), có cắt đường thẳng y = x, là Cm +n .

Trong hình trên, đường cũ là đứt nét, còn đường mới là liền nét.

Thật vậy, xét đường thẳng (d) : y = x + 1, rõ ràng các đường đi đơn cắt y = x đều
sẽ có điểm chung với đường thẳng (d) này. Tại các điểm chung đó, ta thực hiện đối
xứng trụcl đểmđược một đường đi mới xuất phát từ O → A0 (n − 1, m + 1).
Rõ ràng phép đối xứng trục trên là song ánh, biến các đường cần tìm (cắt y = x),
n −1
thành các đường từ O → A0 ; do đó, số lượng đường cần tìm là Cm +n .
2 m
Trở lại bài toán. Số đường đi đơn từ O → A(m ; m) là Cm2 +m vì nó bằng số cách chọn
m lần đi lên trong tổng số m2 + m lần di chuyển, trong đó số đường đi cắt OB bằng
m −1
số đường đi cắt AC và bằng Cm 2 + m (theo bổ đề).

Do đó, ta chỉ cần tìm số đường đi cắt cả OB, AC với ý tưởng đối xứng hai lần đã dùng
để chứng minh bổ đề.
Đầu tiên, ta thực hiện đối xứng qua đường thẳng y = x + 1; khi đó, các đường đi đơn
sẽ xuất phát từ O → A0 (m − 1; m2 + 1). Do các đường ban đầu còn vượt qua AC nên
các đường mới phải cắt thêm y = x + m2 − m + 3. Tiếp tục đối xứng qua đường thẳng
này, ta đưa về đếm số đường đi đơn từ O → A00 (m − 2, m2 + 2). Suy ra số đường đi
m −2
trong trường hợp này là Cm 2 +m .

166
m −2
Vậy theo nguyên lý bù trừ, kết quả cần tìm sẽ là Cm 2 + m . Thay m = 2018, ta có số lượng

đường đi, cũng chính là số bộ thoả mãn đề bài.

Nhận xét. Bài toán thoạt nhìn có vẻ có thể xử lý được bằng truy hồi hoặc bù trừ trực
tiếp, nhưng quả thật không dễ. Việc tiếp cận theo hướng dùng “lưới nguyên” đòi
hỏi ít nhiều kinh nghiệm và các kỹ thuật liên quan, vì bài toán này sau khi mô hình
hóa xong còn phải thêm bước “đối xứng hai lần” mới có thể giải quyết triệt để được.
Đường đi trong bài toán còn gọi là đường đi Dyck hoặc Catalan, liên quan đến các bài
toán nổi tiếng như: bỏ phiếu bầu cho các ứng viên sao cho ứng viên này luôn thắng
ứng viên kia tại mọi thời điểm, mua vé với tiền 1 đồng, 2 đồng sao cho không có ai
cần phải chờ tiền thối lại,. . .
Bài toán cũng có thể mô phỏng theo dãy các đường đi chéo, lên hoặc xuống 1 đơn vị,
từ (0, 0) đến (n2 + n; n2 − n) sao cho đường đi không xuống dưới trục hoành và cũng
không vượt lên trên y = n2 . Thực ra nếu trình bày theo hướng này thì việc lấy đối
xứng sáng sủa hơn.
Ứng với n = 1, 2, 3, 4 ta có các giá trị 0, 4, 100, 2755 là một dãy số không quen thuộc
nên chúng tôi cũng không rút gọn đáp số trên.
Dưới đây là một số kết quả tương tự về đường đi đơn trong đề bài:

a) Số đường đi đơn từ (0; 0) → (m; n) mà không có điểm chung với y = x là


m−n m
C .
m + n m+n
n n −1
b) Số đường đi đơn từ (0; 0) → (m; n) mà không vượt qua y = x là Cm +n − Cm+n .
c) Số đường đi gồm n bước mà không vượt qua y = x là
n n!(2i + 1 − n) [n/2]
∑ = Cn .
i =n/2 (i + 1) ! ( n − i ) !
d) Số đường đi đơn từ (0; 0) → (m; n) mà không có điểm chung với y = x + t là
n m−t
Cm +n − Cm+n .
e) (Việt Nam TST 2003) Tính số cặp đường đi đơn (0; 0) → A(m, n) và B( p; 0) →
C (m; q) với p < m; q < n sao cho chúng không có điểm chung.

Bài 8. Cho đường tròn (C ) có tâm I nội tiếp tam giác ABC và tiếp xúc với các cạnh AB, AC
tại E, F. Gọi AM, AN là phân giác trong và ngoài của tam giác với M, N ∈ BC. Gọi d M , d N
lần lượt là tiếp tuyến khác BC của (C ) và đi qua M, N.

a) Chứng minh rằng d M , d N và EF đồng quy. Gọi điểm đồng quy đó là D.


b) Trên AB, AC lần lượt lấy P, Q thoả mãn DP k AC, DQ k AB. Gọi R, S lần lượt là
trung điểm DE, DF. Chứng minh rằng I thuộc đường thẳng qua trực tâm của các tam
giác DPS, DQR.

Lời giải.

a) Gọi X, Y lần lượt là tiếp điểm của tiếp tuyến thứ hai kẻ từ M đến I và D 0 là tiếp
điểm của ( I ) trên BC. Gọi K là trung điểm EF.

167
Xét trong đường tròn ( I ) thì EF là đường đối cực của A và K ∈ EF nên đối cực
của K sẽ đi qua A, mà N A ⊥ I A nên N A chính là đường đối cực của K.
Đường đối cực của K đi qua N nên đối cực của N là D 0 Y sẽ đi qua K. Dễ thấy rằng
AM là trục đối xứng của tứ giác D 0 EF nên suy ra D 0 X ⊥ EF. Xét D 0 ( EF, XY ),
ta có D 0 Y đi qua trung điểm của EF và D 0 X ⊥ EF nên D 0 ( EF, XY ) = −1 hay tứ
giác EXFY điều hòa. Suy ra MX, NY, EF đồng quy. Ngoài ra ta cũng có X, Y, A
thẳng hàng.
b) Dễ thấy các tam giác PED và DQF là các tam giác cân. Gọi H1 , H2 lần lượt là
trực tâm của tam giác DPS, DQR. Ta có ∠ PH1 S = ∠ PDF = ∠ AFE = ∠ PES
nên EPSH1 là tứ giác nội tiếp. Suy ra

RH1 · RP = RS · RE.

Ngoài ra, KA · KI = KE · KF nên

RP RH1 RE RS
· = · .
KA KI KE KF
RP RE RH1 RS
Theo định lý Thales thì = nên = , mà
KA KE KI KF
DE − DF EF
RS = RD − SD = = = KF
2 2

nên RH1 = KI, mà RH1 k KI (do cùng vuông góc với EF) nên IKRH1 là hình
chữ nhật, kéo theo I H1 k EF. Một cách tương tự, ta có I H2 k EF nên H1 H2 đi
qua I.

Bài toán kết thúc.

Nhận xét. Câu a của bài toán đơn giản nhưng nhiều kỹ thuật, câu b thì phức tạp
nhưng lại giải được nhẹ nhàng. Bài toán có nhiều ý mới, đẹp và thú vị. Mô hình trên
còn có nhiều ý để khai thác:

• MX là trục đẳng phương của ( I ) và đường tròn bàng tiếp góc A là ( Ia ).


• Đường trung bình của tam giác DMN đi qua K.
• Điểm I thuộc cả hai đường tròn ( PES), ( QFR).

Chú ý rằng câu a của bài toán vẫn đúng khi thay M, N bởi cặp điểm liên hợp điều
hòa tuỳ ý với B, C ; không nhất thiết phải là chân các đường phân giác.
Ngoài cách dùng đối cực, ta có thể dùng bổ đề: Tam giác ABC có ( I ) nội tiếp tiếp xúc
với AB, AC ở D, E thì BI, CI cắt DE tại các điểm thuộc đường tròn đường kính BC.
Khi đó, bài toán sẽ được giải quyết nhẹ nhàng hơn nhiều.

168
GIỚI THIỆU MỘT SỐ BÀI TOÁN HAY
Ban biên tập

GIỚI THIỆU. Trong phần này, chúng tôi giới thiệu một số bài toán sơ cấp hay và
thú vị để bạn đọc tự luyện tập. Các bài toán được giới thiệu trong chuyên đề này
đến từ đủ 4 phân môn chính là Đại số, Hình học, Số học và Tổ hợp. Một số bài toán
được tham khảo từ những tài liệu khác đều được trích dẫn nguồn gốc rõ ràng.

1. ĐẠI SỐ
Bài toán 1 (PTNK 2016). Tìm số nguyên dương k nhỏ nhất để bất đẳng thức sau đúng:

x k yk zk ( x3 + y3 + z3 ) ≤ 3 với mọi x, y, z > 0 và x + y + z = 3.

Bài toán 2 (IMO Shortlist 2016). Tìm tất cả các hàm số f : R+ → R+ thoả mãn:
 
2 2 2
x f ( x ) f ( f (y)) + f (y f ( x )) = f ( xy) f ( f ( x )) + f ( f (y )) ∀ x, y ∈ R+

Bài toán 3 (ELMO Shortlist 2017). Tìm tất cả các hàm số f : R → R thoả mãn đồng thời
các điều kiện sau:

• Nếu a + b + c ≥ 0 thì f ( a3 ) + f (b3 ) + f (c3 ) ≥ f (3abc).


• Nếu a + b + c ≤ 0 thì f ( a3 ) + f (b3 ) + f (c3 ) ≤ f (3abc).

Bài toán 4 (Hàn Quốc 2017). Với mỗi số nguyên dương n, đặt cn = 2017n . Cho một hàm
số f : N → R thoả mãn đồng thời các điều kiện sau:

• Với mọi số nguyên dương m, n thì f (m + n) ≤ 2017 f (m) f (n + 325).


• Với mọi số nguyên dương n thì 0 < f (cn+1 ) < f (cn )2017 .

Chứng minh rằng tồn tại dãy số a1 , a2 , ... thoả mãn: ∀n, k mà ak < n thì f (n)ck < f (ck )n .

Bài toán 5 (Russia 2014). Một tập hợp hữu hạn điểm nằm trên mặt phẳng toạ độ được gọi
là thích hợp nếu chúng có hoành độ đôi một khác nhau và mỗi điểm được tô bởi một trong
hai màu: xanh hoặc đỏ. Ta nói đồ thị của một đa thức phân tách tập điểm đó nếu ở phần mặt
phẳng phía trên của đồ thị chỉ có các điểm thuộc cùng một màu và ở phần mặt phẳng phía
dưới của đồ thị chỉ có các điểm thuộc màu còn lại. Ngay trên đồ thị có thể có các điểm thuộc cả

169
hai màu. Với mỗi số tự nhiên n > 1, hãy xác định số k nhỏ nhất với tính chất: với mọi bộ n
điểm thích hợp, luôn tồn tại đa thức bậc không quá k mà đồ thị của nó phân tách tập điểm này.

2. HÌNH HỌC
Bài toán 6. Cho tam giác ABC nội tiếp (O). Tiếp tuyến tại B, C của (O) cắt nhau tại X.
Các điểm Y, Z xác định tương tự. Gọi A1 , B1 , C1 lần lượt nằm trên OX, OY, OZ sao cho:

A1 O BO CO
= 1 = 1 =k
A1 X B1 Y C1 Z

a) Chứng minh rằng AA1 , BB1 , CC1 đồng quy tại N.


A2 O
b) Gọi A2 là điểm nằm trên OA sao cho = k. Đường thẳng d a qua A2 vuông góc với
A2 A
OA cắt BC tại A3 . Các điểm B3 , C3 xác định tương tự. Chứng minh rằng A3 , B3 , C3
cùng nằm trên một đường thẳng vuông góc với ON.

Bài toán 7 (Nguyễn Tiến Hoàng). Cho tam giác ABC. D là điểm bất kỳ trên đường thẳng
BC. X là điểm đối xứng của A qua D. Y, Z lần lượt là điểm đối xứng của D qua B, C. Gọi T
là giao điểm thứ hai của ( XBY ) và ( XCZ ).

a) Chứng minh rằng XT luôn đi qua điểm P cố định khi D thay đổi.
b) Chứng minh rằng XT là trục đẳng phương của các đường tròn ( PBY ) và ( PCZ ).

Bài toán 8 (Nguyễn Tăng Vũ). Cho tam giác ABC nội tiếp (O) có M, N là trung điểm
CA, AB. BM, CN cắt lại (O) tại X, Y. Gọi (K ) là đường tròn đi qua X và tiếp xúc với CA
tại C. ( L) là đường tròn đi qua Y và tiếp xúc với AB tại B. Gọi P là tâm đường tròn Euler
của tam giác ABC. Chứng minh rằng ( PCK ) và ( PBL) cắt nhau trên đường tròn (O).

Bài toán 9. Cho tam giác ABC có (ω ) là đường tròn Euler. Chứng minh rằng tồn tại đúng
3 điểm nằm trên đường tròn ngoại tiếp tam giác ABC sao cho đường thẳng Simson của điểm
đó với tam giác ABC thì tiếp xúc với (ω ).

Bài toán 10 (Trần Quang Hùng). Cho tam giác ABC có K, L là tâm bàng tiếp góc B, C.
Lấy P, Q trên CK, BL sao cho LP k AC, KQ k AB. PQ cắt KL tại R. H là hình chiếu của R
trên đường thẳng OI của tam giác ABC. AD là phân giác của tam giác ABC. G nằm trên
AD sao cho KG ⊥ GA. HB, HC cắt ( HGD ) tại M, N khác H. DN, DM cắt CA, AB tại
E, F. Chứng minh rằng A, E, H, F thuộc cùng một đường tròn.

3. SỐ HỌC
Bài toán 11. Cho đa thức P( x ) = a0 + a1 x + ... + an x n ( ai ∈ Z, i ∈ {0; 1; ...; n}). Giả sử
tồn tại số nguyên dương m thoả mãn đồng thời các điều kiện sau:

• a2 , a3 , ..., an chia hết cho tất cả các ước nguyên tố của m.


• a1 và m là hai số nguyên tố cùng nhau.

170
Chứng minh rằng ∀k ∈ N∗ , tồn tại số nguyên dương c để P(c) là bội của mk .

Bài toán 12. Cho ( an ) là dãy các số nguyên dương sao cho k số hạng đầu tiên a1 , ..., ak là
các số nguyên dương phân biệt. Với mỗi n > k, ta xác định an là số nguyên dương nhỏ nhất
không thể biểu diễn được như tổng của một vài số (có thể một) số trong a1 , ..., an−1 . Chứng
minh rằng với mỗi n đủ lớn ta có an = 2an−1 .

Bài toán 13 (IMO 2016). Cho P = A1 A2 ...Ak là một đa giác lồi trong mặt phẳng tọa độ.
Tất cả các đỉnh A1 , A2 , ..., Ak đều có tọa độ nguyên và cùng nằm trên một đường tròn. Kí
hiệu S là diện tích của P. Cho n là một số nguyên dương lẻ mà bình phương độ dài mỗi cạnh
của P là một số nguyên chia hết cho n. Chứng minh rằng 2S là một số nguyên chia hết cho n.

Bài toán 14 (Serbia TST 2017). Hàm số f : N → N được gọi là tốt nếu như:

f a (b) = f ( a + b − 1) ∀ a, b ∈ N; a > 0

Với ký hiệu f a (b) là tác động hàm f đúng a lần. Gọi g là một hàm số tốt thoả mãn bài toán,
và đồng thời giả sử rằng tồn tại số nguyên A sao cho g( A + 2018) = g( A) + 1.

a) Chứng minh rằng g(n + 20172017 ) = g(n) ∀n ≥ A + 2.


b) Nếu như g( A + 1) 6= g( A + 1 + 20172017 ), hãy xác định g(n) với n < A.

Bài toán 15 (Serbia TST 2017). Với k là số nguyên dương, gọi n là số nguyên dương nhỏ
nhất có đúng k ước số. Giả sử rằng n là một lập phương đúng. Liệu n có ước nguyên tố dạng
3j + 2 hay không ? Hãy chứng minh.

4. TỔ HỢP
Bài toán 16. Cho 2017 số thực có tổng bằng 0. Hỏi có ít nhất bao nhiêu cặp số từ 2017 số
cho trước mà có tổng là một số không âm ? Hãy chứng minh.

Bài toán 17. (China TST 2017) Có 2017 kỹ sư tham gia một hội nghị. Với 2 kỹ sư bất kỳ, khi
họ trò chuyện với nhau, họ chỉ sử dụng tiếng Trung hoặc tiếng Anh. 2 kỹ sư bất kỳ thì trò
chuyện với nhau không quá 1 lần. Biết rằng với 4 kỹ sư bất kỳ thì ta luôn có:

• Số cuộc trò chuyện giữa họ luôn là số chẵn.


• Có ít nhất 1 cuộc trò chuyện được diễn ra bằng tiếng Trung.
• Hoặc là không có cuộc trò chuyện nào bằng tiếng Anh, hoặc là số cuộc trò chuyện bằng
tiếng Anh không ít hơn số cuộc trò chuyện bằng tiếng Trung.

Chứng minh rằng tồn tại 673 kỹ sư mà 2 người bất kỳ trò chuyện với nhau bằng tiếng Trung.

Bài toán 18. Một nhà ảo thuật cùng với trợ lý của mình biểu diễn một màn ảo thuật như
sau. Họ dùng một bộ bài tây gồm 52 lá bài. Các lá bài đều được lật ngửa. Người trợ lý bịt mắt
nhà ảo thuật gia sau đó mời một khán giả chọn 13 lá bài bất kì và công bố cho tất cả mọi người
biết (trừ nhà ảo thuật). Người trợ lý sau đó sẽ chọn ra 13 lá trong số lá bài còn lại và đưa cho

171
người khán giả. Người khán giả sẽ trộn tất cả 26 lá bài này theo cách tùy ý rồi đưa cho nhà ảo
thuật. Thật ngạc nhiên là nhà ảo thuật nói đúng tất cả 13 lá bài mà người khán giả đã chọn.
Hỏi nhà ảo thuật và người trợ lý đã làm thế nào ?

Bài toán 19. Có 2n tấm thẻ, mỗi tấm ghi một số 0. Hai bạn An và Bình cùng chơi một trò
chơi như sau. Ở mỗi lượt An chia 2n tấm thẻ ra hai phần, mỗi phần gồm n tấm thẻ. Bình được
phép chọn ra một trong hai phần, công thêm 1 vào các số được trên các tấm được chọn và trừ
đi một vào các số ở trên các tấm thẻ còn lại. Chứng minh rằng Bình
 có chiến
 thuật sao cho tại
2n − 2
mọi thời điểm thì số viết trên mỗi tấm thẻ đều không vượt quá .
n−1

Bài toán 20. Cho bảng ô vuông kích thước n × n. Ban đầu tất cả các ô vuông đơn vị được tô
màu trắng. Ta tô màu đen một số ô vuông đơn vị sao cho với một ô màu trắng bất kỳ thì có
chung cạnh với ít nhất một ô màu đen. Xác định số lượng ô màu đen ít nhất trên bảng.

172

You might also like